Está en la página 1de 135

An alisis Num erico II

Apuntes Curso C odigo 525441 Primer Semestre 2011 Raimund B urger & Rommel Bustinza Centro de Investigaci on en Ingenier a Matem atica (CI2 MA) & Departamento de Ingenier a Matem atica Facultad de Ciencias F sicas y Matem aticas Universidad de Concepci on Casilla 160-C Concepci on, Chile

23 de diciembre de 2011

Indice general
Cap tulo 1. Conceptos b asicos Cap tulo 2. M etodos directos para la soluci on de sistemas lineales (Parte I) 2.1. Sistemas lineales escalonados. Matrices triangulares y su inversi on 2.2. El m etodo de eliminaci on de Gauss 2.3. Descripci on matricial del algoritmo de Gauss y el teorema LR 2.4. La descomposici on de Cholesky 2.5. Aplicaciones de la descomposici on triangular y casos especiales Cap tulo 3. M etodos directos para la soluci on de sistemas lineales (Parte II) 3.1. Normas de vectores y matrices 3.2. El problema de la sensitividad para un sistema lineal 3.3. El m etodo de cuadrados m nimos y la transformaci on de una matriz n n a una matriz triangular superior Cap tulo 4. M etodos iterativos para la soluci on de sistemas de ecuaciones lineales 4.1. Un ejemplo 4.2. Metodolog a general del desarrollo de m etodos iterativos 4.3. Teoremas de convergencia para m etodos iterativos 4.4. M etodos de iteraci on por bloque 4.5. El m etodo de gradientes conjugados (cg) de Hestenes y Stiefel Cap tulo 5. El problema de valores propios de una matriz 5.1. La localizaci on de valores propios y la sensitividad del problema 5.2. Transformaci on de similaridad unitaria de una matriz n n a una forma de Hessenberg o tridiagonal 5.3. Computaci on de los valores propios de una matriz tridiagonal hermitiana 5.4. Determinaci on de los vectores propios de una matriz tridiagonal hermitiana 5.5. Valores propios de una matriz de tipo Hessenberg 5.6. La iteraci on directa seg un von Mises y el m etodo de Wielandt 5 11 12 13 19 29 35 37 37 44 61 73 73 74 79 95 96 107 107 115 118 124 126 127

Indice general

Cap tulo 1

Conceptos b asicos
Designaremos por Cnn el espacio de las matrices cuadradas de orden n en el cuerpo C, mientras que cuando los coecientes pertenezcan al cuerpo R, usaremos la notaci on Rnn . Denici on 1.1. Para A Cnn se denen la matriz transpuesta como la matriz B de elementos bij := aji , y la matriz conjugada transpuesta de A como la matriz C de elementos cij := a ji . Notaci on: AT := B y A := C. Denici on 1.2. Una matriz A Cnn se dice sim etrica si A = AT , hermitiana si A = A , ortogonal si AAT = AT A = I y unitaria si AA = A A = I. Una manera de caracterizar la ortogonalidad, respectivamente la unitariedad, de una matriz A es a trav es de las igualdades A1 = AT y A1 = A , respectivamente. Denici on 1.3. Un escalar C se dice un valor propio de una matriz A Cnn si existe n x C , x = 0 tal que Ax = x. En tal caso, el vector x se llama vector propio de A asociado a . Denici on 1.4. Sea C un valor propio de A. Se llama espacio propio asociado a al conjunto L() := {x Cn | Ax = x}. Note que L() contiene, adem as del vector nulo, a todos los vectores propios asociados a . Se puede demostrar que L() es un subespacio vectorial de Cn con dimensi on () dada por () = n rango(A I). El n umero () se llama tambi en multiplicidad geom etrica de . Lema 1.1. Sea A Cnn . Un escalar C es un valor propio de A si y s olo si det(A I) = 0.
5

(1.1)

(1.2)

1. CONCEPTOS BASICOS

Demostraci on. De acuerdo a la Denici on 1.3, un escalar C es un valor propio de A si y s olo si existe x Cn , x = 0 tal que Ax = x, equivalentemente, si y s olo si (A I)x = 0 con x = 0. Esta u ltima relaci on es un sistema lineal de ecuaciones lineales homog eneo de n ecuaciones y n inc ognitas. Para no obtener u nicamente la soluci on trivial x = 0, que no nos interesa, imponemos la condici on necesaria y suciente det(A I) = 0. La expresi on fA () := det(A I) es un polinomio de grado n que se llama polinomio caracter stico de A, y tiene la forma fA () = (1)n (n + n1 n1 + + 1 + 0 ). (1.3)

Si 1 , 2 , . . . , k son los ceros del polinomio caracter stico, entonces fA () puede factorizarse como fA () = (1)n ( 1 )1 ( 2 )2 . . . ( k )k , donde 1 , . . . , k son n umeros naturales tales que 1 + + k = n. El n umero i , i = 1, . . . , k de veces que se repite el factor ( i ) se llama multiplicidad algebraica de i . Al valor propio i pueden corresponder a lo m as i vectores propios linealmente independientes. El n umero de vectores propios de A asociados al valor propio i , y que son linealmente independientes, es igual a (i ). En otras palabras, se tiene que (i ) i , i = 1, . . . , k. (1.5) (1.4)

Ejemplo 1.1. La matriz diagonal de orden n, D := I con C, tiene el polinomio caracter stico fD () = ( )n . Luego = , u nico valor propio de D, tiene multiplicidad algebraica n y multiplicidad geom etrica n. Esto indica que L() = Cn , es decir, todo vector x Cn , x = 0 es vector propio de D asociado a . Ejemplo 1.2. Consideremos la matriz A Rnn 1 0 0 1 . . . A = . . .. .. . ... . . 0 ... dada por ... 0 ... . . . .. . 0 , 1 ... 0

1. CONCEPTOS BASICOS

con C, que tiene el mismo polinmomio caracter stico que la matriz del ejemplo anterior, en efecto, fA () = 0 . . . . . . 0 1 0 1 .. .. .. = ( )n . . . . 0 .. . 1 0 ... 0 . . .

En este caso, el u nico valor propio de A, = , tiene multiplicidad algebraica n, mientras que su multiplicidad geom etrica es () = 1. En efecto, de la Denici on 1.4 tenemos L() = x Cn | = x Cn (A I)x = 0 0 x1 0 1 ... 0 x2 0 0 1 . . . . . = . .. .. . , . 0 0 1 xn1 0 0 0 xn

esto es,

Lo anterior muestra que () = dim L() = 1.

L() = {x Cn | x2 = 0, . . . , xn = 0} = x = (x1 , 0, . . . , 0)T | x1 C .

Denici on 1.5. Sean A y B Cnn . Las matrices A y B se dicen similares si existe una matriz P Cnn invertible tal que B = P1 AP.

(1.6)

Lema 1.2. Sean A y B Cnn . Si A y B son similares, entonces ellas tienen los mismos n valores propios, contando su multiplicidad algebraica. Adem as, si x es un vector propio de 1 A, entonces P x es vector propio de B, con P que satisface (1.6). Demostraci on. Como A y B son similares, existe P invertible tal que B = P1 AP. De lo anterior se deduce que A = PBP1 y luego Puesto que entonces fA () = det(A I) = det(PBP1 PP1 ) = det P(B I)P1 . A, B Cnn : det(AB) = det(A) det(B),

det(P) det(P1 ) = det(PP1 ) = det(I) = 1,

1. CONCEPTOS BASICOS

y en consecuencia, Eso muestra que A y B tienen el mismo polinomio caracter stico y por lo tanto los mismos n valores propios, contando su multiplicidad algebraica. Consideremos ahora un valor propio de A y un vector propio x asociado a . Multiplicando a la izquierda por P1 la ecuaci on Ax = x obtenemos P1 Ax = (P1 x). Por otra parte, P1 Ax = P1 A(PP1 )x = (P1 AP)(P1 x), lo cual, en virtud de la igualdad B = P1 AP, conduce a P1 Ax = B(P1 x). Se sigue de las igualdades (1.7) y (1.8) que B(P1 x) = (P1 x). Notando que P1 x = 0, concluimos que P1 x es un vector propio de B asociado al valor propio . Denici on 1.6. Sea B = {u1 , . . . , un } Cn . Se dice que B es una base ortonormal de Cn si u i uj = ij = 1 para i = j , 0 para i = j . (1.8) (1.7) fA () = det(P) det(B I) det(P1 ) = det(B I) = fB ().

Teorema 1.1 (Forma normal de Schur). Sea A Cnn . Entonces existen matrices U, T Cnn , U unitaria y T triangular superior, tales que T = U AU = U1 AU. Es decir, A es unitariamente similar a una matriz triangular superior. Demostraci on. Procedemos por inducci on sobre el orden n de la matriz A. Para n = 1, es trivial porque basta elegir U = 1 y T = A. Supongamos que el resultado es v alido para todas las matrices de orden k 1. Probemos que es cierto para todas las matrices de orden k . Sea A Ckk y consideremos un valor propio 1 de A y u(1) un vector propio asociado elegido de manera que u(1)
2 2

(1.9)

= (u(1) ) u(1) = 1.

Ahora, de acuerdo al Teorema de Completaci on de Base, podemos elegir una base ortonormal de Ck que contenga a u(1) , digamos B = {u(1) , . . . , u(k) }, y denir la matriz unitaria P1 Ckk como A continuaci on consideremos la matriz B1 = P 1 AP1 . Notemos primero que AP1 = Au(1) Au(2) Au(k) = 1 u(1) v(2) v(k) , (1.10) P1 := u(1) u(2) u(k) .

1. CONCEPTOS BASICOS

donde v(j ) := Au(j ) para j = 2, . . . , k . Al multiplicar por la izquierda (1.10) por P 1 se obtiene
(1) Como P es la primera columna de P1 , entonces 1 P1 = I y dado que u 1 0 (1) , P = 1u . . . (1) B1 = P v(2) 1 1 u (2) (1) P v(k) = 1 P 1v 1u (k) . P 1v

y por lo tanto

donde A2 C(k1)(k1) y 2 , . . . , k son escalares en C. Aplicamos la hip otesis de inducci on 2 C(k1)(k1) , P 2 unitaria, tal que para concluir que existe P A2 P 2 = T , P 2 (1.11) triangular superior. Entonces, al denir la matriz P2 Ckk por con T 1 0 0 0 , P2 := . . . P
2

1 2 k 0 , B1 = . . . A2 0

obtenemos

As , P2 es unitaria y adem as satisface 1 0 0 1 2 k 0 0 P2 P B P = . . 1 2 2 . P 2 . . P . A 2 2 0 0 1 2 k 0 P2 = . . A2 . P 2 0

1 0 0 0 = I. P P = . 2 2 . P 2 . P 2 0

10

1. CONCEPTOS BASICOS

Al realizar la multiplicaci on de matrices indicada y usando (1.11) llegamos a 1 2 k 1 2 k 0 0 =: T, B P = = P . . 1 2 2 . . 2 A2 P . P . T 2 0 0 (j ) , j = (2 , . . . , k )P 2 (j ) es la columna j de P 2. donde P 2 Puesto que
T = P 2 B1 P2 = P2 (P1 AP1 )P2 = (P1 P2 ) A(P1 P2 ),

1 0 0 1 2 k 0 0 . . = . . . 2 A2 . P . P 2 0 0

donde T es una matriz triangular superior y los j , j = 2, . . . , k , est an dados por

podemos elegir U como la matriz unitaria U = P1 P2 con lo cual T = U AU. Del principio de inducci on se concluye la validez del teorema.

Cap tulo 2

M etodos directos para la soluci on de sistemas lineales (Parte I)


En este cap tulo se consideran m etodos directos para la soluci on de sistemas lineales Ax = b, A Knn , b Kn , K = R o K = C, (2.1)

donde se supone que det(A) = 0. (Un m etodo directo entrega la soluci on exacta del problema en un n umero nito de pasos, al contrario de los m etodos iterativos, que se estudiar an m as adelante.) Te oricamente, la soluci on de (2.1) est a dada por x = (1 , . . . , n )T con i = donde A = a1 an . det a1 ai1 b ai+1 det(A) an , i = 1, . . . , n,

Esta regla es conocida como regla de Cramer. Practicamente, s olo en el caso n = 2 o para matrices A especiales, la f ormula es util por razones de esfuerzo computacional y la acumulaci on de errores de redondeo. El problema (2.1) nos lleva al problema m as general AX = B, el cual incluye el problema de AX = I de encontrar la inversa de A. Para resolver (2.2), tomamos en cuenta que este problema representa la soluci on simult anea de p problemas del tipo (2.1), dado que para X = x1 tenemos que AX = B Axi = bi , AX = I Axi = ei ,
11

A Knn ,

X Knp ,

B Knp ,

(2.2)

xp ,

B = b1

bp ,

i = 1, . . . , p;

el problema de encontrar A1 es equivalente a n problemas del tipo (2.1), dado que i = 1, . . . , n,

12

DE SISTEMAS LINEALES (PARTE I) 2. METODOS DIRECTOS PARA LA SOLUCION

con X = x1

xn y 0 . . . 0 ei := 1 i. 0 . . . 0

(2.3)

2.1. Sistemas lineales escalonados. Matrices triangulares y su inversi on Denici on 2.1. Sea A = (ij ) Knn . Si ij = 0 para j < i, entonces A se llama matriz triangular superior; si ij = 0 para i < j , entonces A se llama matriz triangular inferior. Un sistema lineal con una matriz triangular se llama escalonado. Los sistemas escalonados juegan un rol importante, dado que sus soluciones pueden ser determinadas f acilmente, por ejemplo en el caso de una matriz A triangular superior: 11 x1 + 12 x2 + + 1n xn = b1 , 22 x2 + + 2n xn = b2 , . . . n1,n1 xn1 + n1,n xn = bn1 , nn xn = bn .

(2.4)

Se usa la u ltima ecuaci on para calcular xn = bn /nn , luego se remplaza xn en la pen ultima ecuaci on para determinar xn1 , etc etera. Recordamos que para una matriz A triangular,
n

det(A) =
i=1

ii = 0 i = 1, . . . , n :

ii = 0.

Una matriz triangular puede ser invertida f acilmente resolviendo los n sistemas lineales con las n columnas unitarias. Dado que la inversa nuevamente es una matriz triangular del mismo tipo, resultan simplicaciones signicativas. Considerando el sistema Rx = ei , nos damos cuenta que x no depende de las columnas i +1, . . . , n de R. Entonces, si particionamos la matriz R como R= R11 r , 0 R11 K(n1)(n1) ,
1 R 11 0 1

r Kn1 ,

K,

esta observaci on se reeja en la f ormula R1 =


1 1 R 11 r

Eso signica que para la implementaci on de la inversi on de una matriz triangular superior, podemos sucesivamente remplazar las columnas n, n 1, . . . , 2, 1 de R por las columnas de R1 .

DE GAUSS 2.2. EL METODO DE ELIMINACION

13

2.2. El m etodo de eliminaci on de Gauss La idea del m etodo de eliminaci on de Gauss consiste en transformar un sistema arbitrario con una matriz n n regular en un sistema con una matriz triangular superior, usando a lo m as n 1 pasos de transformaci on de equivalencia: . . . . . . . . . 0 . . . . . . 0 . . . . . 0 . . . . . . . . . . . . . . . . . . . = . . . . . . . . . . . . . . . . . . 0 . . . . 0 . . . . . . . . . = . . . . . . . . . . 0 . . . . . . . . . . . . . 0 0 . . . . . . . . . . . . . . . = . . . . . . . . . . . . . . . . . . . . . . . . .. . = . . . . . . . . . ... . . . . 0

. 0 . . . . . . . . 0

En esta representaci on esquem atica del algoritmo, el s mbolo representa un elemento que puede asumir un valor diferente de cero, mientras que por 0 se marca cualquier elemento que debe asumir el valor cero debido a la denici on del algoritmo. En el i- esimo paso, i = 1, . . . , n 1, usamos las siguientes transformaciones: a) Si es necesario, intercambiamos la la i con una de las las i + 1, . . . , n del sistema. b) Si as se desea, intercambiamos la columna i con alguna de las columnas i +1, . . . , n del sistema. Tal medida sirve para reducir el efecto de acumulaci on de errores de redondeo. c) Sustracci on de m ultiplos apropiados de la la i de las las i + 1, . . . , n. Para la administraci on de los pasos, usaremos el siguiente esquema, que tambien incluye los n umeros de las y columnas. Sean ij := ij ,
(1)

i, j = 1, . . . , n;

(1)

:= i ,

i = 1, . . . , n.

(2.5)

Al iniciarse la computaci on, el esquema est a dado por 1


(1)

2
(1)

n 1n . . .
(1)

1 11 12 . . . . . . (1) (1) n n1 n2

1 . . .

(1)

nn n

(1)

(1)

14

DE SISTEMAS LINEALES (PARTE I) 2. METODOS DIRECTOS PARA LA SOLUCION

Despu es de i 1 pasos, el esquema asume la siguiente forma: 1 1 . . . . . . . . .


(1) (1)

11 0 . . . . . . 0 0 . . . 0

(1)

.. . ...
(i)

i1 . . .

(i1)

(i)

1,i1

(i1)

1,i . . .

( i)

n . . .

(i)

1,n

(i)

(i1) i 1 (i) i


(i1)

0 0 . . . 0

(i1) i1,i1

0 . . . 0

(i) i1,i ( i) ii

(i) i1,n ( i) in

. . .

(1) 1 . . . . . . . . . . (i1)
i1

. . .

n Aqu

(i)

. . . ( i) ni
(1)

i . . .

(i)

nn
(i)

(i)

(i)

(i) 1 , . . . , i1 , i , . . . , n

(1)

(i) 1 , . . . , i1 , i , . . . , n

(i1)

son permutaciones del vector de ndices (1, . . . , n), kj ,


(k)

(k) , k
(i)

k = 1, . . . , i 1, k ,
( i)

j = k, . . . , i 1

son elementos listos del sistema nal, y kj , j, k = 1, . . . , n

son elementos del sistema restante antes de los intercambios. El i- esimo paso de transformaci on consiste en primer lugar en un intercambio de las (columnas) entre la la i y una la j > i y posiblemente entre la columna i y una columna (i) k > i, de tal forma que jk se cambia a la posici on (i, i). Los elementos intercambiados los llamamos (i) (i) (i) , (i) , j = i, . . . , n. , , j = 1, . . . , n, k = i, , . . . , n;
k jk j j

= 0 (mas adelante demostraremos que esto siempre se puede Ahora supongamos que lograr). La i- esima la ya no se modica. Los ceros en las posiciones i + 1, . . . , n de la columna i se generan de la siguiente forma:
(i+1) jk

(i) ii

:=

(i) jk (i)

ji

( i) (i+1) j

( i) , (i) ik ii

(i) := j

ji

(i)

(i) ii

(i) , i

i+1

j, k

n, (2.6)

(i+1) (i+1) (i)

:= j , := k ,
(i) (i)

i+1 i+1

j k

n, n.

El cuociente ji / ii se llama multiplicador para la la j . Despu es de n 1 pasos ponemos por unicaci on formal
(n) (n) nn := nn ,

(n) := (n) , n n

(n) (n) n := n ,

(n) (n) n := n .

DE GAUSS 2.2. EL METODO DE ELIMINACION

15

Al nal, llegamos al esquema 1 1 . . .


(1) (1) (1)

11

n el cual puede ser escrito en forma

(n)

(n) (1) 1n 1 , . . .. . . . . . (n) (n) n nn

(n)

(2.7)

Ry = c con una matriz triangular superior R. La soluci on de este sistema es y = (1 , . . . , n )T , y es de podemos recuperar la soluci on x = (1 , . . . , n )T del sistema original a trav (i) = i ,
i

i = 1, . . . , n,

(2.8)

usando la informaci on de la primera la del diagrama (2.7), que indica los ndices de las componentes de x correspondientes. La f ormula esencial para la conversi on del sistema restante depu es de los cambios es la siguiente: (j, k )nuevo = (j, k )antiguo (j, i)antiguo (i, k )antiguo , (i, i)antiguo i+1 j n, i+1 k n. (2.9)

El divisor de los multiplicadores, el elemento (i, i)antiguo , se llama elemento pivote. Es un elemento diagonal de la matriz triangular correspondiente. La parte derecha se transforma a trav es del mismo esquema. Resulta u til guardar los multiplicadores da cada paso de transformaci on; se pueden almacenar en las posiciones ocupadas por cero, y tambien se intercambian. Ejemplo 2.1. Para ilustrar el algoritmo, 0 1 A = 1 1 1 1 1 = 1
(1) (1) (1) (1)

La aplicaci on del algoritmo genera la siguiente sucesi on de esquemas. Partimos de 2 = 2


(1)

consideramos el sistema Ax = b con 3 3 3 , b = 4 . 3 5 3 = 3


(1)

1 = 1 0 = 11 2 = 2 1 = 21 3 = 3 1 = 31

(1) (1) (1)

1 = 12 1 = 22 1 = 32

(1) (1) (1)

3 = 13 3 = 23 3 = 33

(1) (1) (1)

3 = 1 4 = 2 5 = 3

(1) (1) (1)

16

DE SISTEMAS LINEALES (PARTE I) 2. METODOS DIRECTOS PARA LA SOLUCION

Intercambiamos las y columnas para que el elemento (2, 3) asume la posici on diagonal (1, 1). Es decir, este elemento lo consideramos como pivote: 3= 1 1 = 2
(1) (1) 3 (1) (1) (1) (1)

2= 2

(1) (1)

1= 3

(1)

3= 11 3=

1= 12 1= 22 1 =

1= 13 0= 23 1=

(1) (1)

21 2 = 1 3 = =3 Ahora calculamos los multiplicadores

(1)

(1) 4 = 1 . (1) 3= 5=
2 (1) 3

(1) 31

(1) 32

(1) 33

21 :=

21 11

(1)

(1)

= 1,

el cual corresponde a la sustracci on de la la 1, multiplicada por (1), de la la 2, y 31 := 31


(1) (1)

11

= 1,

que corresponde a la sustracci on de la la 1 de la la 3. El resultado de estas operaciones es 3= 1 1 = 2


(2) (2) (1) (1)

2 = 2

(2)

1 = 3

(2)

3= 11

(1)

1= 12

(1) (2)

1= 13 0 = 33

(1) (2)

(1) 4 = 1 1 = 2 9 = 3
(2) (2)

2 = 1 1 = 21 3 = 3

2 = 22

1 = 23

1 = 31 2 = 32

(2)

(2)

Intercambiamos las las 2 y 3 por motivo de ilustraci on: 3= 1 1 = 2 2 = 3


(2) 3 (2) (1) (1)

2= 2

(2)

1= 3

(2)

3= 11

(1)

1= 12 2=

(2) (2)

1= 13 0= 23 1=

(2) (2)

1 = 21 2 = 22

(1) 4 = 1 , (2) 9= 1 =
2 (2) 3

= 1 1 = 31

(2) 32

(2) 33

donde los multiplicadores fueron intercambiados con las las y luego renombrados. Ahora calculamos que 32 := 32
(2) (2)

22

= 1,

DE GAUSS 2.2. EL METODO DE ELIMINACION

17

el cual corresponde a la sustracci on de la la 2, multiplicada por (1), de la la 3. As nalmente llegamos al esquema 3= 1 1 = 2 2 = 3


(3) (2) (1) (1)

2= 2

(2)

1= 3

(3)

3= 11

(1)

1= 12

(2) (2)

1= 13 0= 23

(3) (3) (3) (3)

1 = 21 2 = 22

(1) 4 = 1 (2) 9 = 2 . 8 = 3
(3)

3 = 1 1 = 31 3
(3)

1 = 32 1 = 33 33

(3) 3

Entonces obtenemos la matriz triangular superior R y la parte derecha transformada c dadas por 3 1 1 4 9 . R = 0 2 0 , c = 0 0 1 8 La soluci on del sistema Ry = c entrega 3 = 8(= 1 ), 9 2 = (= 2 ), 2 1 = 1 3 4 + 9 8 2 5 = (= 3 ). 2

Hasta ahora siempre se ha presumido que cuando la matriz A es no singular, el intercambio de las (y columnas) siempre nos permite lograr que ii = 0,
(i)

i = 1, . . . , n.

Ahora vemos que este enunciado realmente es v alido. Usaremos las siguientes estrategias de pivote: la b usqueda del pivote en la columna, donde en el k - esimo paso determinamos el ndice k tal que kk = m ax ik
i k (k ) (k)

(2.10)

y s olo se intercambain las, o la b usqueda del pivote en la matriz restante, donde determinamos el ndice k tal que kk = m ax ij ,
i,j k (k) (k)

(2.11)

la cual implica el intercambio de las y columnas. En ambos casos, los multiplicadores satisfacen ji / ii Ejemplo 2.2 (Tarea 17, Curso 2006). 7 A = 10 1000
( i) (i)

1,

lo que causa un comportamento favorable del error de redondeo. Se considera el sistema lineal Ax = b dado por 1 1 10 1 1 , b = 13 . 0 1 1001

18

DE SISTEMAS LINEALES (PARTE I) 2. METODOS DIRECTOS PARA LA SOLUCION

La soluci on exacta del sistema es x = (1, 2, 1)T . Resolvemos ahora el sistema usando una aritm etica con cuatro d gitos signicativos, usando el algoritmo de Gauss a) sin pivoteo, b) con b usqueda del pivote en la columna. c) Interpretar los resultados. Usaremos la representaci on cient ca de los n umeros, por ejemplo 1234,567 1,234567 103 1,235E + 3 3,141759 3,141759 100 3,142E + 0 0,000654321 6,5432 104 6,543E 4.

Transformamos cada resultado intermedio a esta forma y redondeamos hasta el u ltimo d gito. Ojo: Internamente, las calculadoras usan una exactitud mayor que la desplegada en la pantalla. a) Sin pivoteo obtenemos: 7,000 100 1,000 100 1,000 100 1,000 101 A = 1,000 101 1,000 100 1,000 100 1,300 101 Fila 2nueva = Fila 2antigua 1,419 100 Fila 1antigua y Fila 3nueva = Fila 3antigua 1,429 102 Fila 1antigua : 7,000 100 1,000 100 1,000 100 4,290 101 4,290 101 A(1) = 1,429 102 1,419 102 1,000 103 0,000 100 1,000 100 1,001 103 1,000 101

Ahora calculamos Fila 3nueva = Fila 3antigua 3,331 102 Fila 2antigua : 7,000 100 1,000 100 1,000 100 1,000 101 4,290 101 4,290 101 1,290 100 . A(2) = La resubstituci on entrega 1,000 102 1,700 100 x3 = 1,700 100 , x2 = 1,307 100 , x1 = 0,999 100 .

4,280 102

1,290 100 .

b) Con pivoteo obtenemos 7,000 100 1,000 100 1,000 100 1,000 101 A = 1,000 101 1,000 100 1,000 100 1,300 101 . Intercambiamos la primera y la tercera la: 1,000 103 0,000 100 1,000 100 1,001 103 1,000 101 1,000 100 1,000 100 1,300 101 . 1,000 100 1,000 100 1,000 100 1,000 101 1,000 103 0,000 100 1,000 100 1,001 103

MATRICIAL DEL ALGORITMO DE GAUSS Y EL TEOREMA LR 2.3. DESCRIPCION

19

Fila 3nueva = Fila 3antigua 1,000 100 Fila 2antigua : 1,000 103 0,000 100 1,000 100 1,000 100 9,900 101 A(1) = La resubstituci on entrega x1 = 1,000 100 , x2 = 2,000 100 ,

Fila 2nueva = Fila 2antigua 1,000 102 Fila 1antigua y Fila 3nueva = Fila 3antigua 7,000 103 Fila 1antigua : 1,000 103 0,000 100 1,000 100 1,001 103 1,000 100 9,900 101 2,990 100 . A(1) = 1,000 100 9,930 101 2,993 100 1,001 103 2,990 100 .

3,000 103 3,000 103 x3 = 1,000 100 .

c) Con pivoteo, no hay errores de redondeo en este ejemplo, mientras que sin pivoteo, el error en la segunda componente es de aprox. 35 % y en la tercera de aprox. 70 %. 2.3. Descripci on matricial del algoritmo de Gauss y el teorema LR La transformaci on descrita en la secci on anterior, Ax = b Ry = c, ser a descrita ahora como operaci on matricial. Recordamos que en el algoritmo aparecen las sigiuentes operaciones: intercambios de las y combinaciones lineales de las, que son operaciones matriciales de la izquierda, y el intercambio de columnas, lo cual es una operaci on matricial de la derecha. El intercambio de la la i con una la k > i es efectuado por multiplicaci on de la izquierda con una matriz de permutaci on T e1 1 . . ... . T e i 1 1 T 0 0 0 1 ek . eT . 1 . 0 i+1 . . . . . . . . P= . = . . . . T . ek1 0 1 . . T 1 0 0 ei 1 eT k+1 . .. . . . 1 eT n Q= e1 ei1 ej ei+1 ej 1 ei ej +1 en .

An alogamente, las columnas i y j > i se intercambian a trav es de la multiplicaci on de la derecha por

20

DE SISTEMAS LINEALES (PARTE I) 2. METODOS DIRECTOS PARA LA SOLUCION

alg un vector (que corresponde a una de las columnas 1 a i 1 de la matriz transformada en el i- esimo paso). Entonces
T Ti g = (I qi eT i )g = g qi ( ei g ) = g, =0

Aqu qi es el vector compuesto por i ceros y los multiplicadores del i- esimo paso. Para explicarlo, sea 1 . . . g = l , l < i 0 . . . 0

Nos damos cuenta que P = PT , Q = QT y P2 = Q2 = I. Finalmente, los ceros debajo del (i) elemento ii se generan por multiplicaci on de la izquierda por la matriz 1 0 . .. . . . 0 1 (i) (i) i+1,i = I qi e T i+1,i . 1 , q := Ti = i i (i) ( i) ii ii . . . . .. . . . ( i ) ( i ) n,i n,i ( i) 1 (i) ii ii

o sea, las i 1 primeras columnas quedan sin cambiar. La columna j , j i, es de la siguiente forma: (i) (i) 1j 0 1j . . . . . . . . . (i) (i) 0 (i) ( i ) ( i ) ( i ) ( i ) i 1 ,j 1,j , aj = gj := hj := (i) (i) = gj + hj , i . ij 0 ij . . . . . . . . . (i) ( i) n,j 0 n,j Entonces Ti aj = Ti gj + hj
( i) (i) ( i)

= Ti gj + Ti hj = gj + hj qi eT i hj
= ij

( i)

( i)

(i)

(i)

( i)

(i)

MATRICIAL DEL ALGORITMO DE GAUSS Y EL TEOREMA LR 2.3. DESCRIPCION

donde ki = 0 para k i + 1. Escrita en forma de matrices, la transformaci on equivalente efectuada por el algoritmo de Gauss es la siguiente: Ax = b T1 P1 AQ1 Q1 x = T1 P1 b T2 P2 T1 P1 AQ1 Q2 Q2 Q1 x = T2 P2 T1 P1 b . . . Tn1 Pn1 . . . T1 P1 AQ1 . . . Qn1 Qn1 . . . Q1 x = Tn1 Pn1 . . . T1 P1 b .
=R =y =c

(i+1)

0 ( i) . . (i) 1j . 0 1j . . . 0 . . . . . . (i) ( i) (i) 0 ij i +1 ,i ( i ) 1,j + (i) = = ij , i ( i) ( i +1) ii 0 ij i+1,j . . . . . . . . . . . . ( i) ( i ) ( i +1) 0 n,i n,j n,j ( i) ii

21

j = 1, . . . , n,

Sea Q := Q1 Q2 . . . Qn1 . La matriz Q describe el efecto combinado de todas las permu(n) (1) n , tenemos taciones de columnas. Eso signica que con la notaci on denitiva 1 , . . . , e (1) (2) Q = e 1 2 Entonces QT x = y, o sea (i) = i ,
i

e (n) . n

i = 1, . . . , n,

identidad que ya usamos m as arriba. Ahora podemos escribir R =Tn1 Pn1 Tn2 Pn2 . . . T1 P1 AQ
n1 =:T n2 =:T

= Tn1 (Pn1 Tn2 Pn1 )(Pn1 Pn2 Tn3 Pn2 Pn1 ) . . .


n2 =:T

. . . (Pn1 Pn2 . . . P2 T1 P2 . . . Pn2 Pn1 )(Pn1 Pn2 . . . P1 AQ),


1 =:T =:P

es decir, deniendo n1 := Tn1 , T i := Pn1 . . . Pi+1 Ti Pi+1 . . . Pn1 , T P := Pn1 Pn2 . . . P1 ,

obtenemos la f ormula

i = 1, . . . , n 2,

n1 T n2 . . . T 1 PAQ. R=T

22

DE SISTEMAS LINEALES (PARTE I) 2. METODOS DIRECTOS PARA LA SOLUCION

Podemos aprovechar P2 j = I para concluir que i = Pn1 . . . Pi+1 (I qi eT )Pi+1 . . . Pn1 T = I Pn1 . . .
=: qi i T Pi+1 qi ei

puesto que las matrices Pi+1 , . . . , Pn describen intercambios de elementos con ndice i + 1, T es decir, no afectan a ei . Seg un nuestra construcci on, el vector qi es el vector de los multiplicadores del i- esimo paso de eliminaci on, los cuales estan sujetos a los mismos intercambios de las que el sistema restante en los pasos de eliminaci on i + 1, . . . , n 1. i es una matriz triangular inferior con diagonal (1, . . . , 1), En virtud de lo anterior, T 1 y el producto T 1 . . . T 1 . Entonces tenemos que tanto como las matrices T 1 n1 i 1 1 PAQ = T . . . T R = LR,
1 n1 =:L

=I

i eT q i ,

i = 1, . . . , n 1,

donde R es una matriz triangular superior y L es una matriz triangular inferior con diagonal (1, . . . , 1). Adem as, sabemos que 1 = I + q i eT , i = 1, . . . , n 1, T
i i

lo que implica que

k = 0 para j Dado que eT j q

1 eT 2 eT n1 eT L = (I + q 1 )(I + q 2 ) . . . (I + q n1 ). k , podemos escribir L=I+


n1 k=1

k eT q k,

es decir, los elementos de L debajo de la diagonal son los multiplicadores (intercambiados). Ahora queda para demostrar que el algoritmo nunca termina para una matriz A regular, ( i) o sea, que aplicando intercambios apropiados siempre podemos lograr que ii = 0 para i = 1, . . . , n. Eso incluso es v alido si no usamos intercambios de columnas (s olamente de las). Si no fuera as , existir a un ndice k tal que ik = 0, o sea
(k )

i = k, . . . , n, . . . . . . . . = det(A) = 0, . . . . . . .

... ... . . . . . . . . . . . . . .. . . . . . . . Tk1 Pk1 Tk2 . . . T1 P1 A = . . . det(... )=0 0 . . . . . . . . 0 ...

MATRICIAL DEL ALGORITMO DE GAUSS Y EL TEOREMA LR 2.3. DESCRIPCION

23

una contradicci on. La consideraciones anteriores pueden ser resumidas en el siguiente teorema. Teorema 2.1. Sea A Knn una matriz regular. Entonces existen una matriz de permutaci on P, una matriz triangular inferior L con diagonal (1, . . . , 1) y una matriz triangular superior R, todas pertenecientes a Knn , tales que PA = LR. Si el algoritmo de Gauss es aplicado a un sistema lineal Ax = b, la matriz P es la matriz de permutaci on que describe el efecto de todos los intercambios de las, Q es la matriz de permutaci on que describe el efecto de todos los intercambios de columnas, R es la matriz triangular superior que resulta y L es la matriz triangular inferior con diagonal (1, . . . , 1) y los multiplicadores (adecuadamente intercambiados), entonces tenemos que PAQ = LR. Ejemplo 2.3. 0 P= 0 1 Para la matriz del 1 0 0 0 1 , Q= 0 0 0 1 Ejemplo 2.1, obtenemos que 0 1 1 0 0 1 0 , L = 1 1 0 , 0 0 1 1 1 3 1 1 R = 0 2 0 . 0 0 1

Ejemplo 2.4. a) Nos interesa calcular una descomposici on pivote en la matriz restante, de la matriz 1 3 A= 2 1 2 2

triangular PAQ = LR, con b usqueda de 2 4 . 8

(2.12)

Indicar expl citamente las matrices P, Q, L y R b) Utilizando la descomposici on de (a), queremos calcular A1 . Soluci on sugerida. a) Las etapas consecutivas del algoritmo de Gauss son las siguientes: 1 1 1 2 2 3 2 Entonces 2 3 3 3 2 3 8 1 4 2 4 2 8 1 2 2 2 1 3 3 1 2 2 1 2 2 1 1 1 4 2 7 2 0 2 3 8 2 2 2 7 2 1 2 3 3 8 2 2 7 2 4 7 1 2 3 . 2 15 7

1 3 1 4 3 1 2 2 2

0 0

1 L = 4 1 2

1 0 , 4 1 7

8 R = 0 0

0 0 1 0 0 1 3 , P = 1 0 0 , Q = 0 1 0 . 2 0 1 0 1 0 0 15 7

24

DE SISTEMAS LINEALES (PARTE I) 2. METODOS DIRECTOS PARA LA SOLUCION

b) Usando 1 1 0 = , R1 4 5 4 1 14 7 1 0 0 1 1 1 8 14 15 2 1 , = 0 7 5 7 0 0 15 1 6 0 . 1

L1

obtenemos

A 1

Ejemplo 2.5 (Certamen 1, Curso 2006). Se consideran la matriz A y el vector b dados por 6 3 1 1 A = 8 5 2 , b = 2 . 9 7 4 3

1 1 7 1 4 12 15 15 30 15 2 1 1 P = 2 = QR1 L1 P = Q 0 5 5 5 5 1 4 1 1 7 6 15 15 30 15

12

a) Usando el algoritmo de Gauss con b usqueda del pivote en la columna, determinar una matriz P de permutaci on, una matriz L = (ij ) triangular inferior con 11 = 22 = 33 = 1 y una matriz R triangular superior tales que PA = LR. b) Resolver el sistema Ax = b. c) Usando el algoritmo de Gauss con b usqueda del pivote en la matriz restante, determinar matrices P, Q de permutaci on, una matriz L = (ij ) triangular inferior con 11 = 22 = 33 = 1 y una matriz R triangular superior tales que PAQ = LR. Soluci on sugerida. a) Salimos del esquema 1 6 3 1 1 A= 2 8 5 2 2 3 9 7 4 3 donde 9 es el pivote. Intercambiando Fila 1 con Fila 3, obtenemos 3 9 7 4 3 2 8 5 2 2 1 6 3 1 1

MATRICIAL DEL ALGORITMO DE GAUSS Y EL TEOREMA LR 2.3. DESCRIPCION

25

Ahora, 8 Fila 2nueva = Fila 2antigua Fila 1antigua 9 2 Fila 3nueva = Fila 3antigua Fila 1antigua : 3 3 9 2 1
8 9 2 3

7 5 3

4 5 3

3 , 1

2 11 14 9 9 3

2 y 3 son multiplicadores almacenados y 5 es el nuevo pivote. Luego, interdonde 8 9 3 cambiando Fila 2 con Fila 3,

3 9 1 2
2 3 8 9

7
5 3

4
5 3

3 1 ,

11 14 2 9 9 3

Ahora, calculamos Fila 3nueva = Fila 3antigua plicador


11 15

para obtener

11 Fila 2antigua y almacenamos el multi15

3 9 1 2 El u ltimo esquema 0 P = 1 0
2 3 8 9

7
11 15

4 1 3

3
1 15

5 1 3 5 3

b) Aplicando una resubstituci on a la parte derecha co-transformada, obtenemos 1 5 1 x3 3 4x3 7x2 1 1 4 15 3 x3 = = . = , x2 = = , x1 = 1 5 5 5 9 5 3 3 c) Una inspecci on del primer paso de (a) muestra que 9 seria escogido como pivote tambi en por la b usqueda en la matriz restante, asi que este primer paso es igual al 5 primer paso del m etodo con b usqueda en la matriz restante. Asimismo, 3 tambi en seria el pivote en el segundo paso. Concluimos que la b usqueda del pivote en la matriz restante genera el mismo resultado que (a), asi que las matrices P, A y R son las especicadas arriba, y Q = I.

implica que 1 0 1 2 0 0 , L = 3 8 1 0 9

0 1
11 15

0 0 , 1

9 7 4 5 3 R = 0 5 . 3 1 0 0 3

26

DE SISTEMAS LINEALES (PARTE I) 2. METODOS DIRECTOS PARA LA SOLUCION

El signicado de los elementos pivotes se aclara en el siguiente teorema, formulado para una descomposici on triangular sin intercambios. Para el caso general, hay que remplazar A por PAQ. Teorema 2.2. Si el algoritmo de Gauss se ejecuta hasta el paso k , 1 intercambios de las o columnas, entonces tenemos que
k (j ) jj j =1

n 1, sin

11 . . . . = . . k 1 . . .

1k . . esimo subdeterminante principal de A. . = k - kk

Entonces, si todos los subdeterminantes principales de A son diferentes de cero, se puede ejecutar el algoritmo de Gauss sin intercambio de las ni de columnas. En este caso, nalmente obtenemos que
n

det(A) =
j =1

jj .

(j )

Demostraci on. Primero notamos que 11 . . . 1k . . . . . . = e1 k1 . . . kk

ek

A e1

ek .

Luego tomamos en cuenta que seg un nuestra construcci on, (1) 11 . . . ... . . . 0 . . . . . .. (k ) . . . . k . . Rk A kk . . Tk . . . T1 A = . , = . . 0 . . . . 0 . . . . . . . . . . . . . . 0 0 o sea, dado que 1

1 1 T 1 . . . Tk

. 21 . . . .. . . 1 . . = . k+1,k 1 . . . . . . . 0 .. . . . . . . . . . . . .. n1 nk 0

.. 0

. 1

MATRICIAL DEL ALGORITMO DE GAUSS Y EL TEOREMA LR 2.3. DESCRIPCION

27

donde ji son los multiplicadores, obtenemos que T 11 . . . 1k e1 . 1 1 . . . . = . . . . T1 . . . Tk k1 . . . kk eT k T e1 1 1 . = . . T1 . . . Tk eT k 1 . 21 . . = . ... ... . . k1 k,k1 1 Puesto que det(Lk ) = 1, concluimos que 11 . . . . . . k 1 . . .

k Rk A 0 Rk 0 0 . . . . . . 0

e1

ek

0 . . . Rk = Lk Rk . . . . 0 0
k

1k . . . = det(Lk Rk ) = det(Rk ) = kk

jj .
j =1

(j )

Para k = n 1, de Tn1 . . . T1 A = R resulta det(Tn1 ) det(Tn2 ) . . . det(T1 ) det A = det R =


=1 =1 =1

jj .
j =1

(j )

Las hip otesis del Teorema 2.2 son satisfechas para matrices estrictamente diagonal dominantes y matrices denidas positivas. Denici on 2.2. Una matriz A Knn se llama estrictamente diagonal dominante si
n

i = 1, . . . , n :

|ii | >

j =1 j =i

|ij |.

Una matriz A = A se llama denida positiva si Teorema 2.3. Todos los subdeterminantes principales de una matriz A Cnn estrictamente diagonal dominante son diferentes de cero. Demostraci on. Para demostrar el teorema, es suciente demostrar que una matriz A = nn (aij ) C estrictamente diagonal dominante es no singular, dado que cada submatriz principal de una matriz estrictamente diagonal dominante es estrictamente diagonal dominante. Para tal efecto, supongamos que A es una matriz estrictamente diagonal dominante del tama no n n, pero que existe un vector x = (x1 , . . . , xn )T = 0 tal que Ax = 0. (2.13) x Cn , x = 0 : x Ax > 0.

28

DE SISTEMAS LINEALES (PARTE I) 2. METODOS DIRECTOS PARA LA SOLUCION

Dado que x = 0, existe un ndice m {1, . . . , n} tal que |xm | = m ax |x1 |, . . . , |xn | > 0. Evaluando la m- esima componente de (2.13), tenemos
n

(2.14)

amm xm +
j =m j =1

amj xj = 0,

lo que podemos reescribir como


n

amm xm = Tomando valores absolutos, tenemos

amj xj .
j =m j =1

|amm ||xm | y usando (2.14), llegamos a

j =m j =1

|amj ||xj |,

|amm ||xm | Dividiendo por |xm |, obtenemos

|xm |

j =m j =1

|amj |.

|amm |

j =m j =1

|amj |,

una contradicci on a la diagonaldominancia estricta de A. Teorema 2.4. Todas las submatrices principales de una matriz denida positiva y hermitiana son denidas positivas y tienen determinante positivo. Todos los valores propios de una matriz denida positiva son positivos. Demostraci on. Sea A hermitiana, entonces existe una matriz U unitaria tal que U AU = diag (1 , . . . , n ), donde 1 , . . . , n son los valores propios de A. Sean y1 , . . . , yn las columnas de U. Ahora, sea x := yi para i = 1, . . . , n. Entonces tenemos y1 T . 0 < x Ax = yi y1 yn diag(1 , . . . , n ) . . yi = ei diag (1 , . . . , n )ei = i . yn

DE CHOLESKY 2.4. LA DESCOMPOSICION

29

Dado que det A = 1 2 . . . n , resulta det A > 0. Ahora sea Ak una submatriz principal de A, es decir, ai 1 i 1 ai 1 i k i1 . . . . . . Ak = , y sea xk = . . . = 0. Ahora sea i = 0 si i {1, . . . , n}\{i1 , . . . , ik } y x = (1 , . . . , n )T . Entonces x Ax = x k Ak xk > 0. ai k i 1 ai k i k ik

Ademas, la matriz A es hermitiana, por lo tanto podemos aplicar la misma conclusi on que para la matriz A a la matriz Ak . 2.4. La descomposici on de Cholesky Seg un el Teorema 2.4, en el caso de una matriz hermitiana denida positiva no es necesario intercambiar columnas y las durante la ejecuci on del algoritmo de Gauss, es decir, al calcular la factorizaci on en matrices triangulares. Puesto que 11 . . . . . . k = 1, . . . , n y ii =
(i)

11 . . . . . . k 1 . . .
(i)

1k . . . > 0, kk

1i . . .

i1 . . . ii , 11 . . . 1,i1 . . . . . . i1,1 . . . i1,i1

tenemos que ii > 0 para i = 1, . . . , n. Finalmente, resulta que todas las matrices restantes ij
(k+1)

k+1

i, j

n,

son hermitianas, o sea llegamos a A = LR con (1) (n) 11 1n . (2) . . 22 , R= . .. . . . (n) nn Entonces, deniendo

k = 1, . . . , n 1 (1) 12 (1) 11 L= . . . (1) 1n 11


(1)

1 .. . .. .

(i) ii

> 0,

n1,n n1,n1
(n1)

(n1)

llegamos a

D := diag

1
(1) 11

,...,

1
(n) nn

. 1

L , A = LR = LD1 DR = L

30

DE SISTEMAS LINEALES (PARTE I) 2. METODOS DIRECTOS PARA LA SOLUCION

:= LD1 . Esa forma sim donde denimos L etrica de descomposici on en matrices triangulares se llama descomposici on de Cholesky. Existe solamente para matrices hermitianas denidas ij de la matriz triangular inferior L pueden ser calculados sucesipositivas. Los elementos con la vamente por columnas, aprovechando que ij es el producto escalar de la la j de L ii > 0 para todo i. En este caso, . El c columna i de L alculo es u nico cuando exigimos que la identidad
i

ii =
k=1

ik ik

nos lleva a la identidad |ii | = ii


2 i 1 k=1

ik

> 0,

ii de forma u de la cual podemos despejar nica de la siguiente forma: ii := Ahora, para j > i sabemos que
i

ii

i1 k=1

ik 2 .

(2.15)

ji =
k=1

ik , jk

por lo tanto, ji = 1 ii ji
i 1 k=1

ik jk

i = 1, . . . , n.

(2.16)

Las ecuaciones (2.15) y (2.16) implican que jk jj , k = 1, . . . , j,

j = 1, . . . , n.

(2.17)

de Cholesky es grande comparado con Eso signica que ninguna componente del factor L los elementos de A, lo que signica que el algoritmo no es muy sensible con respecto a errores de redondeo. Ejemplo 2.6. Aplicando las f ormulas (2.15) matriz 60 A = 30 20 : los siguientes elementos de L 11 = 60 = 2 15, 30 21 = = 15, 60 y (2.16), calculamos sucesivamente para la 30 20 20 15 15 12

DE CHOLESKY 2.4. LA DESCOMPOSICION

31

20 5 31 = =2 , 3 60 22 = 20 ( 15)2 = 5, 1 32 = 5 33 = 15 2 15 2 5 3
2

5 3

5,

12

1 ( 5)2 = . 3

El siguiente teorema es una consecuencia inmediata del Teorema 2.4 y de la Denici on 2.2. Teorema 2.5. La matriz A es hermitiana y denida positiva si y s olo si ella posee una descomposici on A = LL , donde L es una matriz triangular inferior invertible. Ejemplo 2.7 (Tarea 4, Curso 2006). Queremos L tal que A = LLT , donde 2 1 A Rnn , A = determinar una matriz triangular inferior 1 ... ... , .. .. . 1 . 1 2

(2.18)

Para vericar que (2.19) realmente es la soluci on deseada, denimos los vectores li := 0, . . . , 0, i1 , i i+1 , 0, . . . , 0 i
i

donde el resultado debe ser general con respecto a casos con n peque no, una soluci on razonable es 2 3 1 2 2 2 4 L= 3 3 . ..

n. Desp ues de calcular a mano algunos

..

. n1 n

. n + 1 n

(2.19)

i1 T e + i i 1

i+1 T e . i i

Entonces tenemos li , li = 2, li1 , li = li , li+1 = 1, y li , lj = 0 si |i j | 2.

32

DE SISTEMAS LINEALES (PARTE I) 2. METODOS DIRECTOS PARA LA SOLUCION

Ejemplo 2.8 (Certamen 1, Curso 2006). Se 1 A = 2 0

Se pueden encontrar n umeros , y Soluci on sugerida. a) Calculando sucesivamente los elementos 1 L = 2 0 b) Sean A1 = x y z ,

a) Econtrar una matriz triangular inferior L tal que A = LLT (descomposici on de Cholesky). b) Determinar la ultima columna de A1 , usando la descomposici on de Cholesky. c) Las matrices B y L sean dadas por 1 3 2 1 2 1 0 0 0 0 3 10 10 6 11 1 0 0 0 B= 2 10 24 18 26 , L = 2 4 0 0 1 6 18 39 24 1 7 2 5 0 2 11 26 24 32 2 5 1 1 tales que A = LLT ? de L, resulta 0 0 3 0 . 4 5 I = e1 e2 e3 .

considera la matriz 2 0 13 12 . 12 41

De la identidad AA1 = I sacamos que el vector z deseado satisface el sistema lineal Az = e3 . Para aprovechar la descomposici on de Cholesky, LLT z = e3 , determinamos primero un vector w tal que Lw = e3 , luego determinamos z de LT z = w. Este procedimiento entrega 1 1 4 8 w1 = 0, w2 = 0, w3 = ; z3 = ; z2 = ; z1 = . 5 25 75 75 Entonces, el vector deseado es 1 z = (3, 4, 8)T . 75 c) En clase demostramos que los elementos en la j - esima la de L son menores o iguales en valor absolutos que la raiz del j - esimo elemento diagonal de A. En la la 4, aparece el elemento 42 = 7. Pero 72 = 49 < 39, lo cual es el elemento diagonal de A, independiente de , y . Entonces nunca se pueden determinar tales que A = LLT . Ejemplo 2.9 (Tarea 7, Curso 2006). 1 A= 2 1 2 1 0 2 , 2 1 Sean 1 0 0 I = 0 1 0 . 0 0 1

(2.20)

DE CHOLESKY 2.4. LA DESCOMPOSICION

33

Usando el algoritmo de la descomposici on de Cholesky, calcular hasta un decimal Soluci on sugerida. Seg un el Teorema 2.5, la matriz A es denida positiva si y solo si el algoritmo de Cholesky puede ser ejecutado. Tratamos de hacerlo para la matriz A + tI e identicamos las restricciones para t que aparecen. Recordamos que los elementos diagonales de L deben ser reales. Para la primera columna de L = (ij ) obtenemos 11 = t 1, (2.22) 2 21 = , (2.23) t1 1 31 = . (2.24) t1 Obviamente, de (2.22) obtenemos el requerimiento t > 1. Para los elementos de la segunda columna tenemos 22 = 32 = t 4 t2 t 4 = , t1 t1 t1 21 ) = (2 21 2 t t4 (2.26) t2 t 1 2t 4 = t4 t1 1 2 17 ; 4 2(t 2) . (2.27) (2.25) t0 := m n t R : A + tI es denida positiva . (2.21)

La soluci on de t2 t 4 = 0 es t= usando (2.25) concluimos que

(t 1)(t2 t 4)

1 17 + = 2,56155 . . . . 2 4 Finalmente, para el u ltimo elemento de L tenemos t> 33 = donde la funci on (t) = (t2 2t)(t2 t 4) 4(t 2)2 = t4 3t3 6t2 + 24t 16 t1 1 4(t 2)2 = t 1 (t 1)(t2 t 4) (t) , (t 1)(t2 t 4)

(2.28)

debe ser positiva. Ahora, tratando t = 3, obtenemos (t) = 2 > 0. Usando que para cualquier matriz B denida positiva, tambien B + tI es denida positiva para t > 0, tenemos que buscar t0 1 + 2 17 ,3 4

34

DE SISTEMAS LINEALES (PARTE I) 2. METODOS DIRECTOS PARA LA SOLUCION

Usando (2,75) = 0,5742 . . . , (2,8) = 0,2304 . . . y (2,9) = 0,7011, la respuesta es t0 = 2,8 . . . . (De hecho, (2 2) = 0.) Ejemplo 2.10 (Certamen 1, Curso 2010). 1 2 A= 1 1 Se considera la matriz 2 1 1 8 4 2 . 4 18 3 2 3 11

Soluci on sugerida. a) Sea

a) Econtrar una matriz triangular inferior L tal que A = LLT (descomposici on de Cholesky). b) Determinar la tercera columna de A1 , usando la descomposici on de Cholesky. c) Se puede aplicar la descomposici on de Cholesky a la siguiente matriz? 5 4 2 1 1 2 4 3 1 1 1 1 2 1 2 0 1 4 B= 1 1 0 3 2 1 1 1 1 2 0 1 2 1 4 1 1 2 l1 l2 L= l4 l7 0 0 0 0 . l6 0 l9 l10

0 l3 l5 l8

Entonces, comparando los elementos de LLT , obtenemos sucesivamente


2 l1 = 1 l1 = 1, l1 l2 = 2 l2 = 2, 2 2 l2 + l3 = 8 l3 = 2, l1 l4 = 1 l4 = 1,

1 l2 l4 + l3 l5 = 4 l5 = (4 (2)) = 1, 2 2 2 2 l4 + lh + l6 = 18 l6 = 18 1 1 = 4, l1 l7 = 1 l7 = 1, 1 l2 l7 + l3 l8 = 2 l8 = (2 (2) 1) = 0, 2 1 l4 l7 + l5 l8 + l6 l9 = 3 l9 = (3 1 1) = 1, 4 2 2 2 2 l7 + l8 + l9 + l10 = 11 l10 = 11 1 1 = 3,

TRIANGULAR Y CASOS ESPECIALES 2.5. APLICACIONES DE LA DESCOMPOSICION

35

es decir

b) Sea z la tercera columna de A1 , entonces z es la soluci on del sistema lineal Az = e3 = (0, 0, 1, 0)T . Utilizando la descomposici on de Cholesky, podemos determinar Z resolviendo primeramente el sistema Ly = e3 y luego LT z = y. As obtenemos 0 4 5 1 1 0 ; . z= y= 12 3 144 10 1 4 c) No. La matriz contiene la submatriz principal = a11 a12 5 4 = a21 a22 4 3

1 0 0 2 2 0 L= 1 1 4 1 0 1

0 0 . 0 3

con det = 1 < 0; seg un el Teorema 2.4, B no es denida positiva y no se puede aplicar la descomposici on de Cholesky. 2.5. Aplicaciones de la descomposici on triangular y casos especiales Cuando conocemos una descomposici on PAQ = LR de una matriz A donde P y Q son matrices de permutaci on, podemos f acilmente resolver el sistema lineal Ax = b para una parte b derecha arbitraria: en virtud de A = PT LRQT , tenemos que Con la notaci on d := Pb, y := QT x y z := Ry procedemos de la siguiente forma: 1. Denimos T 1 1 e1 . . . . i := i , i = 1, . . . , n, donde d = . . , b = . . , P = . . T n n en 2. Resolver Lz = d para determinar z. 3. Resolver Ry = z para determinar y. 4. Obtenemos i = i , i = 1, . . . , n, Ax = b PT LRQT x = b LRQT x = Pb.

Eso signica que es posible tratar primero la matriz A por el algoritmo de Gauss para determinar su descomposici on triangular, y luego resolver el sistema Ax = b siguiendo los pasos 1 a 4. En comparaci on con el algoritmo original, este procedimiento no signica ning un aumento del tiempo computacional ni del espacio de almacenaje. La descomposici on

1 1 . . . donde y = . , x = . . , Q = e1 n n

en .

36

DE SISTEMAS LINEALES (PARTE I) 2. METODOS DIRECTOS PARA LA SOLUCION

triangular tambi en puede ser usada para invertir la matriz A, aunque esta tarea se presenta solo rara vez. Sea PAQ = LR, donde P y Q son matrices de permutaci on, L es triangular inferior y R es triangular superior. Entonces sabemos que A = PT LRQT , A1 = QR1 L1 P. (2.29) Las matrices R y L pueden ser invertidas (sin espacio de almacenaje adicional) si formamos sucesivamente las columnas n, n 1, . . . , 1 de R1 y 1, 2, . . . , n de L1 (aprovechando que la diagonal (1, . . . , 1) de L es conocida). Al formar el producto R1 L1 podemos usar la estructura especial de la matriz. Si ij , ik y ij son los elementos de L1 , R1 y A1 , respectivamente, sabemos que
n

ij =
k=m ax{i,j }

ik kj ,

i, j = 1, . . . , n,

donde jj = 1. Finalmente, hay que aplicar las permutaciones decritas por P y Q. dado que P = Pn1 . . . P1 y Q = Q1 . . . Qn1 , (2.29) implica que los intercambios de las aplicados durante la descomposici on triangular deben ser aplicados en el orden revertido a las columnas del producto, y analogamente los intercambios de las columnas a las las del producto. Se puede demostrar que el esfuerzo computacional es de n3 + O(n2 ) operaciones del tipo := + o := + / . Denici on 2.3. Una matriz A Knn se llama matriz casi triangular o matriz de Hessenberg si ij = 0 para j < i 1.

Denici on 2.4. Una matriz A Knn se llama (p, q )-matriz de banda si ij = 0 para j < i p y j > i + q. En las aplicaciones frecuentamente aparecen matrices tridiagonales con p = q = 1. Si no se usa el intercambio de columnas para una matriz de Hessenberg, no es necesario eliminar en cada paso la matriz restante entera, sino que solo una la de ella. (Por ejemplo, la desconocida 1 aparece solamente en la primera y la segunda ecuaci on.) Eso signica que abajo de la diagonal, la matriz L tiene a lo m as un elemento diferente de cero en la primera subdiagonal. En el caso que no se necesita ning un intercambio, la matriz L es una matriz bidiagonal (p = 1, q = 0). Si para una matriz de banda no se necesita ning un intercambio, la matriz L tiene p + 1 bandas y la matriz R tiene q + 1 bandas, o sea la informaci on sobre la descomposici on ocupa solo n (p + q + 1) elementos de almacenaje.

Cap tulo 3

M etodos directos para la soluci on de sistemas lineales (Parte II)


3.1. Normas de vectores y matrices Denici on 3.1. Sea A Cnn . Se dene el espectro de A, denotado (A), como el conjunto de todos los valores propios de A. Adem as, se llama radio espectral de A a r (A) := m ax ||.
(A)

Denici on 3.2. Sea V un espacio vectorial sobre el cuerpo C. Se llama norma de vector a toda aplicaci on : V R+ 0 tal que para todo x, y V y C se verica: 1. x > 0 si x = 0 y x = 0 si y s olo si x = 0. 2. x = || x . 3. x + y x + y . Damos a continuaci on algunos ejemplos de normas para el espacio Cn :
n

:=
i=1

|xi |,
n 1/2 1/2

(3.1)
2

x x

:= (x x)
1 i n

=
i=1

|xi |

(3.2) (3.3)

:= m ax |xi |,

a las que nos referimos como norma 1, norma 2 y norma , respectivamente; en general, para p [1, ) denimos
n 1/p

x como norma p. Teorema 3.1. Una norma

:=
i=1

|xi |

(3.4)

Demostraci on. Tomamos en cuenta que para x = (1 , . . . , n )T , y = (1 , . . . , n )T x y = xy


n i=1

on continua. : Kn R+ 0 es una funci x y |i i | ei


37

38

DE SISTEMAS LINEALES (PARTE II) 3. METODOS DIRECTOS PARA LA SOLUCION


1 i n

m ax ei m ax |i i |,
1 i n

y aplicamos la denici on de la continuidad. Teorema 3.2. Si y tales que

son dos normas sobre Kn , entonces existen constantes m, M > 0 x Kn : m x

M x .

(3.5)

Demostraci on. Sean := Sea

arbitraria (el caso general sigue por transitividad).

S := x Kn xm

x = (1 , . . . , n )T , m ax |i | = 1 .
1 i n

El conjunto S es compacto. Puesto que = m n x


xS

=: m,

es continua, existen xm , xM S tales que xM

= m ax x
xS

=: M.

Entonces, x Kn , x = 0 : m x x M,

lo que implica (3.5) en virtud de la homogeneidad (item 2. en la Denici on 3.2). (Para x = 0, (3.5) es trivial.) Denici on 3.3. En el espacio Cnn se llama norma de matriz a toda aplicaci on + nn nn C R0 tal que para todas matrices A, B C y todo C se verica: 1. A > 0 si A = 0 y A = 0 si y s olo si A = 0. 2. A = || A . 3. A + B A + B . 4. AB A B . En correspondencia con cada norma vectorial matrices A Cnn por medio de la expresi on A := m ax
x=0

de Cn , se dene una norma para (3.6)

Ax = m ax Ax . x =1 x

Esta norma matricial se dice inducida por la norma vectorial. En particular, las normas vectoriales 1, 2 e inducen las siguientes normas matriciales, a las cuales igualmente nos referimos como norma 1, norma 2 y norma , respectivamente:
n

A A A

:= m ax Ax
x
1 =1

:= m ax

1 j n

i=1

|aij |,

(3.7) (3.8)

:= m ax Ax 2 ,
x
2 =1

:= m ax
x

=1

Ax

= m ax

1 i n

j =1

|aij |.

(3.9)

3.1. NORMAS DE VECTORES Y MATRICES

39

Tambi en se dene sobre Cnn la siguiente norma, la cual no es inducida por una norma vectorial,
n 1/2

A y que se llama norma de Frobenius.

:=
i,j =1

|aij |2

(3.10)

Denici on 3.4. Una norma de matriz se dice compatible con una norma de vector si, para cada A Cnn y para cada x Cn , se tiene que Ax
vector

matriz

vector .

Note que de la denici on (3.6) de desprende que cada norma matricial inducida por una norma vectorial es compatible con la norma vectorial que la induce. As tenemos, en particular, que las normas matriciales 1, 2 e son compatibles con las correspondientes normas vectoriales 1, 2 e . Por otra parte, la norma de Frobenius, que como indicamos no es inducida por norma vectorial alguna, es compatible con la norma vectorial 2. Teorema 3.3. Si A Cnn , entonces A
2

r (A A).

Demostraci on. Es claro que A A es una matriz hermitiana. Por el Teorema del Eje Principal sabemos que A A tiene n vectores propios que forman una base ortonormal de Cn . Veremos a continuaci on que los valores propios de A A son adem as no negativos. En efecto, si es un valor propio de A A y v es un correspondiente vector propio asociado, entonces A Av = v y adem as, Av
2 2

= (Av) (Av) = v (A A)v = v (v) = v 2 2. Av 2 2 v 2 2

Como v = 0, de esta u ltima relaci on deducimos que = 0. (3.11)

Ahora sean 1 2 ... n son los valores propios de A A y {v(1) , v(2) , . . . , v(n) } un conjunto de vectores propios asociados que forman una base ortonormal de Cn . Entonces, para x Cn \{0} existen escalares 1 , . . . , n tales que
n

x=
j =1

j v(j ) .

(3.12)

Por otro lado, Ax


2 2

= (Ax) Ax = x (A A)x.

(3.13)

40

DE SISTEMAS LINEALES (PARTE II) 3. METODOS DIRECTOS PARA LA SOLUCION

Remplazando (3.12) en (3.13) y reordenando,


n n n n

Ax

2 2

=
j =1

j v

(j )

(A A)
i=1

i v

(i)

=
j =1 i=1

i j i v(j ) v(i) ,

y como v(j ) v(i) = ij , entonces


n n

Ax En forma an aloga calculamos

2 2

=
i=1

|i | i
n

1
i=1

|i |2 .
n

(3.14)

2 2

=x x=
j =1 i=1

i j v

(j ) (i)

=
i=1

|i |2 .

(3.15)

De (3.14) y (3.15) concluimos que x Cn \{0} : lo cual equivale a A


2

Ax 2 x 2 Ax 2 x 2

1 ,

:= m ax
x=0

1 .

(3.16)

Para mostrar que la cota 1 se alcanza, basta exhibir un vector no nulo para el cual la igualdad se cumpla en (3.16). Con esta nalidad sea v1 un vector propio asociado a 1 . Entonces, de (3.11) obtenemos inmediatamente que 1 = Av1 2 , v1 2

esto es, el m aximo de (3.16) se alcanza en x = v1 y es igual a se concluye la demostraci on. Corolario 3.1. Si A es hermitiana, entonces A Demostraci on. Puesto que A se tiene que inmediatamente se llega a A que es lo que se quer a demostrar.
2 2 2

1 . Notando que 1 = r (A A)

= r (A).
2

r (A A) y A = A, entonces A r (A2 ) = (r (A))2 , = (r (A))2 = r (A),

r (A2 ). Como

Teorema 3.4. Sea alguna norma vectorial sobre Cn y la norma matricial inducida. En este caso, B Cnn : r (B) B .

3.1. NORMAS DE VECTORES Y MATRICES

41

Demostraci on. Sea un valor propio de B con || = r (B) y x = 0 un vector propio asociado. Entonces es decir, x = || x = r (B) x = Bx B Bx = r (B). x B x ,

Teorema 3.5. Sea B Cnn una matriz arbitraria, con > 0 arbitrario dado. Entonces existe una norma vectorial B sobre Cn tal que para la norma matricial asociada, B
B

r (B) + .

Para los elementos arriba de la diagonal (


n ik n

Demostraci on. Seg un el Teorema 1.1 (sobre la forma normal de Schur), existe una matriz U unitaria tal que 1 ... ... . . . 0 U BU = . . =: ( ik ). .. ... . . 0 0 n
ik

con k > i) sabemos que U =: (sk ).

=
s=1 l=1

li ls sk ,

Eso signica | Sean := m n En este caso, D1 U BUD = ( entonces


n ik k i ik |

n2 ,

:= m ax |ls |.
1 l,s n

n3 (

,1 , + 1)

D := diag(1, , . . . , n1 ).

),

D1 U BUD

= m ax

1 i n

k =i ii |

ik

ki

|
n

1 i n

m ax |

+ m ax

1 i n

r (B) + (n 1)n r (B) +

k=i+1 2

ik |

n2 (n 1) < r (B) + . n3 ( + 1)

42

DE SISTEMAS LINEALES (PARTE II) 3. METODOS DIRECTOS PARA LA SOLUCION

w x v Ux x e1
2

Figura 3.1. Ilustraci on de (3.17), (3.18) (demostraci on del Teorema 3.6). Por otro lado, D1 U BUD

= m ax
x=0 B

D1 U BUDx x
.

= m ax
y=0

D1 U By . D1 U y

Entonces la norma deseada es x

:= D1 U x

Teorema 3.6. Sea x Cn , x = 0. Entonces existe una matriz U unitaria y hermitiana tal que Ux = exp(i ) x 2 e1 Espec camente para x Rn , podemos elegir U = I 2uuT , ( R apropiado).

donde uT u = 1 (Matriz de Householder).

Demostraci on. Sea x = exp(i) x 2 e1 , sino ponemos U := I 2e1 eT 1. Podemos ilustrar el problema de la siguiente forma: estamos buscando una transformaci on unitaria del vector x al primer eje de coordenadas. Cuando x ya es un m ultiple de e1 , podr amos elegir U = I. Pero, para lograr una f ormula u nica para todo vector x, ponemos U := I 2e1 eT 1

3.1. NORMAS DE VECTORES Y MATRICES

43

(cambio de signo). Ahora, si x no es un m ultiple de e1 , podriamos transformar x a x 2 e1 a trav es de una rotaci on. Pero como exigimos que U = U, U2 = I, la aplicaci on deseada debe ser involutiva, es decir, una reexi on (ver Figura 3.1). Ahora que conocemos el resultado de la aplicaci on a un vector x, podemos elegir U como la reexi on de x en un hiperplano con vector normal w, es decir, cuando x = w + v , debemos tener Esto se satisface para Uw = w y Uv = v. Si {w, v1 , . . . , vn1 } es un sistema ortonormal completo de Cn , entonces U = w v1 = w v1 = I 2ww , vn1 vn1 w v1 w v1 vn1 vn1

w v = 0 (v arbitrario),

(3.17) (3.18)

Ux = w + v.

2 w 0

0 w v1

vn1

con w w = 1. Ahora falta determinar el vector w. Queremos que Entonces, cuando w = (w1 , . . . , wn )T y := w x, se debe cumplir exp(i) x 2 0 , . x 2 w = . . 0 lo que signica que 2 n , . . . , wn = . 2 2 Queda para determinar y w1 . Sabemos que w2 = lo que es equivalente a 1 2 w1 = exp(i ) x w1 =
2

Ux = x 2(ww )x = exp(i) x 2 e1 .

( apropiado),

1 exp(i ) x 2 . 2 Si 1 = exp(i)|1 |, sea := + , entonces exp(i ) = exp(i) y luego w1 = En virtud de w w = 1 tenemos que 1 |1 |2 + 2|1 | x 4| |2 lo que implica exp(i)(|1 | + x 2 ) . 2 + x
2 2

+ |2 |2 + + |n |2 = 1,
2

4| |2 = 2 x

|1 | + x

44

DE SISTEMAS LINEALES (PARTE II) 3. METODOS DIRECTOS PARA LA SOLUCION

es decir, podemos elegir 2 = 2 x


2

|1 | + x

En este caso, w es determinado unicamente hasta un factor complejo de valor absoluto 1. Eso entrega exp(i)(|1 | + x 2 ) 2 1 w= . . . 2 x (| | + x )
2 1 2

Para la aplicaci on del Teorema 3.6 hay que tomar en cuenta que la matriz U puede ser escrita como exp(i)(|1 | + x 2 ) 2 , w , w := U = I w . . . n donde exp(i) = 1 si 1 = 0, 1 /|1 | sino, = 1 . x 2 (1 + x 2 )

con 1 = exp(i)|1 | y Ux = exp(i) x 2 e1 . Especialmente para x Rn tenemos que exp(i) = 1, entonces U = I 2uuT con u R, uT u = 1.

Para aplicar U a alg un vector y, tomamos en cuenta que es decir, nunca hay que almacenar la n n-matriz U, sino que s olo la informaci on esencial, y w. 3.2. El problema de la sensitividad para un sistema lineal Consideremos el siguiente problema: est a dado el sistema Ax = b, suponiendo que el del problema tiene una u nica soluci on. Ahora, cu al es la relaci on entre x y la soluci on x = b, cuando A A y b b son sucientemente peque sistema Ax nas? Del mismo 1 A1 . Empezamos con un caso simple, la tipo es el problema de estimar la norma A pertubaci on de la matriz unitaria. Teorema 3.7. Sea una norma vectorial sobre Cn . Como norma matricial sobre Cnn se usa la norma matricial asociada. Si H Cnn cumple H < 1, entonces I + H es regular y tenemos que 1 , (3.19) (I + H)1 1 H H (I + H)1 I . (3.20) 1 H y)w , Uy = y ( w

3.2. EL PROBLEMA DE LA SENSITIVIDAD PARA UN SISTEMA LINEAL

45

Demostraci on. Dea x = 0 un vector arbitrario. Primero hay que demostrar que (I + H)x = 0, lo que es equivalente a (I + H)x = 0. Pero (I + H)x = x + Hx x Hx = 1 H x H x x > 0.

Adem as, tenemos que

1 = I = (I + H)(I + H)1 (I + H)1 H(I + H)1 = 1 H (I + H)1 H (I + H)1 , (I + H)1

lo que implica (3.19). Por otro lado,

(I + H)1 I = (I + H)1 (I + H)1 (I + H) = (I + H)1 H H (I + H)1 ,

lo que demuestra (3.20). Corolario 3.2. Si r (H) < 1, entonces I+H es regular y tambi en en este caso el Teorema 3.7 es v alido. Demostraci on. Usar el Teorema 3.5. Corolario 3.3. Si r (H) < 1, entonces I + H es regular y (I + H)1 =
k=0

(1)k Hk
n

(Series de Neumann).

(3.21)

Demostraci on. Usamos el Corolario 3.2 y denimos Sn :=


k=0

(1)k Hk . 1 1 H

Entonces, para m > n,


m

Sn Sm

H
k=n+1

n+1

<

para n > N (), es decir existe el l mite S := l m Sn . En virtud de


n

Sn (I + H) = Sn + Sn H

46

DE SISTEMAS LINEALES (PARTE II) 3. METODOS DIRECTOS PARA LA SOLUCION


n n

=
k=0

(1) H
n+1

= I (1) resulta

k=0 n+1

(1)k+1 Hk+1

S(I + H) I = Sn (I + H) I + (S Sn )(I + H) 1 < H n+1 + S Sn 1 H para n > N (), mientras que la parte izquierda no depende de , lo que concluye la demostraci on de (3.21). con Corolario 3.4. Para una matriz A regular y otra matriz A A 1 es invertible, y A 1 A 1 A A1 Demostraci on. Usamos =A+A A = A I + A1 (A A) A y denimos A). H := A1 (A es regular. Luego obtenemos Entonces H < 1, por lo tanto I + H es invertible y A 1 A 1 A (I + H)1 A1 A1 = (I + H)1 I A1 A1 A H A 1 A . A 1 H 1 A1 A Denici on 3.5. Sea A regular. La cantidad cond (A) := A A1 se llama n umero de condici on de A para la soluci on de un sistema lineal. Kn . Sea Knn , b Teorema 3.8. Sea A Knn regular y 0 = b Kn , A matricial inducida por la norma vectorial y A 1 A < 1. A la norma A 1 A A 1 1 A1 A A . A < 1, A

3.2. EL PROBLEMA DE LA SENSITIVIDAD PARA UN SISTEMA LINEAL

47

satisface x =b Adem as, sea x := A1 b. Entonces la soluci on u nica de A b A x x b A 1 cond (A) + . A x b A A 1 cond (A) A 1 sigue del Corolario 3.4. Luego calculamos que Demostraci on. La existencia de A = A1 b + A1 (b b) + (I + A1 (A b), A))1 I A1 (b + b 1 b =A x

(3.22)

A), es decir, deniendo H := A1 (A b) + (I + H)1 I x + A1 (b b) . = x + A1 (b x A < 1, llegamos a Entonces, aprovechando (3.20) y H A 1 A x x x Dado que A
1

b A b A 1 A + A x 1 A 1 A A x = 1 x

1 + A 1

b b x

b = Ax y deniendo A 1

A , b

:= A

A A A 1 A A1 A A A

resulta la desigualdad x x x + A A 1 b b (1 + ). b (3.23)

Obviamente, siempre se tiene que cond (A) r (A)r (A1 ) 1. Cuando cond (A) 1, eso signica que la inuencia de errores menores (por ejemplo, de errores en A o en errores de redondeo) pueden causar cambios fuertes en la soluci on del sistema lineal. Se dice entonces que el sistema es mal acondicionado. (Recordamos que los errores de redondeo pueden ser interpretados como una modicaci on la matriz A seguida por la soluci on exacta del sistema.) Este problema se ilustra en el siguiente ejemplo. dados por x =b Ejemplo 3.1. Consideramos los sistemas Ax = b y A 1 1 1 2 3 6 0,5 0,337 0,165 A= 1 1 , b = 1 , A = 0,337 0,246 , b = 0,165 3 4 6

48

DE SISTEMAS LINEALES (PARTE II) 3. METODOS DIRECTOS PARA LA SOLUCION

con las respectivas soluciones x= En este caso, A A A

1 , 2

= x

1,5920898 . 2,8517654 cond (A) = 50.

(3.24)

0,007 6 = 0,0092, 0,83 x x x

b b = 0,01, b 16 = 1. 7, 9

Insertando esto en (3.22), resulta la cota

mientras que usando las verdaderas soluciones (3.24) x x = 0,42588, x es decir, en este caso la cota sobreestima el verdadero error relativo en un factor un poco m as de 4. Ejemplo 3.2 (Tarea 8, Curso 2006). Se considera la matriz 10 5 1 A := 8 9 1 . 0 1 3

, respectivamente. Determinar x sean la soluci =b Los vectores x y x on de Ax = b y A / x sin calcular x o x . una cota superior (la mejor posible) para x x Soluci on sugerida. a) Usamos A = D + B = D(I + D1 B), donde 1 1 0 2 10 8 1 1 . D B= 0 9 9 1 0 0 3 1 Dado que D B 1 = 8/9 < 1, la matriz A es invertible.

a) Usando el Teorema 3.7, demostrar que A es invertible. Aviso: Usar A = D + B, donde D = diag(a11 , a22 , a33 ). b) Determinar una cota superior para cond (A) en una norma apropiada sin invertir A o calcular det A. c) Adem as consideramos 10,1 10,05 5,1 1,05 10 = 9,8 , A = 8,1 9,1 0,95 . b = 10 , b 9,7 0,05 1 3,1 10

3.2. EL PROBLEMA DE LA SENSITIVIDAD PARA UN SISTEMA LINEAL

49

b) Obviamente A A 1

1 1

= 18. Usando la parte (a), tenemos = (I + D1 B)1 D1 1 D1 1 1 D B 1


1 1

(I + D1 B)1 1 1 = = 3, 8 3 1 9

D1

entonces cond c) Obtenemos

(A)

54.

b b b 1

= 0,02,
1

A A

= 0,2 0,2 1 54 18
1

x x = x 1

0,2 54 0,02 + 18

= 4,2.

Ejemplo 3.3 (Tarea 10, Curso 2006). Se desea resolver el sistema Ax = b con 1000 10 1 b1 A = 1000 0 0 , b = b2 , 1 b1 , b2 , b3 10. 1000 0 1 b3 Los coecientes de A y b han sido pertubados por ciertos errores A y b. a) Determinar cotas para y con := A , A := b b

tales que puede ser garantizado que / x < 0,01, xx donde Ax = b y (A + A) x = b + b. b) Supongamos que de la soluci on x nos interesa solamente la tercera componente. Indicar una transformaci on simple del sistema original que permite una cota signicativamente mejor (que la de (a)) de |x 3 x3 |/|x3 | en dependencia de las perturbaciones de los coecientes del sistema transformado. Soluci on sugerida. a) En lo siguiente, sea = x x x Dado que A1 0 0,001 0 0 0,1 , = 0,1 0 1 1
.

Tenemos con la notaci on indicada ( + ) cond (A) . 1 cond (A)

50

DE SISTEMAS LINEALES (PARTE II) 3. METODOS DIRECTOS PARA LA SOLUCION

sabemos que A = 1011, A1 = 2 y por lo tanto cond (A) = 2022. Sean , entonces x x s 4044 < 0,01. x 1 2022s Dado que 2as 1 as b = s b , a(2 + b)

s,

Ahora uno podr a pensar que debido a la t ecnica de estimaci on usada en la demostraci on del Teorema 3.8, siempre se sobreestima bruscamente el error, es decir la cantidad x x / x . Demostraremos ahora que esto no es as . Para tal efecto, vamos a construir matrices para las cuales esta cantidad alcanza la cota establecida por el lado de derecho de (3.22) hasta un error arbitrariamente peque no. Para construir tales matrices necesitamos el concepto de la descomposici on en valores singulares. Teorema 3.9 (Descomposici on en valores singulares). Sea A Cmn con m n. Enmm nn tonces existen matrices unitarias U C y V C y una matriz diagonal = diag(1 , . . . , n ) con elementos diagonales i 0, i = 1, . . . , n, tales que A=U V 0 (3.25)

resulta s = 2,47 106 . b) Dividimos la primera columna por 1000 y la segunda por 10, lo que es equivalente a resolver un sistema para ( x1 , x 2 , x3 ) con 1000 x1 = x1 y 10 x2 = x2 . En este caso, 1 1 1 0 1 0 = 1 0 0 A 1 = 1 0 1 cond (A ) = 6, s = 8,3 104 . A 1 0 1 0 1 1

Demostraci on. Las matrices AA y A A son ambas hermitianas y denidas semi-positivas, dado que x A Ax = Ax Por lo tanto, existe una matriz unitaria V tal que
2 2 V A AV = diag(1 , . . . , n ), 2 2 donde 1 , . . . , n son los valores propios (no negativos) de A A. Ahora, si 2 2

0. (3.26)

AA y = y, tenemos = 0 o A y = 0. Si A y = 0, entonces

y = 0,

A AA y = A y, o sea A y es un vector propio de A A y es el vector propio correspondiente, es decir, 2 2 {1 , . . . , n }. Entonces AA posee el valor propio 0 con multiplicidad m n (A y = 0 posee por lo menos m n soluciones linealmente independientes) y los valores propios

3.2. EL PROBLEMA DE LA SENSITIVIDAD PARA UN SISTEMA LINEAL

51

2 2 2 = 0). En virtud de lo anterior, existe una matriz (pueden ocurrir valores i , . . . , n 1 mm unitaria U C tal que

U AA U = La matriz B := A U satisface b1 . B B= . . b1 b m o sea Eso signica que B = b1 bn 0

2 0 . 0 0

2 2 bm = diag(1 , . . . , n , 0, . . . , 0),

0 ,

2 donde b i bj = i ij , i, j = 1, . . . , n.

0 B = V 0 =V unitaria. Dado que con una matriz V BB = A A = V2 V , es la misma matriz V que aparece en (3.26). Finalmente, resulta esta matriz V A = UB = U V . 0

Ejemplo 3.4 (Tarea 11, Curso 2006). Sea

Determinar matrices unitarias U C44 y V C22 y una matriz diagonal = diag(1 , 2 ) con 1 , 2 0 tales que A=U V . 0 0

5 5 A= 1,4 0,2

2,5 2,5 . 7,7 1,1

Aviso: calcular primero V de V A AV = 2 , y luego para la computaci on de U, usar AV = U

y un m etodo de ortonormalizaci on para las otras columnas de U. Soluci on sugerida. El polinomio caracter stico de la matriz A A = tiene los ceros
2 2 (52 )(73 ) = 1296 = 1 = 25 = 1 , 2 = 100 = 2 .

52 36 36 73

52

DE SISTEMAS LINEALES (PARTE II) 3. METODOS DIRECTOS PARA LA SOLUCION

Los vectores propios correspondientes son v1 = 0,8 , v2 = 0,6 0,6 0,8 = V = 0,8 0,6 . 0,6 0,8

Para determinar las primeras dos columnas de U, u1 y u2 , usamos que 2,5 5 2,5 5 AV = 3,5 7 = 1 u1 2 u2 , 0,5 1 donde 1 = 5 y 2 = 10. Entonces 0,5 0,5 u1 = 0,7 , 0,1 0,5 0,5 u2 = 0,7 . 0,1 0,5 0,7 0,1 0,5 0,7 0,1 . 0,7 0,02 0,14 0,1 0,14 0,98

La descomposici on en valores singulares sirve para la computaci on de la pseudo-inversa de Moore-Penrose de una matriz. Denici on 3.6. Sea A Cm,n . La pseudo-inversa de Moore-Penrose es una matriz A+ con A+ A = (A+ A) ,
+ + + +

Como dos vectores ortonormales adicionales podemos usar 0,5 0,1 0,7 0,5 0,1 0,7 u3 = 0,02 , u4 = 0,14 = U = 0,7 0,1 0,98 0,14

(3.27) (3.28) (3.29) (3.30)

AA = (AA ) , A AA = A , AA+ A = A. Se puede demostrar que la matriz A+ siempre existe y es u nica. Lema 3.1. Sea A Cmn , m n, y A=U V 0
+

la descomposici on en valores singulares de A. Entonces A+ = V + 0 U , Demostraci on. Tarea.


+ + + := diag(1 , . . . , n ), + i :=

1/i 0

si i > 0, sino.

3.2. EL PROBLEMA DE LA SENSITIVIDAD PARA UN SISTEMA LINEAL

53

Lema 3.2. Sea A Cnn una matriz regular con la descomposici on en valores singulares := b + un , A = UV con 1 2 . . . n > 0, b := u1 (la primera columna de U), b . En este caso, x := A un v , Ax = b y A =b > 0, A 1 x x x 2
2 2

1 1 1+ n 1

mientras que la evaluaci on del lado derecho de la desigualdad (3.22) entrega la cota , n es decir, para , la cota establecida por el Teorema 3.8 se alcanza hasta O(2 ). 1 x x x 2 1 1 1+ n 1 1

Demostraci on. Primero demostramos que para cada matriz A Cnn , cada vector b Cn y cada matriz unitaria B Cnn se cumple A
2

= BA 2 ,

= Bb 2 .

(3.31)

Para demostrar (3.31), notamos que b A


2 2 2 2

= b b = b B Bb = Bb 2 2, = r (A A) = r (A B BA) = r (BA) BA = BA 2 2.
2 2

Luego, usando A = UV y tomando en cuenta que r (VV ) = 1, tenemos A Dado que A obtenemos (usando x = v1 ): Ax
2 2 2

= U A

2 2

= V

2 2

2 2 ax i . 2 r (VV ) = m 1 i n

= m ax Ax 2 ,
x
2 =1

= UV v1

2 2

= e1

2 2

Luego derivamos una expresi on para cond (A). (En lo que sigue, usamos Para tal efecto, notamos que A1 = (UV )1 = (V )1 1 U1 = V1 U .

2 = 1 = A

= 1 . =

(3.32)
2 .)

Eso signica que A1 posee la siguiente descomposici on en valores singulares con las matrices := V, V := U: unitarias U 1 V , A1 = U y an alogamente a la derivaci on de (3.32) tenemos A1 = 1 = m ax
1 i

1 1 = . n i n (3.33)

Combinando este resultado con (3.32), tenemos cond (A) = 1 . n

54

DE SISTEMAS LINEALES (PARTE II) 3. METODOS DIRECTOS PARA LA SOLUCION

Ahora consideramos el sistema lineal Ax = b con b = u1 . Entonces 1 1 1 V x = U u1 = e1 = V x = 1 e1 = e1 = x = Ve1 = v1 . 1 1 1 Por otro lado, i = Avi un v vi = UV vi un v vi = U(i ei ) un 1i Av 1 1 = 1 u1 un i ui si i = 1, sino.

, podemos usar la f Para determinar x ormula de Sherman-Morrison. Alternativamente, dado = v1 + vn . En este caso, el que b = u1 + un , podemos tratar un planteo de la forma x = b se transforma a sistema Ax (1 u1 un ) + n un = u1 + un , entonces 1 = 1 = = Ambos m etodos entregan = x lo que implica xx = x n 1 +1 1 1 v1 1 vn = 1 n 1 +1 . 1 1 v1 + 1 n 1 + 1 vn , 1 1 , 1 n = = = n 1 +1 . 1

Ahora, evaluando la parte derecha de la desigualdad (3.22) del Teorema 3.8, tenemos xx x = Queda para demostrar que
un v1 = 1.

1 n

un + u1

un v1 1

1 n

1 un v1 1

1 un v1 1+ n 1

1 . 1 un v1 n (3.34)

Sea x Cn . Entonces podemos escribir x=

i vi ,
i=1

3.2. EL PROBLEMA DE LA SENSITIVIDAD PARA UN SISTEMA LINEAL

55

y entonces para x = 0
n x un v1

= un

v1 i=1

i vi

= 1 un = |1 | 1.

|1 | un v1 x = = 2 x (|1 | + + |n |2 )1/2 Por otro lado, para x = v1 tenemos un v1 x = 1, x

asi que (3.34) es v alido. Finalmente, denimos 1 1 f () := = f () = 1 n 1 n n

2,

es decir, para un (0, ) sigue (desarrollando f () por = 0) f ( ) = 1 + 2, n 1 n entonces 1 1 xx 1+ 1+ x n 1 n (1 /n )2 1 1 1 1 1 . = 1+ + 2 2 1 + n 1 1 (1 n /n )2


=O(2 )

Entonces, si A Cnn y A = UV es la descomposici on en valores singulares, sabemos que A es regular si y s olo si es regular, y A A1 cond

2 2

= m ax{1 , . . . , n }, si 1

(A) = 1 /n

1 1 , . . . , n }, = m ax{1

...

n > 0.

En muchos casos, los coecientes A y b de un sistema lineal se conocen solamente aproximadamente, a lo m as se puede estimar el orden de magnitud de los errores. Sean A0 y b0 dados con En esta situaci on no hay mucho sentido en resolver el sistema lineal A0 x0 = b0 con gran , cuya exactitud corresponde a la exactitud, sino que se busca una soluci on razonable x de A0 y b0 . Sorpresivamente, podemos decidir sin conocer el n umero de condici on de A0 si es una aproximaci x on razonable de x0 . |A0 A| E,
n E Rn + ,

|b0 b|

d,

d Rn +.

56

DE SISTEMAS LINEALES (PARTE II) 3. METODOS DIRECTOS PARA LA SOLUCION

Teorema 3.10 (Criterio de Prager & Oettli). Sean


n n Rn es dado. Entonces existen una matriz A A donde E Rn + , d R+ son dados, y x = b si y s y un vector b B tales que Ax olo si

A := A | |A A0 |

E ,

B := b | |b b0 |

d ,

se llama soluci En este caso, el vector x on aproximada compatible con A0 x0 = b0 . Demostraci on. Hay que demostrar dos implicaciones. = b. Entonces, dado que b Ax = 0, podemos escribir: : Supongamos que Ax | = b0 b + b Ax + (A A0 ) |b0 A0 x x |b0 b| + (A A0 ) x | d + |A0 A||x |. d + E|x

| |b0 A0 x

| + d. E|x

(3.35)

: Supongamos que

Entonces denimos
i

| |b0 A0 x

| + d, E|x

con E = (ij ), d = (1 , . . . , n )T . i = 1, . . . , n.

Sabemos que | i |

), := eT i (b0 A0 x i , es decir
i

| + d , i := eT i E|x 1 para i = 0.
(0) (0)

Adem as, supongamos que A0 = (ij ), b0 = (1 , . . . , n )T . Ahora vamos a construir = b. Sean A = (ij ), expl citamente una matriz A y un vector b tales que Ax 1 , . . . , n ) tales que = ( b = (1 , . . . , n )T y x (0) (0) i i si = 0, i ij sgn(j ) ij + si i = 0, i i i ij = i = i (0) (0) sino. ij sino, i Entonces tenemos que |A A0 |
n

(0)

E, |b b0 |

d, y

b) = eT i (Ax

n (0) (0) ij =0 j i j =1
n =
i

j =1

i i ij si i = 0,
n

i (0) (0) ij j i + i j =1

j ) j + i = 0 sino. ij sgn(
j =1 =i

3.2. EL PROBLEMA DE LA SENSITIVIDAD PARA UN SISTEMA LINEAL

57

El sistema de desigualdades | |b0 A0 x | + d E|x | d E|x b0 A0 x | + d E|x representa un sistema de desigualdades lineales por trozos, dado que hay desigualdades dife . El conjunto rentes para las 2n distribuciones posibles de los signos de las componentes de x de sus soluciones, es decir, el conjunto de las soluciones aproximadas compatibles con (E, d) se reduce para E = 0, d = 0 al punto x0 , la soluci on de A0 x0 = b0 . Si para (E, d) peque no el conjunto es grande, eso signica que la matriz A es mal acondicionada. Las consideraciones de esta secci on son de gran inter es para el an alisis del efecto de errores de redondeo durante la soluci on de sistemas lineales. Los computadores y las calculadoras representan un n umero real con un n umero jo de d gitos en la forma
t

k i=1

i i ,

{2, 10, 16},

donde es la base de la representaci on, k es un n umero entero (en un cierto rango) y i {0, 1, . . . , 1}, con 1 = 0 si = 0. Ahora, las operaciones aritm eticas con tales n umeros no entregan precisamente n umeros del mismo tipo. El redondeo tiene como consecuencia que las operaciones aritm eticas no pueden ser ejecutadas de forma exacta, sino que s olo de forma aproximadamente exacta como aritm etica de m aquina o pseudo-aritm etica. Ahora, si m(# ), # {+, , , /} # {+, , , /}, (3.36) denota esta aritm etica, se puede demostrar que cas siempre tenemos m(# ) = (# )(1 + ), donde | | , y := t+1 es la exactitud de m aquina. Ahora, si usamos el algoritmo de Gauss con esa aritm etica de m aquina y x denota el resultado, se puede demostrar (usando (3.36)) que (A + E)x = b, donde A y b son los coecientes de entrada realmente usados y la matriz E satisface la siguiente desigualdad: E

1,2(n3 + n2 )
(k )

si n i, j

0,09, n, 1 k n .

:= m ax ij | k

(3.37)

El valor de depende decisivamente de la estrategia del pivoteo. Junto con la tarea de hacer ejecutable el algoritmo (evitando divisiones por cero), la estrategia de pivoteo sirve para garantizar que no crece demasiado. Se puede demostrar que 2n1 m ax |ij |
1 i,j n

con b usqueda del pivote en la columna, y n 2 31/2 n1/(n1) m ax |ij |


1 i,j n

con b usqueda del pivote en la matriz restante.

58

DE SISTEMAS LINEALES (PARTE II) 3. METODOS DIRECTOS PARA LA SOLUCION

Los valores de / m ax1 i,j n |ij | observados en la pr actica son de la magnitud entre 1 y 10. Eso signica que x es la soluci on exacta de un sistema lineal con datos de entrada ligeramente modicados, por ejemplo datos compatibles en el senito del criterio de Prager & Oettli con d = 0 y 1 1 . . . E = 1,2(n3 + n2 ) . . . . 1 1 En virtud de esta discusi on, el algoritmo de Gauss con estrateg a de pivoteo se llama benigno o estable. Si se calcula la descomposici on triangular de una matriz singular con estrateg a de pivoteo ( i) en aritm etica de m aquina, al lugar de una columna con ki = 0 para k i aparece una columna con ki
( i)

c,

i,

(3.38)

donde c es una constante similar a la de (3.37). Eso signica que bajo el efecto de errores de redondeo, ya no podemos seguramente detecter la singularidad de una matriz. Se terminar a la computaci on al detectar (3.38) con c = n . Entonces, usando aritm etica de m aquina es posible que no se puede decidir cual es el rango de una matriz. La descomposici on en valores singulares es una buena herramienta para la denci on de un rango num erico de una matrix A. Por supuesto, esta denici on considera la incerteza en los elemntos de A y la exactitud de la aritm etica usada. Por ejemplo, una aproximaci sea A on de A (por ejemplo, por redondeo) con A A , conocida, y = U V A 0 con los valores singulares 1 . . . n > 0 y U, V unitarias. Entonces se aceptar an solo aquellos valores singulares como intrinsicamente diferentes de cero para los cuales i > . Si en tal caso tenemos 1 . . . r > r+1 . . . 1 , el n umero r se llama rango num erico o pseudo-rango de A. Ejemplo 3.5 (Certamen 1, Curso 2010). a) Calcular una descomposici on triangular PAQ matriz restante, de la matriz 1 1 2 1 1 3 A= 2 1 3 1 2 1 = LR, con b usqueda de pivote en la 4 4 . 6 8

(3.39)

Indicar expl citamente las matrices P, Q, L y R. b) Resolver el sistema Ax = b, donde b = (15, 7, 24, 17)T . c) Sean e := (1, 1, 1, 1)T , E := eeT , y d := e. Decidir si los siguientes vectores son una soluci on aproximada (en el sentido del criterio de Prager & Oettli) del sistema

3.2. EL PROBLEMA DE LA SENSITIVIDAD PARA UN SISTEMA LINEAL

59

Ax = b (i) para = 0,1, (ii) para = 0,5: 1,01 1,98 x1 := 3,99 , 2,01 Soluci on sugerida. a) Se obtiene la siguiente sucesi on de esquemas, donde los elementos marcados con estrella corresponden a multipolicadors y los elementos con marco son el pivote actual: 1,05 1,95 x2 := 3,95 . 2,05

1 2

1 1 1

2 1 1 1 2 4 8 1 2

3 2 3 3 1 2 2 2 1 2

4 4 4 6 8 3 1 5 2 15 4 5 2 2 2

15 7 24 17 1 1 3 2 11 4 1 2

4 8

2 2 1 1 1 4

3 1 3 3 2 4 8

1 1 1

17 7

3 2 4 1

2 4 3 1 6 4

2 24 1 3 1 15 2 2 1 1

17

17 45 4 31 2 13 2

2 3 1

3 4 1 2 4 8 3 4 1 2 1 2

2 3 1

31 3 3 2 4 1 45 2 2 4 1 13 1 2 2 17 4 4 8

15 1 11 4 2 4 5 3 2 2 2 5 1 2 2 2 3 1 1 1 2 2

4 3

1 1

17

15 1 11 4 2 4 2 7 3 3 2 5 3 3 10 3 7 3

45 3 4 8 1

2 1

3 4 1 2 2 1 1 2

15 11 1 45 4 4 2 4 2 10 7 8 3 3 3 2 7 5 1 3 3 3

60

DE SISTEMAS LINEALES (PARTE II) 3. METODOS DIRECTOS PARA LA SOLUCION

4 3

4 8

3 1

1 1

2 2

17

3 4 1 2 2 1 1 2 es decir

45 15 11 1 4 4 2 4 , 2 10 7 8 3 3 3 2 7 33 33 3 10 10 5 0 0 P= 0 1 1 0 0 0 1 0 0 0 1 0 0 0 1 0 ; 0 0 0 0 0 0 Q= 0 1 1 0 8 1 15 0 4 R= 0 0 0 0 1 0 0 0 1 0 1 , 0 0

3 1 0 0 4 L= 1 2 , 1 0 2 3 1 2 7 1 2 3 10 b) Utilizando el u ltimo esquema obtenemos 33 5 = 2, x = 3 x2 = 1 33 10 10 4 45 11 + + 1 = 4, x4 = x3 = 15 4 4

11 4 10 3 0

1 2 . 7 3 33 10

14 3

= 1,

1 (17 + 4 1 4) = 2. 8

c) Aqu se calcula para b0 = b, A0 = A: 0,05 0,2 0,08 , |b0 A0 x2 | = 0,35 , |b0 A0 x1 | = 0,01 0 0,06 0,4 9,99 10 9,99 T 10 eeT |x1 | + e = 9,99 , ee |x2 | + e = 10 . 10 9,99 Puesto que |b0 A0 xi | < 0,1 eeT |xi | + e < 0,5 eeT |xi | + e ,

3.3. CUADRADOS M INIMOS Y TRANSFORMACION A FORMA TRIANGULAR SUPERIOR

61

para i = 1, 2, ambos vectores x1 y x2 son una soluci on aproximada (en el sentido del criterio de Prager & Oettli) para = 0,1 y = 0,5. 3.3. El m etodo de cuadrados m nimos y la transformaci on de una matriz n n a una matriz triangular superior En muchas aplicaciones se presenta el siguiente problema. Para una matriz A Rmn (en general, m n) y un vector b Rm se busca un vector x Rn tal que x Rn : Ax b
2 2

Ax b 2 2.

(3.40) 3, buscamos

Por ejemplo, cuando tenemos puntos de mediciones (ti , yi ), i = 1, . . . , m, m una funci on


2 t 0 + 1 t + 2 t

y , 1 que aproxima nuestros datos. Para tal efecto, hay que determinar los coecientes 0 2 o ptimos, en el sentido de m 2 yi (0 + 1 ti + 2 ti ) 2 m

i=1

0 ,1 ,2 R

m n

i=1

yi (0 + 1 ti + 2 t2 i) .

Deniendo

obtenemos el problema planteado (3.40). El problema (3.40) signica que entre todas las combinaciones lineales de las columnas de A, buscamos la combinaci on que minimiza la distancia (euclidiana) del vector jo b. El vector b puede ser interpretado como una funci on sobre {1, . . . , m}. Eso motiva la denominaci on aproximaci on lineal discreta en L2 para el problema (3.40). Este tipo de aproximaci on fue por primera vez denido por Gauss (m etodo de cuadrados m nimos). Tiene una motivaci on que proviene de la estad stica: si los yi son hasta ciertos errores i iguales a 0 + 1 ti + 2 t2 i , entonces resulta que 0 , 1 y 2 , en un sentido, son las mejores aproximaciones a los verdaderos valores 0, 1 y 2 ; de tal forma, el problema (3.40) minimiza la inuencia de los errores i para la determinaci on de los s (compensaci on de la inuencia de los errores). El problema (3.40) admite una soluci on elemental si usamos el Teorema 3.6 y las matrices de Householder. Primero, recordamos que Q(Ax b)
2 2

y1 . b= . . , ym

0 , x = 1 2

1 t1 t2 1 . . . . A = . . . . . 1 tm t2 m

para cada matriz ortonormal Q Rmm . Supongamos ahora que se conoce una matriz ortonormal Q tal que QA = R , 0 R Rnn triangular superior.

= Ax b

2 2

62

DE SISTEMAS LINEALES (PARTE II) 3. METODOS DIRECTOS PARA LA SOLUCION

Ahora, denimos Qb =: c = Entonces, Q(Ax b) x Rn :


2 2

c1 , c2

c1 Rn ,
2 2

c2 Rmn .

R c x 1 0 c2
2 2 2 2

= Rx c1
2 2

2 2

+ c2 2 2.

Supongamos que rango(A) = n. En este caso, R es regular y Rx c1 + c2 c2 + Rx c1


=0 2 2,

es decir, x es la soluci on del sistema escalonado Rx = c1 , o sea x = R 1 c1 . Entonces, cuando se ha encontrado la matriz de transformaci on Q, solamente hay que aplicar Q a b y resolver el sistema escalonado Rx = c1 . La determinaci on de Q (y entonces la de R) se ejecuta en n pasos (o n 1 pasos si n = m). Sea (1) 1j . (1) (1) (1) (1) A = a1 an , b := b, aj := . . . nj
(1)

Deniendo

sgn0 (x) :=

1 si x = 0, sgn(x) sino,

T 1w 1 denida por formamos una matriz U1 ortonormal y sim etrica U1 := I 1 w (1) (1) (1) sgn0 11 11 + a1 2 (1) 1 . 21 1 := (1) , w = 1 (1) (1) . . a1 2 11 + a1 2 . (1) m1

(3.41)

Entonces tenemos

U1 a1 = sgn0 11 Ahora denimos

(1)

(1)

a1 e1 .

T (1) 1 1. b(2) := U1 b(1) = b(1) 1 w b w

(2) (1) (1) T (1) 1 1, ai := U1 ai = ai 1 w ai w

i = 2, . . . , n,

3.3. CUADRADOS M INIMOS Y TRANSFORMACION A FORMA TRIANGULAR SUPERIOR


(1) a1

63
(2) a2 ,

Queremos aplicar la misma t ecnica que para a las u ltimas m 1 componentes de mientras que la nueva transformaci on debe dejar sin cambio a la la 1 y la columna 1 de la matriz transformada. Eso se logra deniendo 1 0 0 0 , U2 = . T . 2w 2 . I 2 w 0 donde 2 = w sgn0 22
(2) (2) (2) 22 + a 2 2

2 :=

1 (2) a 2 2
(2) 22

(2) a 2 2

32 . . .

(2)

(2) m 2

As se continua la construcci on. En general, obtenemos el siguiente algoritmo. Algoritmo 3.1. 1. Denici on de n : n 2. do i = 1, . . . , n
(i) ai i) ( a i i) ( a i

(2) a 2

(2) 22 . := . . . m2
(2)

n 1 si m = n, n sino. ( i) b , ( i) b

b(i) =

i) (i) ( a Rmi+1 , i ,b

i) ( if a i = 0 then

i else

1
i) ( a i 2 (i) ( i) i ii + a 2

i w

sgn0 ii

( i)

i) (i) ( ii + a i

i+1,i . . . mi
(i)

(i)

endif do j = i, . . . , n

i 0,

i 0 w

(o sea, Ui = I)

i+1) i) T ( i) ( ( i j w i, a a a j j i w (i+1) b (i) , b

i+1) i) ( ( a a j j ,

64

DE SISTEMAS LINEALES (PARTE II) 3. METODOS DIRECTOS PARA LA SOLUCION


T ( i) (i+1) b (i) i w i i b b w

enddo enddo 3. El resultado de los pasos anteriores es

Un . . . U1 A = a(2) 1
=:Q

an

(n +1)

Un . . . U1 b = b(n +1) =

c1 . c2

(2) 11 0 . . . = 0 0 . . . 0

12

(3) (3)

22 ...

(n +1) 1n . . . . ... . . R (n +1) = , 0 nn 0 0 . . .

Como producto secundario de este algoritmo obtenemos una demostraci on del siguiente teorema. Teorema 3.11. Sea A Rmn con m Rmm tal que n. Entonces existe una matriz ortonormal Q R , 0

QA =

donde R es una matriz triangular superior. Si rango(A) = n, entonces el problema (3.40) tiene una u nica soluci on x, la cual se calcula de Rx = c1 , donde Qb = En este caso, R es regular. El m etodo descrito aqu es conocido como transformaci on de Householder u ortogonalizaci on de Householder. Esta nomenclatura se explica de la siguiente forma: QA = R R QT = A = QT = QT 1 2 0 0 R = QT 1 R, 0 c1 , c2 c1 Rn .

es decir, las n columnas de QT 1 (o sea las primeras n las de Q) forman una base ortonormal m del subespacio de R generado por las columnas de A, y las m n u ltimas las de Q son una base ortonormal del complemento ortogonal, es decir, del espacio nulo de AT . Hay que tomar en cuenta, sin embargo, que la matriz Q solo aparece en forma factorizada. Para el c alculo de A = UR, UT U = I, U Rmn (donde U corresponde a QT 1) podr amos tambi en usar el m etodo de ortogonalizaci on de Gram-Schmidt. Pero este m etodo es inestable num ericamente, as que se preere la transformaci on de Householder. Por otro lado, uno podr a aplicar el m etodo en el caso m = n, es decir, para la soluci on de sistemas. El esfuerzo es el doble del algoritmo de Gauss, asi que efectivamente se preere el

3.3. CUADRADOS M INIMOS Y TRANSFORMACION A FORMA TRIANGULAR SUPERIOR

65

algoritmo de Gauss, sobre todo en virtud de la equivalencia de las propiedades de estabilidad de ambos algoritmos. Ejemplo 3.6. Para la transformaci on de Householder de la matriz 4 3 4 1 A= 4 3 4 1 calculamos sucesivamente las siguientes cantidades: 4 4 = 4 , 4 12 4 1 = w 4 , 4

(1) a 1

1 =

1 1 = , 8(4 + 8) 96

luego 8 0 1 (1) 1 = = a1 (48 + 16 + 16 + 16)w 0 , 96 0 4 4 3 1 (1) = a2 (12 3 + 4 1 + 4 3 + 4 1) = 2 . 96 3 4 3 10 3 2 2 = w 3 , 4 3

a1

(2)

a2

(2)

Despu es, calculamos

(2) a 2

1 = (16 + 16 + 4)2 = 2, 2 = 3

1 2 4 +2 3

20 1 T (2) 2 2 = (10 4 + 2 2 + 4 4) = . w a 9 3

66

DE SISTEMAS LINEALES (PARTE II) 3. METODOS DIRECTOS PARA LA SOLUCION

Finalmente resulta

a2

(3)

Ejemplo 3.7 (Tarea 13, Curso 2006). Sea A la matriz indicada, y buscamos una descomposicion QR de A con R = QT A. La matriz R dada abajo puede ser correcta? 2 3 2 10 43 4 2 1 0 0 7 0 . A= 1 2 3 , R = 0 0 1 1 4 2 0 0 0 Soluci on sugerida. La matriz R debe ser incorrecta, dado que 432 + 72 = 32 + 12 + 22 + 42 , y porque una aplicaci on ortogonal no cambia la norma 2 de un vector. Sea A = a1 an y R = r1 rn con vectores de columnas ai y ri . Entonces Qai = ri implica que Qai 2 = ri 2 , y como Q es ortogonal, ai 2 = ri 2 . Ejemplo 3.8 (Tarea 15, Curso 2006). Sean 1 1 1 1 1 1 A= 1 1 , b1 = 1 , 1 1 1 2 T 2 u 2u 1 1 b2 = 1 . 1

4 4 10 4 3 3 2 =2 2= , 0 3 3 0 4 4 3 3

o sea

8 4 0 2 . U2 U1 A = 0 0 =Q 0 0

1 y u 2 de a) Determinar la descomposici on QR de A. Para Q, indicar s olo los vectores u la representaci on Q= I 2u T u 2 I 2 T 1 u 1u 1u T u . 1

b) Calcular c1 := Qb1 y c2 := Qb2 . c) Usando los resultados de (a) y (b), determinar la soluci on del problema de compensaci on Ax b1 tb2 para t R arbitrario. Solucion sugerida. a) La matriz dada es de la forma
(1) A = a(1) a2 1 2

= m n 2
x R

3.3. CUADRADOS M INIMOS Y TRANSFORMACION A FORMA TRIANGULAR SUPERIOR

67

con
(1)

a1

T 1 = 12, u 1u

1 1 = 1 , 1

1 = a1

(1) 2

= 2,

1 1 = , 6

(1) T u 1 a1 = 6,

2 1 1 = u 1 , 1

(1) T u 1 a2 = 2.

Entonces obtenemos los vectores 2 0 (2) a1 = 0 , 0 Luego calculamos que 0 1 8 , 2 = u 3 4 4

a2

(2)

0 1 2 . = 3 4 4 0 2 = 0 , 0

2 = 2, o sea

(2) T u 2 a2 =

48 , 9

2 =

9 , 48

a2

(3)

R= b) Calculamos sucesivamente

2 0 . 0 2

c) Aqu obtenemos

0 0 1 1 4 4 (2) (1) (2) (1) (1) T (1) , T u = , b = b u b b1 = b1 1 u u = b 1 1 1 2 2 1 1 1 2 3 2 3 4 4 2 0 0 0 (2) (3) (2) (3) (3) T (3) 0 . T 2 = b1 = b1 2 u u , b = b u b u = 3 3 2 2 2 2 b1 3 0 2 2 0 Ax b1 tb2
2

= Q(Ax b1 tb2 ) = R x c1 tc2 0

es decir, para todo t R, el m nimo se asume para x = 0.

2x1 2x2 = 2t 2

0 0 2t 2

,
2

68

DE SISTEMAS LINEALES (PARTE II) 3. METODOS DIRECTOS PARA LA SOLUCION

Usar el m etodo de Householder para determinar una descomposici on QR de A. Luego determinar un vector x que minimiza Ax b 2 . Cu al es entonces el error en la ecuaci on Ax b? Soluci on sugerida. 1. Consideramos la primera columna a1 de A. Usando a1 2 = 9, obtenemos u = (12, 6, 6, 0)T y = 1/108. Con P1 = I uuT determinamos P1 A calculando para cada columna ai de A el vector P1 ai . El resultado es 9 3 3 1 0 0 4 5 P1 A = 0 3 2 , P1 b = 0 . 0 0 2 11 11 2. Luego, para la primera columna 0 1 = 3 a 0

Ejemplo 3.9 (Tarea 16, Curso 2006). Sean 3 1 1 6 1 6 A= 6 4 1 , 11 0 0

3 2 b= 2 . 2 11

3. Luego, para la primera columna

1 2 = 3, y entonces u = (3, 3, 0)T y = 1/9. de la matriz 3 2 restante obtenemos a Obtenemos 3 2 0 2A = 0 4 . P 5 , P2 b = 0 2 11 11 1 = a 5 11

1 2 = 6, y entonces u = (11, 11)T y = 1/66. de la matriz 21 restante obtenemos a Obtenemos 7 = 3A = 6 , P 3b P . 0 11 Despu es de los 3 pasos anteriores, obtenemos la siguiente descomposici on QR, donde Q es el producto de las matrices de Householder ampliadas a la dimensi on 4: 1 9 3 3 0 3 2 0 QA = 0 0 6 , Qb = 7 . 0 0 0 11

3.3. CUADRADOS M INIMOS Y TRANSFORMACION A FORMA TRIANGULAR SUPERIOR

69

Particionando la matriz del tama no 4 3 en una matriz del tama no 3 3 y otra del tama no 1 3, obtenemos
x R

m n Ax b 4

= m n QAx Qb 4
xR

= m n 4
x R

R c x 1 0 c2

.
2

Obviamente, el sistema Rx = c1 posee una soluci on u nica, entonces 11. m n Ax b 2 = c2 2 = 4


xR

Ejemplo 3.10 (Certamen 1, Curso 2010). Resolver el problema de aproximaci on


m

i=1

yi

(0 0 (ti )

1 1 (ti )

2 2 2 ti

= m n

0 ,1 ,2

i=1

yi (0 0 (ti ) + 1 1 (ti ) + 2 2 ti

para los datos i 1 2 3 4 ti 0 1 2 3 yi 1 -1 1 3 para i (t) = ti , i = 0, 1, 2, transformando la mediante la transformaci on de Householder. Soluci on sugerida. Sea 1 1 t1 t2 1 1 t2 t2 1 2 = A= 1 t3 t2 1 3 1 1 t4 t2 4 matriz A R43 a forma triangular superior 0 1 , 4 9 1 1 b= 1 . 3 3 1 1 = w 1 . 1

0 1 2 3

La transformaci on de Householder se ejecuta en tres pasos. 1. Desde la matriz A identicamos 1 1 1 1 (1) 1 = a 1 , 1 = 2(1 + 2) = 6 , 1 Si

1 T 1w 1 R44 , U1 = I w 6 obtenemos
(2)

a1

1 2 1 0 = U1 1 = 0 , 0 1

a2

(2)

0 3 1 0 = U1 2 = 1 , 3 2

70

DE SISTEMAS LINEALES (PARTE II) 3. METODOS DIRECTOS PARA LA SOLUCION

a3

(2)

2. Considerando los 3 u ltimos componentes de los vectores anteriores obtenemos 0 5 1 1 . 1 (2) = a = , = , w = 1 2 2 2 5 5(0 + 5) 2 2 Deniendo 1 2w 2 T R33 U2 = I w 5 obtenemos 0 5 (3) a2 = U2 1 = 0 , 2 0 4 3 5 3 6 , 7082 5 4 4 5 (3) a3 = U2 3 = 3 + 15 0,7370 , 1,8592 20 2 8 5 + 3 15 3 4 5 5 2 1,7889 4 2 5 0,0944 . b(3) = U2 0 = + 5 5 2 2,1889 2 45 + 5 5

4 0 1 3 = = U1 , 4 5 9 3 20 3

b(2)

2 2 = U1 b = 0 . 2

3. Considerando los 2 u ltimos componentes de los vectores anteriores obtenemos 4 4 5 + 0,7370 15 3 (3) 3 = a 1,8592 , 2 8 5 + 3 15

3.3. CUADRADOS M INIMOS Y TRANSFORMACION A FORMA TRIANGULAR SUPERIOR

71

3 =

Deniendo

1 1 0,1827, = 20 8 5 4 5 2 1 +2 3 15 3 5 10 4 5 + 2,7370 15 3 3 = w 1,8592 . 2 8 5 + 3 15 U3 = I w 3w 3 T R22 20 8 5 3 15 1

con la soluci on

4 4 5 + 2 15 3 4 a3 = U3 = 0 , 2 8 5 + 3 15 4 2 5 2 + 2 5 5 b(4) = U3 = 2 0,8944 . 2 4 5 5 + 5 5 5 Concluimos que la soluci on del problema est a dada por el sistema lineal escaloneado 2 3 7 2 0 0 5 3 5 1 = 4 5 5 2 0 0 2 2 4 11 = 0,8, 1 = = 2,2, 2 = 1. 5 5 La tarea de minimizar Ax b 2 en puede ser tratada por m etodos del c alculo 2 tambi diferencial de varias variables. Para tal efecto, denimos la funci on
0 =

obtenemos

(x) := (Ax b)T (Ax b)

La condici on necesaria para un m nimo en x es = 0, i = 1, . . . , n, (x) i x=x lo cual entrega el sistema lineal (las ecucaciones normales) AT Ax = AT b,

= xT AT Ax 2xT AT b + bT b.

(3.42)

72

DE SISTEMAS LINEALES (PARTE II) 3. METODOS DIRECTOS PARA LA SOLUCION

con la soluci on u nica (si rango(A) = n) x = (AT A)1 AT b. El hecho de que x realmente es el m nimo se reeja en la satisfacci on de la condici on adicional suciente que 2 (x) i j
x=x ij

es denida positiva. Uno podria resolver (3.42) por la descomposici on de Cholesky. Pero este camino no es recomendado, con la excepci on del caso que las columnas de A son casi ortogonales. En efecto, el m etodo de Cholesky es mucho m as sensible para errores de redondeo que la transformaci on de Householder. Frecuentamente los problemas de compensaci on son extremadamente mal acondicionados, sobre todo cuando las funciones de planteo no son las apropiadas. Por ejemplo, para un planteo polinomial siempre se recomienda transformar la variable independiente al intervalo [1, 1] y usar los polinomios de Chebyshev T0 (x) = 1, T1 (x) = x, la u ltima medida para la soluci on del problema es la descomposici on en valores singulares. Si A=U V 0 y la matriz es invertible, entonces la soluci on del problema (3.40) es x = V 1 0 U b. Si no es invertible, la soluci on del problema de compensaci on no es u nica. En este caso, se usa la soluci on o ptima x = A+ b = V + 0 U b, con la longitud euclidiana m nima. Por supuesto, en la pr actica se remplaza + por + () = + diag(i ()),
+ i () =

Tn+1 (x) = 2xTn (x) Tn1 (x),

2.

1/i 0

si i sino,

donde el par ametro > 0 representa la inexactitud en A y en la aritm etica.

Cap tulo 4

M etodos iterativos para la soluci on de sistemas de ecuaciones lineales


4.1. Un ejemplo Muchas aplicaciones requieren la soluci on de sistemas de ecuaciones lineales de extremadamente gran tama no (n 104 ), pero donde la matriz de coecientes tiene una estructura muy especial: cada la contiene s olo muy pocos, por ejemplo cinco, elementos diferentes de cero, en una conguraci on muy particular. En esta situaci on no se recomienda el uso de los m etodos discutidos hasta ahora por razones de espacio de almacenaje y tiempo computacional. Vamos a ilustrar el problema en el siguiente ejemplo. Ejemplo 4.1. Se busca una funci on u : [0, 1]2 R que es soluci on del siguiente problema de valores de frontera (problema de Dirichlet) para la ecuaci on de Poisson: u u u = f (, ), u(, ) = 0, {0, 1} o {0, 1}. (, ) (0, 1)2 , (4.1)

Aqu f es una funci on real y continua dada sobre [0, 1]2 . La tarea de determinar la soluci on u num ericamente se reduce a un sistema de ecuaciones lineales para los valores aproximados Para una funci on z = z ( ) C 4 tenemos seg un la f ormula de Taylor 1 1 1 )h4 , z ( + h) = z ( ) + z ( )h + z ( )h2 + z ( )h3 + z (4) ( 2 6 24 1 1 1 )h4 , z ( h) = z ( ) z ( )h + z ( )h2 z ( )h3 + z (4) ( 2 6 24 = + 1 h y = 2 h. Combinando estas dos ecuaciones obtenemos donde z ( + h) 2z ( ) + z ( h) 1 )h2 . z ( ) = z (4) ( (4.3) 2 h 12 Entonces, para h peque no, el primer t ermino en el lado derecho de (4.3) sirve como buena aproximaci on de z ( ). Ahora [0, 1]2 se cubre con una malla cuadr atica de puntos (i , j ), 0 i, j N + 1, donde 1 i = ih, j = jh, 0 i, j N + 1, h = , N N. N +1 Ahora aproximamos las segundas derivadas parciales u y u usando (4.3). Usando la ecuaci on de derivadas parciales, obtenemos la siguiente ecuaci on para (4.2): 4 uij u i1,j u i+1,j u i,j 1 u i,j +1 = h2 f (i , j ),
73

u ij u(i , j ).

(4.2)

i, j

N.

(4.4)

74

DE SISTEMAS DE ECUACIONES LINEALES 4. METODOS ITERATIVOS PARA LA SOLUCION

Entonces, la aproximaci on genera un sistema de N 2 ecuaciones para N 2 desconocidas. Enumerando los pares (i, j ) en el orden (1, 1), (2, 1), . . . , (N, 1), (1, 2), (2, 2), . . . , (N, 2), . . . , (N, N ), resulta un sistema lineal Ax = b con u 11 u 21 , x= . . . u N N donde f (1 , 1 ) f (2 , 2 ) , b = h2 . . . f (N , N ) I . . I . . . . . . . RN 2 N 2 , .. .. .. A= . . . . . . I I B B

Si se trata aplicar la descomposici on de Cholesky a A, aprovechando la estructura de bandas, 4 resultan N /2 operaciones aritm eticas y N 3 elementos de almacenaje; estos n umeros ya son grandes para 50 N 200. Adem as, no es muy razonable tratar de resolver el sistema exactamente, dado que la soluci on u ij misma s olo representa una aproximaci on (con un error O(h2 ) si |f | 1) de los valores uij . Por supuesto, el caso an alogo tri-dimensional es a un mucho mas complicado. Comentamos que existe un algoritmo especial precisamente para el sistema discutido en este ejemplo, el algoritmo de Buneman, que para N = 2m+1 1 necesita s olo 3N 2 (m + 2 1) operaciones aritm eticas y approx. 6N elementos de almacenaje. Sin embargo, este este algoritmo ya fracasa si los elementos varian con (i, j ), con una matriz que sino tiene la misma estructura que A. Resumiendo, constatamos que se recomienda buscar m etodos simplemente estructurados que aproximan la soluci on x de un sistema lineal por una sucesi on {xk }kN innita, pero donde cada paso xk xk+1 requiere s olo muy poco esfuerzo computacional. 4.2. Metodolog a general del desarrollo de m etodos iterativos La idea b asica es la siguiente. Queremos resolver el problema Ax = b. La matriz A se descompone de la siguiente forma: A = M + N, entonces (4.5) es equivalente a Mx = b Nx . (4.6) (4.5)

4 1 1 4 1 ... ... ... B= RN N . ... ... 1 1 4

4.2. METODOLOG IA GENERAL DEL DESARROLLO DE METODOS ITERATIVOS

75

Entonces, introduciendo un factor y una matriz C arbitraria, (4.6) es equivalente a Las matrices C y M y el factor se eligen de tal forma que M + C es regular y tiene una estructura simple, de manera que un sistema lineal con esta matriz puede ser resuelto m as facilmente que el sistema original (4.5). Ahora, si remplazamos en la u ltima ecuaci on x en el lado derecho por xk y en el lado izquierdo por xk+1 , obtenemos el m etodo de iteraci on el cual podemos escribir como Seg un nuestra construcci on, ( M + C)xk+1 = (C N)xk + b, (4.8) ( M + C)x = (C N)x + b. (4.7)

xk+1 = ( M + C)1 (C N)xk + b =: (xk ). x = (x ), (4.9) (4.10)

es decir, x es el punto jo de la aplicaci on x (x); por lo tanto,


=:B( )

xk+1 x = ( M + C)1 (C N)(xk x ).

En virtud de (4.10), para cualquier vector inicial x0 , se cumple Teorema 4.1. El m etodo (4.8) converge para todo x0 Rn a x si y s olo si r B( ) < 1. xk x = B( ) (x0 x ).
k

(4.11)

Demostraci on. Sea r (B( )) < 1. Entonces I B( ) es regular, lo que signica que la ecuaci on x = B( )x + ( M + C)1 b tiene una u nica soluci on x . Seg un el Teorema 3.5, existe una norma vectorial (con una norma matricial inducida) tal que para un > 0 peque no dado, B( ) Entonces o sea, xk x
k

r B( ) + < 1. B( )
k

x 0 x ,

l m xk+1 = x . 1.

Por otro lado, en el caso cntrario r (B( )) 1, existe un valor propio de B( ) con || Sea v = 0 el vector propio asociado. Entonces B( ) v = k v. Sea x0 = x + v. En este caso, xk x = k v,
k

76

DE SISTEMAS DE ECUACIONES LINEALES 4. METODOS ITERATIVOS PARA LA SOLUCION

es decir, k N : xk x = |k v v > 0.

Para cualquier > 0 sucientemente peque no existe una norma tal que en esta norma, la distancia x x se reduce por un factor r (B( )) + en cada paso. Obviamente, hay que elegir M, N, C y de la forma que r (B( )) sea lo m as peque no posible. Antes de seguir estudiando la teor a de los m etodos (espec camente, el comportamento de B( ) en un caso especial importante), vamos a mencionar los m etodos mas importantes que se usan en la pr actica, deniendo las matrices L, D y U por medio de donde A = L + D U, 0 0 ... ... .. .. . . n,n1 (4.12)

L = .21 . . n1

0 12 1n 11 0 0 0 . . . ... . . . . . . . 0 0 0 22 . . . , U = . . , D = . .. .. .. . . . n1,n . 0 . . .. 0 . 0 0 0 0 0 nn 0 (4.13)

1. El m etodo de Jacobi es denido por M = D, N = L U, C = 0 y = 1. La f ormula de iteraci on vectorial correspondiente es xk+1 = D1 (L + U)xk + b , para las componentes obtenemos i,k+1 1 = i,k + ii
n

k N0 ; i = 1, . . . , n, k N0 .

(4.14)

ij j,k + i
j =1

(4.15)

2. El m etodo de Gauss-Seidel es denido por M = L + D, N = U, C = 0 y = 1. Las f ormulas de iteraci on son (L + D)xk+1 = Uxk + b, i,k+1 =i,k + 1 ii
i 1 j =1 n

k N0 ; ij j,k + i ,

(4.16) (4.17)

ij j,k+1

j =i

i = 1, . . . , n,

3. El m etodo SOR (successive overrelaxation) con el par ametro de relajaci on corresponde a M = L + D, N = U, C = (1 )D y = 0. Las f ormulas de iteraci on son ( L + D)xk+1 = (1 )D + U xk + b, i,k+1 =i,k + ii
i 1 n j =1

k N0 .

k N0 ; ,

(4.18) (4.19)

ij j,k+1

ij j,k + i
j =i

i = 1, . . . , n,

k N0 .

4.2. METODOLOG IA GENERAL DEL DESARROLLO DE METODOS ITERATIVOS

77

La identidad (4.18) ilustra la idea del m etodo SOR: tenemos como ultima (mejor) aproGS ximaci on de x el vector (1,k+1 , . . . , i1,k+1 , i,k , . . . , n,k )T . Despu es se calcula primero i,k +1 , aplicando el m etodo de Gauss-Seidel, luego se determina i,k+1 agrandando o achicando la correcci on de Gauss-Seidel por el factor . Ejemplo 4.2 (Tarea 18, Curso 2006). Se considera el sistema lineal Ax = b con A= 5 4 , 1 3 b= 12 . 2

a) Preparar un dibujo que interpreta ambas ecuaciones del sistema lineal como lineas rectas en el plano x1 -x2 . La soluci on exacta del problema es x 1 = 4, x2 = 2. T b) Ejecutar desde x0 = (8, 8) tres pasos de cada uno de los m etodos de Jacobi, de Gauss-Seidel, y del m etodo SOR con = 1,5 aus. Agregar al dibujo las sucesiones 1,0 , 2,0 1,0 , 2,0 1,1 , 2,0 1,1 , 2,1 1,1 , 2,1 1,2 , 2,2 1,2 ... 2,1 1,3 ... 2,3

en los casos de los m etodos de Gauss-Seidel y SOR y la sucesi on

en el caso del m etodo de Jacobi. Solucion sugerida. a) La Figura 4.1 muestra las dos rectas y las iteradas. 1,k , 2,k ) las del m b) Sean (1,k , 2,k ) las iteradas del m etodo de Jacobi, ( etodo de Gau Seidel y (1,k , 2,k ) las del m etodo SOR, entonces obtenemos las siguientes sucesiones de iteraci on: (1,1 , 2,1 ) = (4, 2), 4 2 , , (1,2 , 2,2 ) = 5 3 (1,3 , 2,3 ) = (1,8 6, 0,9 3); 1,0 , 2,0 ) = (8, 8), ( (1,0 , 2,0 ) = (8, 8),

1,1 , 2,0 ) = (4, 8), ( 1,1 , 2,1 ) = 4, 2 , ( 3 (1,2 , 2,1 ) = (1,86, 0. 6), 1,2 , 2,2 ) = (1,86 , 1,28) , ( 1,3 , 2,2 ) = (3,431 , 1,28) , ( 1,3 , 2,3 ) = (3,43 ( 1, 1,81037); 1,0 , 2,0 ) = (8, 8), (

78

DE SISTEMAS DE ECUACIONES LINEALES 4. METODOS ITERATIVOS PARA LA SOLUCION

Figura 4.1. Interpretaci on de las ecuaciones de un sistema lineal 2 2 como lineas rectas en el plano x1 -x2 y sucesiones de soluciones aproximadas

1,2 , 2,1 ) = (9,4, 4), ( 1,2 , 2,2 ) = (9,4, 3,7), (

1,1 , 2,0 ) = (2, 8), ( 1,1 , 2,1 ) = (2, 4), (

1,3 , 2,2 ) = (3,34, 3,7), ( 1,3 , 2,3 ) = (3,34, 0,82). (

4.3. TEOREMAS DE CONVERGENCIA PARA METODOS ITERATIVOS

79

4.3. Teoremas de convergencia para m etodos iterativos Teorema 4.2. El m etodo de Jacobi (4.14) converge si A = D L U satisface uno de los siguientes criterios: A es estrictamente diagonal dominante por las:
n

i = 1, . . . , n :

|ii | >

j =1 j =i

|ij |,

(4.20)

o A es estrictamente diagonal dominante por columnas:


n

i = 1, . . . , n : Demostraci on. Tarea.

|ii | >

j =1 j =i

|ji |.

(4.21)

Los requerimientos (4.20) y (4.21) son bastante restrictivos y no se cumplen para la mayor a de las matrices. En particular, muchas veces la desigualdad estricta en (4.20) o (4.21) no se cumple en todas las las o columnas, sino que s olo en algunas de ellas, mientras que en las otras |ii | es igual al lado derecho de (4.20) o (4.21). A continuaci on veremos que tambi en en este caso podemos asegurar la convergencia del m etodo (4.14), siempre cuando la matriz A cumpla la propiedad de irreducibilidad. Denici on 4.1. Una matriz A Cnn se llama irreducible si no existe ninguna matriz de permutaci on P tal que 11 A 12 A kk PT AP = 22 , A22 C , k < n. 0 A Para decidir sobre la irreducibilidad de una matriz A, necesitamos el concepto del grafo dirigido de la matriz. Denici on 4.2. Sea A = (ij ) Cnn . A la siguiente construcci on se reere como grafo dirigido G (A) de A: 1. G (A) incluye n v ertices P1 , . . . , Pn . 2. Un arco dirigido Pi Pj junta Pi con Pj , i = j , si y s olo si ij = 0. 3. Los caminos dirigidos en G (A) son compuestos por arcos dirigidos. 4. El grafo dirigido G (A) se llama conexo si para cada par (Pi , Pj ) de v ertices, 1 i, j n, i = j , existe un camino dirigido de Pi a Pj . Ejemplo 4.3. La Figura 4.2 muestra algunas matrices y sus grafos dirigidos. Nos damos cuenta que los grafos G (A), G (B) y G (C) son conexos; obviamente, G (D) no es conexo. Teorema 4.3. Una matriz A Knn es irreducible si y s olo si su grafo dirigido G (A) es conexo. Demostraci on. Hay que demostrar dos implicaciones. : Supongamos primero que G (A) es conexo, y sean los ndices i y j , i = j , 1 arbitrarios. Entonces existen n umeros i1 , . . . , im tales que i,i1 i1 ,i2 i2 ,i3 . . . im ,j = 0. i, j n

(4.22)

80

DE SISTEMAS DE ECUACIONES LINEALES 4. METODOS ITERATIVOS PARA LA SOLUCION

P1 1 0 3 A = 1 2 0 0 4 5 0 1 0 2

G (A ) : P3 P1 G (B ) : P4 P3 P2 P2

2 1 B= 1 0

1 0 3 1 0 1 1 1

2 1 1 2 . C= 0 . . . . . . .. 0 4 1 D= 0 1 0 1 2 0 2 0 0 0 1 0 1

0 .. . . . 1 . .. .. . . 0 1 2 1 0 1 2 0 1 1 0 3 0 0 0 1 0 1

P1 G ( C) :

P2

P3

P4

Pn

P1 G (D ) : P4

P2

P3

P5

Figura 4.2. Algunas matrices y sus grafos dirigidos. Ahora supongamos que A es reducible, es decir s,k = 0, s S, k K, S K = , S K = {1, . . . , n}. (4.23)

Entonces sean i S y j K . Dado que i,i1 = 0, tenemos que i1 K , o sea, i1 S , lo que implica i2 S , etc., entonces ser a imposible construir la cadena (4.22), una contradicci on. : Ahora sea A irreducible e i {1, . . . , n} arbitrario. Denimos I := k | {i1 , . . . , im } : i,i1 . . . im ,k = 0 , notando que I = , puesto que sino i1 = . . . = in = 0, una contradicci on. Supongamos que I = {1, . . . n}, y sea l {1, . . . n}\I . Demostramos que en este caso jl = 0 para j I . (Esto ser a una contradicci on a la irreducibilidad de A.) Si existiera

4.3. TEOREMAS DE CONVERGENCIA PARA METODOS ITERATIVOS

81

j0 ,l = 0 para un ndice j0 I , existir an tambi en ndices {i1 , . . . , im } tales que i,i1 i1 ,i2 . . . im ,j0 = 0, jo ,i = 0, o sea l I , una contradicci on. Esto implica que I = {1, . . . , n}, y dado que i es arbitrario, concluimos que G (A) es conexo. Denici on 4.3. Una matriz A Cnn se llama irreduciblemente diagonal dominante si A es irreducible y
n n

i {1, . . . , n} : |ii |

j =1 j =i

|ij | i0 {1, . . . , n} : |i0 i0 | >

j =1 j =i0

|i0 j |.

(4.24)

Teorema 4.4. Sea A estrictamente o irreduciblemente diagonal dominante. Entonces A es regular y el m etodo de Jacobi converge. Demostraci on. La demostraci on procede en tres pasos: 1. Demostramos que el m etodo est a bien denido. 2. Demostramos que r D1 (L + U) < 1. (4.25)

3. El resultado de 2.) implica que el m etodo de Jacobi converge para b arbitrario a una soluci on de Ax = b, lo que implica la regularidad de A. Enseguida procedemos a la demostraci on de 1.) y 2.): 1. Si A es estrictamente diagonal dominante, esta propiedad es obvia. Si A es irreduciblemente diagonal dominante, entonces ii = 0 para todo i (si existir a un ndice i0 tal que i0 i0 = 0, tendr amos que i0 1 = . . . = i0 n = 0, una contradicci on a la irreducibilidad). 2. Si A es estrictamente diagonal dominante, (4.25) es una consecuencia del Teorema 4.1. En el otro caso, podemos aplicar el Teorema 3.4, aplicado a B := D1 (L + U) =: (ij ), para concluir que r (B) B

1.

Supongamos que r (B) = 1. En este caso existe C, || = 1, tal que (B I)x = 0 con x = 0. Por otro lado, B I es una matriz irreduciblemente diagonal dominante, dado que diere de D1 A s olo en sus elementos diagonales, donde 1 es remplazado por con | | = 1. Supongamos ahora que x = (1 , . . . , n )T con |1 | = . . . |n | = . Esto signica que
n n

i {1, . . . , n} :

ij j =
j =1

j =1

|ij |,

82

DE SISTEMAS DE ECUACIONES LINEALES 4. METODOS ITERATIVOS PARA LA SOLUCION

o sea,
n

i {1, . . . , n} :

j =1

|ij |

1,

en contradicci on a la supuesta diagonaldominancia irreducible de A. Entonces Sea j I . Observando que ii = 0, tenemos que
n

I := j | i : |j | 0=
i=1

|i |, i0 : |j | > |i0 | = .
n

ji i j j =
n n

ji i
i=1

= |j | Notamos que |i |/|j | 1


iI

i=1

|ji ||i | =

i=1

|ji |
n

|i | |j |

1.

1 y |i |/|j | = 1 para i I . Entonces |ji | +


iI

|i | |ji | < |j | |ji | = 0.

i=1

|ji |

1.

Esto es una contradicci on en el caso que


iI

Concluimos que para j I , tenemos que


iI

|ji | = 0,

o sea, usando que ji = ji /jj ,


iI

|ji | = 0 para j I ,

lo que implica que A es reducible, una contradicci on. Uno podr a pensar que seg un nuestra construcci on, el m etodo de Gauss-Seidel siempre converge mejor que el de Jacobi. Pero eso no es v alido en general. Teorema 4.5 (Stein-Rosenberg). Sea A Rnn , ij 0 para i = j y ii = 0 para 1 1 i = 1, . . . , n, J := D (L + U) y H := (L + D) U. En este caso, exactamente una de las siguientes alternativas es v alida: 1. r (H) = r (J) = 0, 2. r (H) = r (J) = 1, 3. 0 < r (H) < r (J) < 1, 4. r (H) > r (J) > 1. Demostraci on. Ver Varga, Matrix Iterative Analysis.

4.3. TEOREMAS DE CONVERGENCIA PARA METODOS ITERATIVOS

83

El Teorema 4.5 dice que para matrices del tipo indicado, el m etodo de Gauss-Seidel converge mejor si alguno de los m etodos converge. Eso implica en particular que ambos m etodos convergen para el sistema del Ejemplo 4.1. En la pr actica, las siguientes matrices son importantes: matrices sim etricas denidas positivas y M-matrices. Denici on 4.4. Una matriz A Rnn se llama M-matriz si ij existe y A1 0. 0 para i = j y A1

Teorema 4.6. Una matriz A Rnn es una M-matriz si y s olo si ii > 0 para i = 1, . . . , n, 1 ij 0 para i = j y r (D (L + U)) < 1, donde A = L + D U es la descomposici on (4.12), (4.13) de A. Demostraci on. Hay que demostrar dos implicaciones. : Sea ii > 0 para i = 1, . . . , n, ij 0 para i = j y r (D1 (L + U)) < 1. Denimos 1 J := D (L + U). Entonces r (J) < 1, y la matriz (I J) existe, y (I J) es decir, lo que implica que A1 existe y A1 0. : Supongamos que A es una M-matriz. Sea ii 0 para alg un i. En este caso, usando que ij 0 para i = j , tenemos Aei 0, es decir, multiplicando con la matriz A1 0, A1 Aei 0, o sea ei 0, una contradicci on. Entonces, ii > 0 para todo i, lo que implica que J := D1 (L + U) es bien denida, J 0 y A1 D = (I J)1 existe. Sea un valor propio de J con vector propio x = 0. En este caso, y dado que (I J)1 |||x| 0, |x| 1 || (I J)1 |x|. 0. Entonces, podemos concluir J|x| = (I J)|x| 1 || |x|, (I J)1 = A1 D,
1 1

Jk

k=0

0 dado que J

0. Pero por otro lado, sabemos que

I J = D1 A,

Dado que |x| = 0, (I J)1 |x| = 0 y (I J)1 |x| que || < 1, o sea r (J) < 1.

Entonces el m etodo de Jacobi converge para cada M-matriz, y usando el Teorema 4.5, podemos concluir que converge tambi en el m etodo de Gauss-Seidel. Denici on 4.5. Sea A Rnn . Llamamos a A = N P una partici on regular de A si nn 1 NR es regular y N 0, P 0.

84

DE SISTEMAS DE ECUACIONES LINEALES 4. METODOS ITERATIVOS PARA LA SOLUCION

Teorema 4.7. Sea A Rnn , con la partici on regular A = NP. En este caso, r (N1 P) < 1 1 si y s olo si A 0. Demostraci on. : Trivialmente tenemos que H := N1 P 0. Ahora, si r (H) < 1, entonces r (H) < 1, o sea (I H)1 existe y (I H)1 0, por lo tanto A1 = (I H)1 N1 0. : Sea A1 0. Sabemos que A1 = (N P)1 = (I N1 P)1 N1 . 0, (I + H + H2 + . . . + Hm )N1 = (I Hm+1 )(I H)1 N1 = (I Hm+1 )A1 A1 , lo que implica que I + H + H2 + . . . + Hm converge cuando m , por lo tanto, r (H) < 1. Teorema 4.8. Sea A estrictamente o irreduciblemente diagonal dominante con ii > 0 para i = 1, . . . , n y ij 0 para i = j (es decir, A es una L-matriz). En este caso, A es una M-matriz. Demostraci on. En este caso, N = D y P = L + U es una partici on regular. Segun el 1 Teorema 4.4, r (D (L + U)) < 1. Luego aplicamos el Teorema 4.7. Ejemplo 4.4 (Certamen 10 2 6 3 5 1 A1 = 2 2 12 1 0 2 1, Curso 2010). Se consideran las matrices 2 0 1 0 4 1 1 0 0 1 1 . , A2 = 1 3 1 1 , A3 = 0 2 0 3 0 5 0 2 4 2 0 4 0 2 0 4 3 1 1 2 4 0

Ahora, con H := N1 P

utilizando los m etodos de Jacobi y de Gauss-Seidel. Soluci on sugerida.

a) Demostrar que para cada una de ellas que el m etodo de Jacobi converge a la soluci on 4 4 de Ai xi = bi para bi R y vectores iniciales xi,0 R arbitrarios. b) Utilizando el vector inicial xi,0 = (1, 1, 1, 1)T , calcular para i = 2 e i = 3 una nueva aproximaci on de la soluci on de Ai xi = bi para 9 2 8 1 b2 = 8 , b3 = 24 , 13 6

4.3. TEOREMAS DE CONVERGENCIA PARA METODOS ITERATIVOS

85

a) Se demuestra facilmente que A1 y A2 son irreduciblemente diagonal dominantes. Por otro lado, la matriz A3 corresponde a dos sistemas lineales desacoplados para (x1 , x3 ) y (x2 , x4 ), respectivamente, con las respectivas matrices A3,1 = 2 1 3 5 y A 3 ,2 = 2 1 , 3 4

ambas de las cuales son estrictamente diagonaldominantes; por lo tanto se puede concluir ambos m etodos convergen tambi en en el caso de A3 . b) Utilizando el m etodo de Jacobi para Ax2 = b2 se genera la sucesi on de vectores 1,7500 1,1667 1,2188 1,0495 2,3333 , x2,2 = 1,6667 , x2,3 = 2,0208 , x2,4 = 1,9306 x2,1 = 2,0000 2,4583 2,7812 2,8750 3,2500 3,2292 3,7708 3,8307 etc., mientras que el m etodo de Gauss-Seidel entrega 1,0358 1,1610 1,3646 1,7500 2,0833 , x2,2 = 1,6910 , x2,3 = 1,8951 , x2,4 = 1,9864 x2,1 = 2,9902 2,9617 2,6649 1,4583 3,9896 3,9668 3,8186 3,0208

etc. Para el sistema Ax3 = b3 obtenemos las respectivas sucesiones 1,91 1,25 1,7 0,5 1 , x3,2 = 0 , x3,3 = 0,5 , x3,4 = 0,25 x3,1 = 5,55 5,82 4,5 5,4 1,25 1,5 2 1

etc. para el m etodo de Jacobi y 1,9325 1,775 1,25 0,5 1 , x3,2 = 0,5 , x3,3 = 0,125 , x3,4 = 0,2188 x3,1 = 5,9595 5,865 4,5 5,55 1,3906 1,4375 2 1,25 etc. para el m etodo de Gauss-Seidel.

Soluci on sugerida. a) La matriz A es irreducible, pero no posee ninguna de las otras propiedades.

Ejemplo 4.5 (Tarea 19, Curso 2006). Analizar si las siguientes matrices poseen algunas de las siguientes propiedades: irreducible, irreduciblemente diagonal dominante, estrictamente diagonal dominante, L-matriz, M-matriz: 0 1 0 0 2 1 0 0 2 2 0 0 0 0 1 0 1 2 1 0 1 2 1 0 A= 0 0 0 1 , B = 0 1 2 1 , C = 0 1 2 1 . 1 0 0 0 0 0 1 2 0 0 2 2

86

DE SISTEMAS DE ECUACIONES LINEALES 4. METODOS ITERATIVOS PARA LA SOLUCION

b) La matriz B igualmente es irreducible. Dado que es diagonal dominante y estrictamente diagonal dominante en por lo menos una la, la estructura de signos implica que B es una L-matriz irreduciblemente diagonal dominante. Esto es una condici on suciente para asegurar que es una M-matriz. c) La matriz C es una L-matriz irreducible, pero no es una L-matriz irreduciblemente diagonal dominante. Como C es singular, no puede ser M-matriz. Ya sabemos que el m etodo de Jacobi converge si A es una M-matriz. Incluso, tenemos el siguiente teorema. Teorema 4.9. Si A es una M-matriz, entonces el m etodo SOR converge para 0 < 1.

Demostraci on. La matriz A puede ser escrita como 1 A = D L (1 )D U (4.26) 1 1 =N P, N := (D L), P := (1 )D + U . Demostramos ahora que (4.26) es una partici on regular. Para tal efecto, demostramos que (D L)1 0; el resto es una consecuencia del Teorema 4.8. Pero, en virtud de D1 L 0, podemos escribir:
n

(D L)

= (I D L) D

= l m

( D1 L)k
k=0

D1

0.

El Teorema 4.9 no es muy interesante para las aplicaciones por que se puede mostrar que para una M-matriz, la funci on r B( ) = r (D L)1 (1 )D + U es estrictamente decreciente para 0 < con > 1. Lo que es interesante es el problema de la convergencia del m etodo SOR para > 1, y el problema de existencia de un posible par ametro o ptimo, opt . En lo siguiente, siempre usamos A = L + D U, B( ) := (D L)1 (1 )D + U . r B( ) Demostraci on. Usando que podemos escribir (D L)1 = (I D1 L)1 D1 , | 1|. (4.27)

Teorema 4.10. La matriz B( ) satisface

det B( ) I = det (D L)1 (1 )D + U (D L) = det (1 )I + D1 U + D1 L .

4.3. TEOREMAS DE CONVERGENCIA PARA METODOS ITERATIVOS

87

Evaluando esta f ormula para = 0, obtenemos


n

det B( ) =
i=1

i B( ) = (1 )n . | 1|.

Esto implica que r B( ) = m ax i B( )


1 i n

En consecuencia, para R nos interesa solamente el intervalo 0 < < 2. Ya sabemos que para M-matrices, r (B( )) es una funci on decreciente en el intervalo 0 < con 1. Para matrices denidas positivas tenemos el siguiente teorema. Teorema 4.11. Sea A Rnn sim etrica y denida positiva. Entonces r (B( )) < 1 para 0 < < 2. Demostraci on. Denimos la funci on 1 f (x) := xT Ax bT x, 2 notando que 1 1 1 f (x) = bT A1 b + (x A1 b)T (x A1 b) bT A1 b. 2 2 2 2 La denci on de f implica f (x) = Ax b y f (x) = A, entonces x := A1 b es el m nimo unicamente denido de f . Adem as, usando la notaci on de la descripici on del m etodo (4.18), (4.19), es decir, escribiendo 1,k+1 . . . y0 := x0 , ykn+i := i,k+1 , 1 i n, k N0 , i+1,k . . . n,k podemos denir rj := Ayj b, j N0 ;
j,i

:= eT i rj .

Entonces, usando (4.19), podemos escribir 0 . . . 0 kn+i1,i T ykn+i ykn+i1 = i,k+1 i,k = ei rkn+i1 ei = ei ii ii 0 . . . 0

88

DE SISTEMAS DE ECUACIONES LINEALES 4. METODOS ITERATIVOS PARA LA SOLUCION

y xk = ykn para k N0 . Una computaci on directa entrega f (ykn+i ) f (ykn+i1 ) =

(2 ) 2ii

2 kn+i1,i .

(4.28)

Entonces, para 0 < < 2 la sucesi on {f (yj )}j N es monotonamente no creciente y acotada hacia abajo. Por lo tanto, existe
k

l m

kn+i1,i

= 0 para i = 1, . . . , n,

(4.29)

y entonces tambi en
k

l m (ykn+i ykn+i1 ) = 0, i = 1, . . . , n. (4.30)

y dado que rj +1 rj = A(yj +1 yj ), existe tambi en el l mite


k

l m (rkn+i rkn+i1 ) = 0, l m rkn = 0.

Todav a hay que demostrar que


k

(4.31)

Ahora, debido a (4.29) y (4.30), | Entonces |


kn,1 | j +1,i

kn+i1,i |

j,i |

para i = 1, . . . , n y j para k

j0 = nk0 ,

k0 () e i = 1, . . . , n.

. Pero | | |
kn+1,2

kn+2,3

kn+1,3

kn+1,3 | kn,3 |

kn,2 |

| | |
k

kn+1,2 | kn+1,3 | kn+2,3 |

= | 2 = | = |

kn,2 | kn,3 |

2, 2, 3,

kn+1,2 |

y nalmente |

kn,n |

n, es decir, rkn

n, lo que implica (4.31), o sea

xk x = A1 b El resultado sigue con el Teorema 4.1. El Teorema 4.11 entrega una interpretaci on interesante del m etodo SOR como m etodo de minimizaci on de la funci on f , cuyo gradiente es el residuo Ax = b. Las supercies f (x) = c en Rn son elipsoides conc entricos, y mediante el m etodo SOR se alcanza el centro n com un x (el u nico m nimo de f en R ) a lo largo de las direcciones de coordenadas con descenso mon otono de f . Para una clase especial de matrices existe un resultado cuantitativo acerca de la dependencia de r (B( )) de . Denici on 4.6. Sea U ), A = D(I L := D1 L, L := D1 U, U

4.3. TEOREMAS DE CONVERGENCIA PARA METODOS ITERATIVOS

89

partiendo de la partici on usual de A en una matriz diagonal D y en matrices L y U estrictamente triangulares. La matriz A se llama ordenada consistentemente si para cada , = 0, L + 1U . + 1U L Teorema 4.12. Supongamos que D1 A12 A21 D2 0 A32 . .. A= . . . . . . . . . 0 Demostraci on. Tarea. La matriz del Ejemplo 4.1 no posee la forma requerida en el Teorema 4.12. Pero si cambiamos la enumeraci on a (N, 1), (N, 2), (N 1, 1), (N, 3), (N 1, 2), . . . , la matriz de coecientes si asume esta forma. En este caso, se puede demostrar que la matriz es ordenada consistentemente (Tarea). Teorema 4.13. Sea A ordenada consistentemente. Entonces sabemos que para J := D1 (L+ U), a) (J) (J), b) (B( )) (J) : 2 2 = ( + 1)2 . Demostraci on. a) Sea 1 1 D U. Entonces J(1) = J(1), mientras que J(1) y J(1) tienen los mismos valores propios seg un hip otesis. b) : Para = 0, sabemos que J() := D1 L + det B( ) I = det D1 U + D1 L + (1 + )I 1 = det D1 U + D1 L + (1 + )I = det J( ) + (1 + )I la matriz A CN N puede ser particionada como 0 0 . . A23 0 . . ... . D3 A34 . . .. .. .. .. . , . . . . . .. .. .. .. . . . . 0 ... An1,n2 Dn1 An1,n 0 An,n1 Dn

donde D1 , . . . , Dn son matrices diagonales regulares. Entonces A es ordenada consistentemente.

+1 = ( )n det J( ) I .

90

DE SISTEMAS DE ECUACIONES LINEALES 4. METODOS ITERATIVOS PARA LA SOLUCION

Entonces, 0 = B( ) +1 +1 J( ) J(1) .

Pero si 0 es un valor propio de B( ), (0 (B( ))), entonces para este caso, b) es trivial. : Sea un valor propio de J y 2 2 = ( + 1)2 . En el caso = 0, eso signica +1 ; = usando la discusi on de (a), podemos elegir +1 , = det B( ) = (1 )n = 0 = = 1;

es decir, (J(1)) y tambi en (J( )), por lo tanto, (B( )). Si = 0, tenemos = 1, pero det(B( )) = 0, o sea, 0 (B(1)).

Podemos concluir que si A es ordenada consistentemente, entonces es decir, esencialmente el m etodo de Gauss-Seidel converge dos veces m as r apido que el 2 m etodo de Jacobi: notamos que para = 1 en b), = . Teorema 4.14. Sea A ordenada consistentemente. Supongamos que los valores propios i de J := D1 (L + U) satisfacen i (1, 1) para i = 1, . . . , n. Entonces para 2 opt := , := r (J), (4.32) 1 + 1 2 el radio espectral de B( ) es dado por 2 1 2 2 + 4( 1) para 0 opt , r B( ) = (4.33) 2 2 1 para opt 2. Demostraci on. La soluci on de la ecuaci on cuadr atica 2 2 = ( + 1)2 del Teorema 4.13 entrega para i = 1, 2 1 2 2 2 i := 2 2 4( 1)2 i 2( 1) i 2( 1) 2 1 2 2 2 4 2 4 = i 2( 1) i 4( 1)i 2 1 1 2 2 2 = 2 i + i + 1 4 4
2 2 i i + (1 ) 2 4 2 2 r (D L)1 U = r D1 (L + U) ,

La funci on r (B( )) es estrictamente decreciente sobre el intervalo [0, opt ].

4.3. TEOREMAS DE CONVERGENCIA PARA METODOS ITERATIVOS

91

1 4

2 2 i + 4(1 )

Aqui el radicando es no negativo siempre que 0 1+ 2 1 2 i =: i . i = 1, 2.

Para i tenemos |i | = 1, lo cual se deriva usando que para z C, |z |2 = 2 1 y es monotonamente (Re z ) + (Im z )2 . Dado que 0 2 i < 1, tenemos que i creciente con i , lo que entrega la segunda parte de (4.33). Para 0 i el valor mayor de valor absoluto de i resulta de 1 |i | = 4
2

|i | +

2 2 i

+ 4(1 )

1,

y este valor crece mon otonamente con i , lo que demuestra la primera parte de (4.33). Diferenciando con respecto a obtenemos el u ltimo enunciado. El Teorema 4.14 parte de la hip otesis que los valores propios de J son, en particular, reales. El siguiente lema informa que esto efectivamente es v alido para una matriz A sim etrica y denida positiva. Lema 4.1. Para una matriz A sim etrica y denida positiva, los valores propios de J = I D1 A son reales. Demostraci on. Puesto que la diagonal D de A es positiva, existe una matriz diagonal F con D = F2 , e I D1 A = F1 (I F1 AF1 )F = F1 MF. Dado que la matriz M = I F1 AF1 es sim etrica y por lo tanto solo posee valores propios reales, tambien la transformada ID1 A solo posee valores propios reales. Ejemplo 4.6. Supongamos que = r (J) = r D1 (L + U) = 0,9. En este caso, de (4.32) obtenemos opt = 2 1+ 1 0,92 1,39286;

la Figura 4.3 muestra la funci on r (B( )), dada por (4.33), que resulta en este caso.

92

DE SISTEMAS DE ECUACIONES LINEALES 4. METODOS ITERATIVOS PARA LA SOLUCION

1 ppppppppppppppppppppppppppppppppppppppppppppppppppppppppppppp pppppppppppppppppppppp pppppppppppppppppp r (B( )) ppppppppppppppp pppppppppppp pppppppppp opt 1,39286 ppppppppp 0,8 pp 0,6 0,4 0,2 0 0 0,5 1 1,5

p pp pp ppp p p p p p pp pp ppp p p p p pppp pp pp ppppppp p p p p pppppp pppp ppppp pp pp pppp p p p p pppp p pp pp ppp pppp ppp p p p ppp pppppp ppp p ppp ppp ppppp ppp

Figura 4.3. El radio espectral r (B( )) dado por (4.33) para = 0,9 (Ejemplo 4.6). Ejemplo 4.7 (Tarea 20 b), Curso 2006). Demostrar que la siguiente matriz es ordenada consistentemente, y determinar (si posible) el parametro optimo para el m etodo SOR. 2 1 1 0 1 4 0 1 A= 1 0 4 1 0 1 1 2 Soluci on sugerida. Descomponiendo A en bloques de los tama nos 1, 2 y 1, 2 1 1 0 1 4 0 1 , A= 1 0 4 1 0 1 1 2

nos jamos que A es una matriz tridiagonal por bloques con bloques diagonales diagonales y regulares. La simetr a y el Lema 4.1 implican que todos los valores propios de la matriz 1 J = I D A son reales. Dado que A es irreduciblemente diagonal dominante, el m etodo de Jacobi converge, tal que los valores propios pertenecen a [1, 1]. Entonces existe opt . Finalmente, tenemos que 0 1/2 1/2 0 1/4 0 0 1/4 = p() = det(J I) = 2 (2 1/2), J= 1/4 0 0 1/4 0 1/2 1/2 0

4.3. TEOREMAS DE CONVERGENCIA PARA METODOS ITERATIVOS

93

entonces opt = 1+ 2 1
1 2

1,171573.

Ejemplo 4.8 (Tarea 21, Curso 2006). Sea 1 0 c 1 d , A= 0 a b 1

a, b, c, d R.

a) Para qu e valores de z = ac + bd el m etodo de Jacobi converge para la soluci on del sistema Ax = r? b) Para qu e valores de a, b, c, d la matriz A es irreducible? c) Indicar la f ormula de iteraci on del m etodo SOR para Ax = r en forma expl cita (o sea, sin matrices inversas) en la forma xk+1 = B( )xk + v( ). d) Sean a = 0,5, b = 0,4, c = 0,7, d = 0,4 y r = (5, 9, 7)T . (La soluci on exacta es = (2, 5, 10)T .) Partiendo de x0 = (1, 1, 1)T , calcular x2 con el m x etodo de GaussSeidel. e) Sean H1 := B(1) y H la matriz de iteraci on del m etodo de Jacobi. Demostrar que A 2 es ordenada consistentemente y que (r (H)) = r (H1 ). Determinar r (H1 ) para los valores num ericos de (d). f) Sea 0 < z < 1. Demostrar que el m etodo SOR aplicado a A posee un par ametro optimo = opt , y calcular el valor de opt para los valores num ericos de (d). Cu al es el valor del radio espectral correspondiente? 2 usando el m g) Partiendo de x0 especicado en (d), determinar x etodo SOR y el par ametro optimo. a) En este caso, 0 0 c J = 0 0 d = det(J I) = (2 + z ), a b 0

Soluci on sugerida.

entonces r (J) < 1 si y s olo si |z | < 1. b) P1 puede ser conectado con P3 si y s olo si c = 0, y P2 con P3 si y s olo si d = 0. Lo mismo es v alido para los arcos dirigidos P3 P1 y p3 P2 , o sea A es irreducible si y s olo si a = 0, b = 0, c = 0, d = 0. c) B( ) = (D L)1 ((1 )D + U) 1 1 0 0 1 0 c 1 0 0 1 d = 0 a b 1 0 0 1

94

DE SISTEMAS DE ECUACIONES LINEALES 4. METODOS ITERATIVOS PARA LA SOLUCION

d) xk+1

1 0 0 1 0 c 1 d = 0 1 0 0 a b 1 0 0 1 1 0 c , 0 1 d = 2 (1 )a (1 )b 1 + z 1 0 0 v( ) = 0 1 0 r. a b 1

e) Puesto que

0 0 0 ,7 5 4,3 = 0 0 0,4 xk + 9 = x1 = 8,6 , 0 0 0,19 8,1 8,29 det L + 1 U I = (2 + z )

0,803 x2 = 5,684 9,6751

g)

independentemente consistentemente. Usando (a), vede , la matriz A es ordenada 2 mos que r (H) = 0,19; entonces (r (H)) = r (H1 ), es decir r (H1 ) = 0,19. f) Seg un (a), r (H) < 1 y todos los valores propios de H son reales. Seg un (e), A es ordenada consistentemente, entonces existe opt . Aqu tenemos 2 20 opt = = = 1,0526, r (Hopt ) = 0,0526. 19 1 + 1 (r (H))2 4,5789 , 9 x1 = 8,795 1,4583 x2 = 5,2968 . 10,0485

Se reconoce la gran ganancia en velocidad de convergencia usando = opt . Pero en la pr actica, se recomienda sobreestimar , dado que en el lado izquierdo de opt la tangente es vertical (como ilustra la Figura 4.3). Entonces, necesitamos una cota superior de r (D1 (L + U)). Veremos en el Cap tulo 5 como nos podemos conseguir una tal cota. Sin demostraci on comunicamos el siguiente teorema. Teorema 4.15. Sea A una matriz tridiagonal por bloques con bloques diagonales I, sim etrica, y denida positiva. Entonces para := cond tenemos la inclusi on 1 +1 r B(opt ) . + (4.34)

( A)

1/2

:=

4 2+

POR BLOQUE 4.4. METODOS DE ITERACION

95

Este resultado signica que para una matriz mal condicionada, el m etodo SOR con opt a un es muy lento, pero mucho m as r apido que los m etodos de Gauss-Seidel o Jacobi, dado que bajo las hip otesis del Teorema 4.15 tenemos que
2 2 r D1 (L + U) = r (L + U) = r (D L)1 U ,

donde r D1 (L + U) = 1 m n (A) = m ax (A) 1 = 1 con 1 m ax (A) 2. 0,9998 para el m etodo de m ax (A) cond 2 (A)

Ejemplo 4.9. Si cond 2 (A) = 10000, tenemos = 100 y r Jacobi y r 0,9996 para el m etodo de Gauss-Seidel, pero 2 = 0,980198 101 para el m etodo SOR con opt . Notar que depu es de 1000 pasos, r B(opt ) 1 0,99981000 = 0,8187, 0,99961000 = 0,67026,

0,9801981000 = 2,059 109 .

Una pregunta obvia es c omo se puede estimar el radio espectral de la matriz de iteraci on con poco esfuerzo computacional. A parte de considerar vectores iniciales especiales, podemos considerar la expresi on Axk b 1/k . Teorema 4.16. Sea A regular y el sistema Ax = b equivalente a x = Gx + g, y la sucesi on {xk }kN0 denida por xk+1 = Gxk + g, Entonces, para cualquier norma , tenemos que
1/k

k N0 . r (G),

(4.35)

l m sup Axk b
k

(4.36)

y existe un vector x0 para el cual (4.36) vale con =. Demostraci on. Tarea. 4.4. M etodos de iteraci on por bloque Se ofrece la siguiente generalizaci on de los m etodos de iteraci on discutidos hasta ahora. Se particiona la matriz A, el vector de soluci on x y la parte derecha b en bloques y subvectores, respectivamente: A11 A12 A1n x1 b1 A21 A22 A2n . . , , x = . A= . . .. . , b = . . . . . . . . . . xn bn An1 Ann

96

DE SISTEMAS DE ECUACIONES LINEALES 4. METODOS ITERATIVOS PARA LA SOLUCION

y en la derivaci on de los m etodos ponemos 0 A11 A21 0 .. , L = D= . . .. .. . . . . Ann An1 An,n1


i 1 j =1

0 0 A12 A1n . . .. .. . . . . . . . . . , U = . . . .. . . . An1,n . . 0 0 0 (4.37)


n

Por ejemplo, la iteraci on del m etodo Bloque-SOR es denida por Aii xi,k+1 = bi i = 1, . . . , n, Aij xj,k+1 Aij xj,k
j =i

+ Aii xi,k ,

(4.38)

Este procedimiento requiere en cada paso la soluci on de n sistemas lineales peque nos. Sin embargo, tal procedimiento puede ser ventajoso cuando, por ejemplo, las matrices Aii son simplemente estructuradas (por ejemplo, tridiagonales). Los Teoremas 4.104.14 pueden ser generalizados facilmente. A modo de ejemplo, tenemos el siguiente teorema. Teorema 4.17. Sea A una matriz tridiagonal por bloques y denida positiva. Entonces, el m etodo (4.38) converge para 0 < < 2. El par ametro optimo opt es dado por 2 , = r D1 (L + U) , opt = 2 1+ 1 donde D, L y U estan dadas por (4.37), y la funci on r (D L)1 (1 )D + U tiene las mismas propiedades que las especicadas en el Teorema 4.14. 4.5. El m etodo de gradientes conjugados (cg) de Hestenes y Stiefel Ya mencionamos en la demostraci on del Teorema 4.11 que el m etodo SOR para la soluci on de Ax = b, donde A es sim etrica y denida positiva, puede ser interpretado como un m etodo de minimizaci on para la funci on 1 f (x) = xT Ax bT x (4.39) 2 con el gradiente donde se procede en orden c clico a lo largo de las direcciones de coordenadas. El caso n = 2 ya ilustra que los ejes de las coordenadas no necesariamente deben ser las m as ventajosas. En el caso n = 2, las curvas f (x) = c son elipses conc entricas. Una minimizaci on de f a lo largo de los ejes principales de la elipse entregar a el m nimo de f , o sea, la soluci on de Ax = b, en dos pasos. El resultado an alogo tambi en es v alido para n 3. Los ejes principales forman un caso especial la las llamadas direcciones A-ortogonales, que en este caso tambi en son ortogonales. f (x) = Ax b, (4.40)

k N0 .

4.5. EL METODO DE GRADIENTES CONJUGADOS (CG) DE HESTENES Y STIEFEL

97

Denici on 4.7. Sea A Rnn sim etrica y denida positiva. Un sistema {p0 , p1 , . . . , pn1 } de vectores se llama A-ortogonal si para 0 j, k n 1, pT j Apk = k jk , j > 0, jk = 0 1 si j = k , si j = k .

Pero, para la soluci on del sistema Ax = b, la determinaci on de los ejes principales de A, es decir, de sus vectores propios, signicaria un esfuerzo computacional altamente exagerado. Demostraremos ahora que la minimizaci on a lo largo de direcciones A-ortogonales, tambien llamadas A-conjugadas, tambi en entrega un m etodo nito. Ejemplo 4.10. Consideramos el sistema Ax = b con A= 2 1 , 1 3 b= 3 , 1 (4.41)

con la soluci on exacta x = (2, 1)T ; la matriz A es sim etrica y denida positiva. Un ejemplo de direcciones A ortogonales son p0 = 1 , 0 1 p1 = 5 1 . 2

La Figura 4.4 muestra las elipses conc entricas f (x) = c, donde f es denida por (4.39) y c = 3, 2, 1, 0, . . . ; el m nimo es f (x ) = 3,5. Teorema 4.18. Sea la funci on f denida por (4.39), con A Rnn sim etrica y denida positiva. Sea {p0 , p1 , . . . , pn1 } un sistema de direcciones A-ortogonales. Sea x0 arbitrario, y xk+1 = xk k pk , pT (Axk b) k := k T , pk Apk k = 0, . . . , n 1. (4.42)

Entonces (i) xk+1 minimiza f (xk pk ) con respecto a , (ii) (Axk b)T pj = 0 para j = 0, . . . , k 1, es decir, xk = argmin f (x) | x = x0 + span{p0 , . . . , pk1 } , (iii) xn = x = A1 b. Demostraci on. (i) Utilizando el c alculo diferencial, obtenemos d f (xk pk ) = 0 d T f (xk pk ) pk = 0 = pT k (Axk b) . pT k Apk (4.43)

pT k A(xk pk ) b = 0

98

ppppppppppppp pppppp p ppppp 3 p p p p p p p p p p p p pp p pp ppppp pp pp pp pp p p p p p p p p p p p pp pp pp ppppp p p p p p p pp p p p p p pp pp pppp ppppppp ppp pp p ppp pp ppppppppppppp pppppppppp ppppp pppp pp p p p p p p p p p p p p p p p pppppp p p p p p p p p p p p p p p p p p p p p p p p p p p p p p p p p p p p p p p p p p p p p p p p p p p p p p p p p p p p p p p p p p p p p p p p p p p p p p p p p p p p p p p p p p p p p p p p p p p p p p p pppppppppppppppppppppp pp p pp pp pp ppp ppp pp pp p p p pp pppppppppppppppppp p p p p p p p p p p p p p p p p p p p p p p p p p p p p p p p p p p p p p p p p p p p p p p p p pp pppppp ppp pp p p p p p p p p p p p p p pp pp pp p pppp pp p pp pp p p p p pp pp pp p ppppp p p ppp p p p p p p p p p p p p p p p p p p p p p p p p p p p p p p p p p p p p p p pppppppppp f= 0 p pp p ppp ppppp pp p p p p p p p pp p pp p pppp p p p pp p pp ppp pp p p ppp ppppppp pppppp p pp p p pp p p pp pppp p pp pp p pp pppppp p pp ppp p p p p p p p p pp p ppppppppppp

DE SISTEMAS DE ECUACIONES LINEALES 4. METODOS ITERATIVOS PARA LA SOLUCION

ppppppp p pp p p p p p p p p p p ppp ppppp p ppp pppp pp p p pppp pp pppppppppp p ppppppppppppppppppppppppppp pppppppppppppppppppppppppppppppppppppppppppppppppppp p pp pppp p pp p ppppp pppppp p p pp pp p ppp pppppp pppppppppppppp p pppp p pppp p pppppp p p p p p p p p p p p p ppppp p pp p pp p p p p p pp pp p pp pp p p pp p p p p p p p p p p p p p p p p p p p p p p p p p p p p p p p p p p p p p p p p p p p p p p p p p p p p p p p pp pp pppp pp p p p p p p p p p p p p p p p p p p p p p p p p p p p p p p p p p p p p p p p p p p p p p p p p p p p p p p p p p p p f = 1 p p p p p p ppppppp p p p pppp p ppppp p ppppp p pp pp p pppp p pp ppp pppp p p pp p p p p pppppp pppppppppppppppppppppppppppppppppppppppppppppppppppppppppppppp pp ppp ppp pppp ppp ppp pppp ppp p p p p p p p p p p pppppp p pp p p p ppppppppppppppppppppppppppp ppppppppppppp ppp p pp p p pp p p p p p p p p p ppppp p p p p p p p p p p p p p p p p p p p p p p p p p p p p p p p p p p p p p p p p p p p p pppp p p p p p p p p p p p p p p p p p p p p p p p p p p p p p p p p p p p p p p p p p p p p p p p p p p p p p p p p p p pppp p p p p p p p p p p p p p p ppppppp pp p pp pp p pp p pppppp p p p pppp pppp pppp pppp pppp p p p p p p ppp p p p p p p p p p p p p p p p p p p p f = 2 p p p p p p p p p p p p p p p p p p p p ppp p p p p p p p p p p p ppppp p pp ppp pppppppppppppppppppppppppppppppppppppppppppppp p p pp p p p p pp pp pp p ppp pppp p p p p p p p p p p p p p p p p ppp p p p p p p p p p p p p p p p p p p p p p p p p p p p p p p p p p p p p p p p p p p p p p p p p p p p p p p p p p p p p p p p p p p p p p p p p p p p p p p ppp p ppppppppp ppp pp pp ppppppp pp p pppp ppp p pp p pppp p p p p p p p p p p p p p p p p p p p p p p p p p p p p p p p p p p p p p p p p p p p p p p p p p p p p p p p p p p p p p p ppp ppppp p p pp p ppp p ppp p ppppp p ppp p pp pp pp pp pppp ppp pp p p p ppp p ppp pp p p p ppp pp ppp ppp pp p p pp p p p p p p p ppp p p p p p p p p p p p p p p p p p p p p p p p p p p p p p pp ppp pp p pp pp f = 3 pp p p p p p p ppp p p p p p p p p p p p p p ppp pp p pppppppppppppppppppppppppppppppppppppppppppppppppppp pp p ppp p ppp p p p p p p p p p p p p p p p p p p p p p p p p p p p p p p p p p p p p p p p p pp p p p p p p p p p p p pppp pp pp pp p pp pp ppp pp ppppp p p p p p p p p p p p p p p p p p p ppp ppp ppp p pp pp p p p p p pp p pp pp p p ppp p pp p p pp p pp p pp p pp p ppp pp p p p p p p p p p p p p p p p p pp p p pp p pp pp pp pp p p p p p p p p p pp p p p p p p p pp p p pp p pp p pp p pp p pp pp p pp ppp pp ppp pp p pp p ppp p p p p p p p p p p p p p p pp pp p p pp pp pp pp 1 ppp pp ppp pp pp x = A b ppp pp ppp p p p p p p p p p p  p p pp pp pp pp pp pp ppp p pp p pp p pp p pp p ppp p p p ppp p pp pppp p p p p p p p p p p p p pp p p p ppp pp pp pp pp p pp p pp p pp p pp p p p p p p p ppp p pp pppp p p ppp p p p p p pp pp pp ppp pp p p p p ppp p p pp pp p pp p pp p p pp pppp pp p p p p p p p p p p p p p p pp p p p p p p pppp pppppp pp pp pp pp p pp ppp pppppppppp p p pp p ppppp p ppp p ppp ppp pp p pp p pp p ppp pppppppppppppppppppp p pp p p p pp pp pppppppp ppp p p p p p p p p p p p p p p p p p p p p p ppp p p p p p pp p p p p p p p ppp p p p p p p p p p p ppp p p p ppp pp pp p pp ppp pp p p p p pppp p p p p p pp ppp p pp p p p p ppppp p p p p p p p p p ppp p p p p p p p p p p p p ppppp p ppp pp  pp p p pp p pp p p p ppp p p pppppp p ppp pppp p ppp ppp p p pp p p pp ppp p p p p p ppp ppp pp pp pp p pppppppp p p p p p p p p p p p p p p p p p p p p p p p p p p p p p p p p ppppppppppp pp ppp pppp p pppp p p pp p p ppp pp p p ppppppp pppppppppppppppppppppp p pp pp pp ppp pp p pppppp ppp ppppp pp ppppppp p p ppp pppppppppppp pp pppp p p pp p p pp p ppp p p p p p p p p pppp p pppppppppppppppppppppppppppppppp pppp p p p p p p p p ppppp p p p p p ppp ppp pppppp ppppp p p p p p p p p p p p p p pp pppppp p pp p pp p pp p p p p p ppp p ppppp pp pp pp p pp ppppp p p p ppppppp pppp ppp ppppppp p p p p p pp ppp p p p p p p p p p p p p p pp p p p p pp ppppp pp pp ppp p ppppppp p pp p p p p p p ppppppppp p p p p p p p p p p p p p p p p p p p p p p p ppppp 1 ppppppppppppp p pp pppppp pp p p pp pp pp ppp ppp ppp pp p pp p p ppppp pp pp p ppp p p ppppppppppppppppppppppp p p pppppppppp ppp pp pp p p p p pp p pppppp p p p pp p pp pp p p p p p p p p p p pp ppp pp pp p p p p p p p p p p p p p ppppppp p p p p pppppppppppppppppppp p p p p p ppppppppppppppppppppp ppppppp ppppppppp p pp p p p p p p p p p p p p ppppppp pp pp p p ppppppppp ppp pp p p p pppp p p pppp p p ppp pp ppp p pp p p p p p pp pp ppp ppppppppppp ppp pppp p p pp p p p p p p p pp pppp pp pp p p p p p p p p p p p p p pppp pp p p p p p p p pp ppp pp pp pp pppp pp p pppp p p p p pppppp p p p p p p p p p p p p p p p p p p p p p p p p p p p p p p p p p p ppppppppppppppppp pppp p ppp pp pp p p pp ppp pp pppppppppp pppppp pppppppppppppppppppppppppppp- pppppp pp pp pp pppppp p p p p p p p p p pp p pp pp ppppp pp p p p p p p p p p p p p p p p p p p p pppp ppp pp p pp p p p p p p p ppppppppp s ppppppppppppppp s pppppppppppppppppppp pppppppppp ppppppppppp pppp p p p p p p p p p p p p p p p p p p p p p pp x0 ppppppppppppppppppp p0 x1 pp ppp pp p p p p p p pp p p p pp pp pp pp p p p p p p p p p p ppp p p p pp p pppppp p ppp pp pp pp ppp pp pp p p pp pp pp p ppppp ppp p p p p p p p p p p p pp pp p p ppp ppppp pp pp p p p p p p p p p p p p p p p p p ppp ppp p pp p ppppppppppppppppp p pp ppp ppp p ppppp p p p p p p p p p p p p p p p p p p p p p p p p p p p p p p p p p p p p p p p p p p p pppppppppppppppppppppppppppppppppppppppppppppppppppppppppppppp ppppppppppppppppppppppppp ppp ppppppppp p pppp p pp p p p p p p pppppp p pp ppppp pppp pppp p p p p pp p pp pp p p pp p p p p p p p ppppppppppp ppp pp p pp p p p ppp pp pppppp pppp pp pp p p p p pp p pp pp pp pp pp pppp p p p pp p p p p p p p p p pp p pp p ppp pp pp ppp pp pp p p p p p p p p p p p pppppppppp ppp p pppppppppp p p p p pp ppppppp ppp pppp pppp pppppppp p p p p

Figura 4.4. Ejemplos 4.10 y 4.11: Las curvas f (x) = c, c = 3, 2, 1, 0, . . . , direcciones A-ortogonales p0 y p1 , la soluci on exacta x = 1 A b, y x0 y x1 . (ii) Procedemos por inducci on. Para k = 1, (ii) es la consecuencia de (i). Supongamos ahora que (Axk b)T pj = 0, Hay que demostrar que (Axk+1 b)T pj = 0, j = 0, . . . , k. (4.44) j = 0, . . . , k 1.

4.5. EL METODO DE GRADIENTES CONJUGADOS (CG) DE HESTENES Y STIEFEL

99

Para j = k , (4.44) es una consecuencia obvia de (i). Pero para j < k , calculamos que (Axk+1 b)T pj = A(xk k pk ) b
T

pj

= (Axk b)T pj k pT k Apj = 0. Esta u ltima expresi on es cero debido a la hip otesis de inducci on y la denici on de los vectores pj . (iii) Para k = n, sabemos que (Axn b)T p0 Dado que p0 pn1 = 0.

= (Axk b k Apk )T pj

pn1 es una matriz regular, se tiene que Axn = b.

Ejemplo 4.11. Continuamos considerando el sistema del Ejemplo 4.10, partiendo de x0 = 0 1 2 0,7 = 0 . 0,67332005

En este caso, obtenemos de (4.42) con k = 0 (1, 0) 0 = 2 1 1 3 0 3 1 2 0,7 1 1 0

2 1 (1, 0) 1 3 2 0,7 1 3 (1, 0) 6 0,7 + 3 1 2 (1, 0) 1

2 0,7 4 = = 2

0,7 2 = 1,16333997347,

entonces x1 = 0 1 ( 0,7 2) 1 2 0,7 0 2 1 1 3 (1, 2) x2 = 2 0,7 1 2 0,7 =

2 0,7 1 2 0,7

1,16333997347 . 0,673320053068

Luego calculamos (1, 2) 1 = 5 2 0,7 3 1 2 0,7 1 1 2 7 , 2

2 1 1 3 7 2

= 2 1

1 5

1 2

= x .

La Figura 4.4 muestra x0 y x1 .

100

DE SISTEMAS DE ECUACIONES LINEALES 4. METODOS ITERATIVOS PARA LA SOLUCION

La construcci on del Teorema 4.18 implica que la direcci on pk s olo se necesita cuando xk ya ha sido determinado. Podemos aprovechar esa observaci on para generar las direcciones A-ortogonales pj mediante un m etodo de ortogonalizaci on sucesiva (con respecto al producto escalar (x, y) = xT Ay) durante la computaci on, partiendo de seg un p0 := Ax0 b = f (x0 ),
k 1 j =0

pk = f (xk ) +

kj pj ,

(4.45)

donde f (xk ) = 0 (sino ya se ha encontrado el m nimo deseado), donde kj se determina de tal forma que pT j Apk = 0, La observaci on importante para poder ejecutar el m etodo es que resultar a que k,0 = . . . = k,k2 = 0, k,k1 = (f (xk ))T f (xk ) . (f (xk1 ))T f (xk1 ) (4.47) j = 0, . . . , k 1. (4.46)

Eso signica que cada paso requiere s olo muy poco esfuerzo computacional y espacio de almacenaje. Teorema 4.19. Sea la funci on f denida por (4.39), con A Rnn sim etrica y denida positiva. Sea x0 Rn arbitrario y xk+1 = xk k pk , donde pk = f ( x k )

para k = 0, f (xk ) pk1 f (xk1 ) 2 2


2 2

k =

f ( x k ) +

para k > 0,

(f (xk ))T pk pT k Apk

Entonces, f (xj ) = 0 para j = 0, . . . , k implica que {p0 , . . . , pk } son direcciones Aortogonales, es decir existe un ndice N n tal que xN = A1 b. Demostraci on. En lo siguiente, sea rj := f (xj ) = Axj b, j := rj 2 2 . rj 1 2 2

Para k = 0, notamos que si r0 = 0, entonces p0 = r0 = 0. Para k = 1, hay que demostrar que r1 = 0 implica que pT 1 Ap0 = 0, pero p1 = 0. Pero sabemos que p1 = r1 + 1 r0 implica que
T pT 1 Ap0 = p1

1 (r1 r0 ) 0

4.5. EL METODO DE GRADIENTES CONJUGADOS (CG) DE HESTENES Y STIEFEL

101

1 T (r + 1 rT 0 )(r1 r0 ) 0 1 1 rT 1 r1 T T = rT r r r + r0 r1 rT 1 r1 . 1 1 1 0 T 0 r0 r0 x1 = x0 0 r0 , 0 =

Ahora, en virtud de rT 0 r0 , T r0 Ar0 r1 = Ax1 b = Ax0 0 Ar0 b = r0 sabemos que


T rT 0 r1 = r0 r0

rT 0 r0 Ar0 , T r0 Ar0

rT 0 r0 rT 0 Ar0 = 0. T r0 Ar0

T T Etonces, resulta pT 1 Ap0 = 0. Puesto que r1 p1 = r1 r1 = 0, se tiene que p1 = 0. Finalmente, consideremos el paso k k + 1. Hay que demostrar ahora que si rk+1 = 0 y {p0 , . . . , pk } son A-ortogonales, entonces {p0 , . . . , pk+1 } son A-ortogonales, es decir, T pT k+1 Apj = 0 para j = 0, . . . , k , y pk+1 = 0. Para tal efecto, notamos que rk+1 pk = 0 implica T rT k+1 pk+1 = rk+1 rk+1 = 0,

es decir, pk+1 = 0. Luego consideramos que 1 Apj = (rj +1 rj ) j 1 = (pj +1 j +1 pj pj + j pj 1 ), j entonces, seg un (ii) del Teorema 4.18,
T T pT k+1 Apj = rk+1 + k+1 pk Apj

= rT k+1 Apj 1 T T T = r pj +1 j +1 rT k+1 pj rk+1 pj + j rk+1 pj 1 = 0. j k+1 1 (rk+1 rk ) k

Para j = k , podemos escribir


T T pT k+1 Apk = rk+1 + k+1 pk

1 T T T rk+1 rk+1 + k+1 pT k rk+1 rk+1 rk k+1 rk pk . k

102

DE SISTEMAS DE ECUACIONES LINEALES 4. METODOS ITERATIVOS PARA LA SOLUCION

Tomando en cuenta que pT k rk+1 = 0, obtenemos pT k+1 Apk 1 = k rT k+1 rk+1 rT pk rT 1 k k+1 rk . T rk rk

Usando que [. . . ] = 0, podemos seguir escribiendo 1 T pT r (pk k pk1 ). k+1 Apk = k k+1 Finalmente, sabemos que rT k+1 pk = 0, por lo tanto pT k+1 Apk = k T r pk1 k k+1 k T = A(xk k pk ) b pk1 k k T r pk1 k pT = k Apk1 = 0, k k

lo que concluye la demostraci on. Ejemplo 4.12 (Tarea 22, Curso 2006). Resolver el sistema Ax = b con 5 1 1 4 A = 1 5 1 , b = 2 1 1 5 4

usando el m etodo cg de Hestenes y Stiefel, x0 = 0, y calculando exactamente con fracciones. Soluci on sugerida. Con D(x) = Ax b obtenemos sucesivamente 1 4 4 T D(x0 ) = 2 , p0 = 2 , (D(x0 )) p0 = 36, Ap0 = 18 1 , 1 4 4 2 1 2 T 1 ; p0 Ap0 = 180, 0 = , x1 = 5 5 2 1 1 2 2 18 18 18 2 , (D(x1 ))T p1 = 4 , Ap1 = 8 , D(x1 ) = 4 , p1 = 5 25 25 25 1 1 2 2 1 18 5 0 . pT Ap = 20 , = , x = 1 1 2 1 25 18 1

Entonces, a pesar de su estructura iterativa, el m etodo entrega (si se usa aritm etica exacta) la soluci on de un sistema lineal con una matriz A sim etrica y denida positiva despu es de a lo m as n pasos. Si A es dispersa, es decir, posee s olo pocos elementos diferentes de cero, cada paso cuesta poco esfuerzo computacional. Sin embargo, el m etodo es muy sensitivo con respecto a errores de redondeo; por lo tanto, despu es de n pasos, obtenemos

4.5. EL METODO DE GRADIENTES CONJUGADOS (CG) DE HESTENES Y STIEFEL

103

solamente una soluci on aproximada (falsicada) y no exacta. Se puede empezar el m etodo de nuevo con xn como vector inicial, o simplemente se puede continuar. Puede parecer sorpresivo que un m etodo iterativo, como SOR, a un puede competir con el m etodo cg. Eso tiene que ver con que en la pr actica, para sistemas lineales de gran tama no no se necesita la soluci on exacta, y frecuentamente se desea terminar el m etodo despu es de pocos pasos de iteraci on. Ahora, mientras que un m etodo tal como el m etodo SOR garantiza una reducci on del error m as o menos igual en cada paso, el m etodo cg es un poco irregular en este aspecto, como ilustra el siguiente teorema. Teorema 4.20. Sea A Rnn sim etrica y denida positiva con los valores propios 1 2 . . . n > 0, b Rn , x = A1 b y 1 E (x) := (x x )T A(x x ). 2 Entonces la sucesi on {xk } generada por el m etodo cg satisface E (xk+1 ) =
2

1 2 m n (x0 x )T A I + APk (A) (x0 x ) 2 Pk k m n m ax 1 + i Pk (i ) E (x0 ) k+1 n k+1 + n


2

Pk k 1 i n

(4.48)

E (x0 ),

k = 0, . . . , n 1.

Recordamos que k es el espacio de los polinomios con coecientes reales del grado m aximo k . Demostraci on. Primero demostramos que pj = Pj (A)r0 , Para tal efecto, notamos primero que p0 = r0 = P0 (A)r0 , P0 1 0 .
j 1 i=0

Pj j ,

r0 = Ax0 b.

(4.49)

Luego supongamos que se ha demostrado (4.49) hasta el ndice j 1. Entonces tenemos que pj = Axj b + j pj 1 = A x0 = Ax0 b = Pj (A)r0 , donde Pj ( ) = 1 + j Pj 1 ( )
j 1 i=0 j 1 i=0

i pi

b + j pj 1
j 1 i=0

i Api + j pj 1 = r0

i Api + j pj 1

i Pi ( ).

Notando que Pi j 1 para i = 0, . . . , j 1 (seg un hip otesis), concluimos que Pj j .

104

DE SISTEMAS DE ECUACIONES LINEALES 4. METODOS ITERATIVOS PARA LA SOLUCION

En virtud de lo anterior, podemos escribir x j x = x0 x = x0 x = I


j 1 i=0 j 1 i=0 j 1 i=0

i pi i Pi (A)r0

i Pi (A)A (x0 x )

donde denimos

= I + AQj 1 (A) (x0 x ), Qj 1 ( ) :=


j 1 i=0

i Pi ( );

Qj 1 j 1 . VT V = VVT = I.

Sea ahora A = VT diag(1 , . . . , n )V, Entonces, usando := diag(1 , . . . , n ), podemos escribir Denimos y := (1 , . . . , n )T = V(x0 x ), y notamos que I + Qj 1 () es una matriz diagonal. Entonces, podemos escribir 1 E (xk+1 ) = (xk+1 x )T VT V(xk+1 x ) 2 1 2 = yT I + Qk () y 2 n 1 2 2 = i i 1 + i Qk (i ) . 2 i=1 En virtud de (4.43), este es el valor m as peque no que puede ser alcanzado a trav es de la construcci on pj = Fj (A)r0 , Tomando en cuenta que la dependencia de los coecientes j y i es presente s olo en el polinomio Qk , podemos escribir 1 E (xk+1 ) = m n Fk k 2
n 2 i i 1 + i Fk (i ) i=1 2 2

V(xj x ) = I + Qj 1 () V(x0 x ).

xj +1 = xj j pj ,

j = 0, . . . , k,

Fj j .

Fk k 1 i n Fk k 1 i n

m n m ax 1 + i Fk (i )

1 2

n 2 i i i=1

(4.50)

= m n m ax 1 + i Fk (i ) E (x0 ).

4.5. EL METODO DE GRADIENTES CONJUGADOS (CG) DE HESTENES Y STIEFEL

105

1,0 pppp 0,9 0,8 0,7 0,6 0,5 0,4 0,3 0,2 0,1 0,0

ppp ppp ppp ppp 1 + Fk () (k = 4), i = 13 2i, i = 1, . . . , n = 6 ppp ppp ppp ppp ppp ppp ppp ppp ppp ppp ppp ppp ppp ppp ppp ppp ppp ppp ppp ppp ppp ppp ppp ppp ppp ppp ppp ppp ppp ppp ppp ppp ppp ppp ppp ppp ppp ppp ppp ppp ppp ppp ppp ppp n + k+1 ppp =2 ppp ppp 2 ppp ppp ppp ppp ppp ppp ppp pp = k+1 pppppppppppppppppppppppppppppppppppp 6 = n pppppp? ppppppp1 ppppppppppppppppppppppppppppppppppppppppppppppppppppppp pppppp3 ppppppppppppppp pppppppp pppppppppppppppppppppppppppppppppppppppppppppppppppppppppppp pppp 5 pp ppp2 pppppp pppppppppppppppppp p p p p p p p ppppp p p pppp p p p p p p p p p p p ppppppp p pppp p p p p p p p p p = p p ppppppppppppp p p p 4 k p ppp p p p p p pppppppppppppp ppp

10

12

Figura 4.5. La funci on 1 + Fk () para k = 4 y i = 13 2i, i = 1, . . . , n = 6. Para acotar el lado derecho de (4.50), escogimos la siguiente funci on Fk k : Fk () =

2 (1)k+1 1 . . . k (k+1 + n )

k+1 + n 2

m=1

( m ) 1 .

(4.51)

Es decir,
1 + Fk () = 0 para {1 , . . . , k }

k+1 + n . 2

106

DE SISTEMAS DE ECUACIONES LINEALES 4. METODOS ITERATIVOS PARA LA SOLUCION

() k+1 , este polinomio es similar al polinomio gracado en la Figura 4.5. Dado que 1+Fk Hasta un valor k+1 + n , k , 2 el polinomio 1+Fk () es mon otonamente decreciente y convexo, y mon otonamente creciente k ]. El valor es denido por (pero decreciente en valor absoluto) en [,

k+1 + n , k , 2 Concluimos que


[n , k+1 ] = 1 + Fk ()

) = 0. ) + F ( ) ( (Fk k

2 k+1 + n

k+1 n . k+1 + n

Entonces, si por ejemplo, 1 n , los n 1 primeros pasos generan solamente una reducci on muy peque na del error. i = i1 , i = 2, . . . , n 1, 1 n ,

Teorema 4.21. Sea A Rnn sim etrica y denida positiva con k < n valores propios reales y distintos 1 , . . . , k . En este caso, el m etodo cg ya converge despu es de k iteraciones para Ax = b, o sea, xk = x . Demostraci on. Usamos el Teorema 4.20, que indica que la cantidad 1 E (x) := (x x )T A(x x ) 2 satisface la desigualdad E (xk+1 )
Pk k 1 i n

m n m ax 1 + i Pk (i ) E (x0 ),

donde k es el espacio de todos los polinomios del grado maximo k . Ahora hay que demostrar que bajo los hip otesis de la tarea, E (xk ) = 0, es decir que existe un polinomio Pk1 k1 tal que 1 + i Pk1 (i ) = 0, i = 1, . . . , n. k k P (4.52) Ahora sabemos que el polinomio 1 ) ( k ), k () = ( P Pk1 () := 1

1 k . Entonces k (i ) = 0, i = 1, . . . , n, con P (0) = (1)k satisface P (1)k P () 1 1 k k tal que se satisface (4.52).

es un polinomio en k1

Cap tulo 5

El problema de valores propios de una matriz


Discutiremos ahora el problema de la localizaci on y la determinaci on num erica de los nn nn valores propios reales y complejos de una matriz A C (o A R ) y los vectores propios asociados, o sea el problema de determinar los ceros del polinomio y la soluci on de los sistemas homog eneos (A i I)xi = 0. Formalmente, la soluci on del problema es dada por (a) la determinaci on de los coecientes de pn (; A), (b) la computaci on (exacta o aproximada) de sus ceros y (c) la soluci on de los sistemas lineales homog eneos. Sin embargo, en la pr actica, este camino es absolutamente in util, bajo el aspecto del esfuerzo computacional tanto que bajo el de la estabilidad num erica. Para ilustrar el u ltimo punto, consideremos un peque no ejemplo. Ejemplo 5.1. La matriz A= 1000 1 1 1000 pn (; A) := det(A I)

tiene los valores propios 1 = 1001 y 2 = 999. Ahora, si modicamos A a 1 = 1000,001 A , 1 1000 obtenemos 1 = 1001,00050 . . . y 2 = 999,00050 . . . . Sabemos que p2 (, A) = 2 2000 + 999999, ) = 2 2000,001 + 1000000. p2 (, A

) se cambia a 1000002 (correspondiente a la magnitud Ahora, si el coeciente 106 en p2 (, A de errores de redondeo en una aritm etica con 6 d gitos), el polinomio modicado tiene los ceros es decir que la inuencia del error en los coecientes de p2 ( : A) es casi 2000 veces mayor que la de los errores en la matriz original. 5.1. La localizaci on de valores propios y la sensitividad del problema Para la siguiente discusi on es u til recordar el siguiente teorema. Teorema 5.1. Sea A Cnn y una norma matricial inducida por una norma vectorial. Entonces cada valor propio (A) satisface |(A)| r (A) A .
107

2 2000,001 + 1000002 = 0 = 1000,0005 0,99999927i,

108

5. EL PROBLEMA DE VALORES PROPIOS DE UNA MATRIZ

Teorema 5.2 (Los c rculos de Gershgorin). Para una matriz A Cnn denimos los c rculos
n n

Ki :=

| ii |

j =1,j =i

|ij |

i := K

| ii |

j =1,j =i

|ji |

para i = 1, . . . , n. Sea (A) un valor propio de A, entonces


n n

(A)

i=1

Ki

i=1

i K

Demostraci on. Sean (A) y x el vector propio asociado, es decir Ax = x, e i {1, . . . , n} tal que |xi | = x . Entonces, la componente i de la ecuaci on vectorial Ax = x es
n n

j =1

ij xj = xi

ij xj + ii xi = xi ,
j =1 j =i

lo cual entrega que


n

ii = Dado que xi = 0, podemos concluir que


n

ij
j =1 j =i

xj . xi

| ii |

j =1 j =i

|ij |

|xj | |xi |

j =1 j =i

|ij |,

y luego se toma en cuenta que A y A tienen los mismos valores propios. Dado que las matrices A y D1 AD poseen los mismos valores propios, a veces se puede precisar el resultado del Teorema 5.2 signicativamente. Ejemplo 5.2. La matriz 1 103 104 2 103 A = 103 4 3 10 10 3

es sim etrica, entonces sus valores propios son reales, y seg un el Teorema 5.2, cada valor propio de A satisface Ahora, usando las matrices [1 0,0011, 1 + 0,0011] [2 0,002, 2 + 0,002] [3 0,0011, 3 + 0,0011]. D1 := diag(1, 100, 10), D2 := diag(100, 1, 100), D3 := diag(10, 100, 1),

obtenemos las siguientes inclusiones respectivas: U1 := [1 2 105 , 1 + 2 105 ] [2 0,11, 2 + 0,11] [3 0,0011, 3 + 0,0011], U2 := [1 0,1001, 1 + 0,1001] [2 2 105 , 2 + 2 105 ] [3 0,1001, 3 + 0,1001],

DE VALORES PROPIOS Y LA SENSITIVIDAD DEL PROBLEMA 5.1. LA LOCALIZACION

109

lo que implica

U3 := [1 0,0011, 1 + 0,0011] [2 0,02, 2 + 0,02] [3 2 105 , 3 + 2 105 ],


3

U1 U2 U3 =

i=1

[i 2 105 , i + 2 105 ].

Teorema 5.3. Consideremos las hip otesis del Teorema 5.2. Sea {i1 , . . . , in } una permutaci on de {1, . . . , n} y Entonces Ki1 . . . Kim contiene exactamente m valores propios de A, contados con su multiplicidad, es decir, cada componente de conectividad por camino de Ki1 . . . Kim contiene tantos valores propios de A que c rculos. Demostraci on. Sean D = diag(11 , . . . , nn ) y B( ) := D + (A D), 0 1, es decir, B(0) = D y B(1) = A. Todos los valores propios de B( ) estan contenidos en K1 ( ) . . . Kn ( ), donde denimos
n

(Ki1 . . . Kim ) Kis = para s = m + 1, . . . , n.

Ki ( ) =

zC

|z ii |

j =1,j =i

|ij |

i = 1, . . . , n.

Obviamente, el Teorema 5.3 es v alido para B(0), adem as, los valores propios dependen de forma continua de (ver Lema 5.1 abajo). Pero como Ki1 (0) Kim (0) contiene exactamente m valores propios de B(0), y
m m

[0, 1] :

j =1

Kij ( )

Kis (1)

j =1

Kij (1)

Kis (1) = , 1.

Lema 5.1. Sean A, B Cnn , 1 , . . . , n los valores propios de A, contados con su multiplicidad, y 1 , . . . , n los valores propios de B contados con su multiplicidad. Sean := m ax |ij |, |ij | : 1 i, j n , 1 := n
n n

entonces Ki,1 ( ) Ki,m ( ) contiene exactamente m valores propios para 0

i=1 j =1

|ij ij |.

Entonces existe una enumeraci on de los valores propios i y i tal que a cada i corresponde un valor i con n |i i | 2(n + 1)2 . Demostraci on. Ver A.M. Ostrowski, Solution of Equations in Euclidean and Banach Spaces, Academic Press, 3rd ed., 1973, pp. 334335, 276279. Ejemplo 5.3. En virtud del Teorema 5.3, podemos mejorar ahora el resultado del Ejemplo 5.2: los valores propios de la matriz A en este ejemplo pueden ser enumerados de tal forma que i [i 2 105 , i + 2 105 ], i = 1, 2, 3.

110

5. EL PROBLEMA DE VALORES PROPIOS DE UNA MATRIZ

Cuando conocemos un vector propio x aproximado (de hecho, como tal vector podemos usar cualquier vector x con Ax = 0), podemos usar el cuociente de Rayleigh R(x; A) := x Ax x x

para denir una aproximaci on del valor propio correspondiente, para la cual tambi en podemos denir una inclusi on. Teorema 5.4. Sea A Cnn similar a una matriz diagonal con los valores propios 1 , . . . , n . Sean x Cn , x = 0 y Ax = 0. Sea := R(x; A). Entonces Ax cx 2 (i) c C : Ax x 2 2. 2 (ii) Existe un valor j = 0, j (A), tal que j j

Ax x Ax 2

cond

(U),

donde U = u1 . . . un es un sistema completo de vectores propios de A. (iii) Si A es normal (es decir, AA = A A), entonces existe un valor 0 j (A) tal que j j Demostraci on. (i) Supongamos que x
2

Ax x 2 . Ax 2

= 1. Entonces
2 2

Ax cx

= (x A c x )(Ax cx) = Ax
2 2 2 2

= x A Ax c x Ax cx A x + |c|2 Ax + |c x Ax|2 |x Ax|2 |x Ax|2

con igualdad para c = . (ii) Sean U1 AU = diag(1 , . . . , n ) =: , y := U1 x y glb(U) := sup | x Entonces Ax x Ax 2


2

Ux

1 . U1

U( I)U1 x 2 UU1 x 2 glb(U) ( I)y 2 U 2 y 2

DE VALORES PROPIOS Y LA SENSITIVIDAD DEL PROBLEMA 5.1. LA LOCALIZACION

111

1 U 2 U1

1/2 2 i 2 |i i |2 |i |2 + || i i=1 i=1 1 i =0 i =0 = n cond 2 (U) |i i |2


n n
i=1 i =0

|i |2 |i |2 i=1 n |i |2 |i |2
i=1

1/2

cond

(U)

1 i n i =0

m n

i . i

(iii) Si A es normal, existe un sistema de vectores propios unitario, o sea, cond

(U) = 1.

Para las matrices hermitianas el cuociente de Rayleigh tiene muchas propiedades interesantes. Teorema 5.5. Sea A Cnn hermitiana con un sistema X = x1 x2 . . . Cn tal que donde Axj = xj para j = 1, . . . , n. Sea x
n

xn unitario,

x = 1, x Entonces

= xj + x
k=1 n

k x k ;

|k |

k = 1, . . . , n.

R( x; A) j

i=1 i=j

|i j ||i |2

2 A (n 1)2 ,

(5.1)

o sea el error en R( x; A) es cuadr aticamente peque no en t erminos de los errores de la aproximaci on del vector propio. Demostraci on. Utilizando que
n

A=
i=1

i xi x i,

podemos escribir
n n n

R( x ; A) = =

xj +
k=1 n

k x k
i=1 n

i xi x i

xj +
k=1 n

k x k k x k
k=1

i
i=1

xj +
k=1

k xk

xi

x xj + i

112

5. EL PROBLEMA DE VALORES PROPIOS DE UNA MATRIZ


n n n

=
i=1 n

ij +
k=1 n

k ki
2

xj + x i
k=1

k x k

=
i=1 n

i ij +
k=1

k ki

=
i=1 i=j

i |i |2 + j |1 + j |2 (i j )|i |2 + j |1 + j |2 +
n

=
i=1 i=j

i=1 i=j

Usando que

|i |2 .

|1 + j | + llegamos a

i=1 i=j

x = 1, |i |2 = x

R( x; A) j =

i=1 i=j

i |i |2 + j |1 + j |2 ,

lo que implica (5.1) si tomamos valores absolutos, aplicamos la desigualdad del tri angulo y la cota trivial |i j | 2r (A) 2 A .

Teorema 5.6. (El principio minimax de Courant) Sea A Cnn hermitiana. Los valores propios de A, contados con su multiplicidad, sean 1 2 . . . n . Sea Vj el sistema de todos los subespacios j -dimensionales de Cn , donde denimos V0 := {0}. Entonces, k = m n m ax R(x; A) | x = 0, v V : x v = 0 ,
V Vk1 V Vnk

(5.2) (5.3)

k = m ax m n R(x; A) | x = 0, v V : x v = 0 .

Demostraci on. Sea u = u1 un un sistema unitario completo de vectores propios de A, o sea, Aui = i ui , U U = I. Si x Cn es arbitrario, podemos escribir
n

x=
i=1

i ui ;

i = u i x,

i = 1, . . . , n.

Deniendo = diag(1 , . . . , n ), podemos escribir R(x; A) = x UU x = x U Ux


n

i=1

|i |2 i , |1 |2 + . . . + |n |2

DE VALORES PROPIOS Y LA SENSITIVIDAD DEL PROBLEMA 5.1. LA LOCALIZACION

113

lo que implica que R(x; A) k k


k

si k+1 = . . . = n = 0, si 1 = . . . = k1 = 0.

Cn tal que Ahora, demostramos que si V Vk1 , existe x = 0, x = x


i=1

i ui ,

v = 0. v V : x

(5.4)

Para demostrar (5.4), consideremos una base ortonormalizada v1 , . . . , vk1 de V . Sea g = (1 , . . . , k )T , g = 0 la soluci on de v1 . . . u1 uk g = 0. En este caso,
vk 1

Cn tal que Si V Vnk , existe x = 0, x

Tenemos igualdad en (5.5) para V = span{u1 , . . . , uk1 }, es decir, 0 . . . g= 0 , k = 0. k


n

m ax R(x; A) | x = 0, v V : x v = 0

k .

(5.5)

= x
j =k

j uj ,

v = 0. v V : x

(5.6)

Para demostrar (5.6), consideremos una base ortonormalizada v1 , . . . , vnk de V . Sea g = (k , . . . , n )T , g = 0 la soluci on de v1 . . . uk un g = 0. En este caso,
vn k

Tenemos igualdad en (5.7) para V = span{uk+1 , . . . , un }, es decir, k 0 , k = 0. g= . . . 0

m n R(x; A) | x = 0, v V : x v = 0

k .

(5.7)

114

5. EL PROBLEMA DE VALORES PROPIOS DE UNA MATRIZ

El siguiente teorema formula una consecuencia del Teorema 5.6. Teorema 5.7. Sean A, B Cnn ambas hermitianas. Sean i (A) y i (B) los valores propios de A y B, enumerados tales que 1 (A) . . . n (A), 1 (B) . . . n (B). Entonces, k = 1, . . . , n : k (A) k (B) r (B A). (5.8)

Demostraci on. Denir B := A + (B A), C := B A, aplicar el Teorema 5.6 (Tarea). Por supuesto, el Teorema 5.7 representa un resultado mucho m as ventajoso que el Lema 5.1. Tambi en es v alido para valores propios m ultiples. Por supuesto, la restricci on que ambas matrices deben ser hermitianas es muy fuerte. Adem as, tenemos tambi en que los valores propios de una matriz diagonalizable dependen de forma Lipschitz continua de los coecientes: Teorema 5.8. Si A Cnn es diagonalizable, es decir, existe un sistema U = u1 un de vectores propios de A, y B Cnn es arbitraria, entonces para cada valor propio j (B) existe un valor propio i(j ) (A) tal que i(j ) (A) j (B) cond

(U) B A

(5.9)

Demostraci on. Nos restringimos al caso no trivial. Sea (B) (B), (B) = i (A) para i = 1, . . . , n, con el vector propio x = 0, es decir, Bx = (B)x. Entonces Bx Ax = (B)x Ax x = (B)I A x
1

implica que

(B A)x

Ahora, usando I = UU1 y A = UA U1 , donde A es una matriz diagonal de los valores propios de A, tenemos que 1 B A x . x m n |(B) i (A)|
1 i n

(B)I A

BA

No existe un resultado an alogo para matrices no diagonalizables. El siguiente ejemplo ilustra que es muy natural que un tal resultado no existe. Ejemplo 5.4. La matriz A= 0 1 0 0 0 1 0 C

posee los valores propios 1 = 2 = 0, y no es diagonalizable. La matriz B=

posee los valores propios , mientras que en cualquier norma matricial, A B (con una constante C que depende de la norma).

DE UNA MATRIZ A FORMA DE HESSENBERG O TRIDIAGONAL 5.2. TRANSFORMACION

115

Finalmente, nos interesan resultados asint oticos sobre los valores y vectores propios. El siguiente teorema, que se comunica sin demostraci on, representa un resultado t pico. Teorema 5.9. Sea A Cnn diagonalizable, X = x1 xn un sistema completo de vectores propios de A, Axi = i xi y xi 2 = 1 para i = 1, . . . , n. Ademas denimos y .1 . , X1 =: Y =: . yn
i para i = 1, . . . , n. A = yi son los vectores propios de la izquierda de A: yi , . . . , yn o sea, y1 nn Sea j un valor propio simple de A. Entonces, para F C con F 2 sucientemente peque na existe un valor propio j de A + F con un vector propio zj , zj 2 = 1, tal que

j = j +

Demostraci on. Se usa el Teorema de Funciones Impl citas para el problema g(x, , ) = 0 con g(xj , j , 0) = 0, g(x, , ) = (A + F0 )x x , xT x 1 F = F 0 ,

z j = xj +

yj Fxj yj 2 xj n
i=1 i=j

yj 2 xj yj xj

yj Fxi yj 2 xj

1 yj 2 xj xi yj xj 2 i j

+O F

2 2

+O F

2 2

con en una vecindad apropiada de cero, y se representa la soluci on (x, ) como funci on de .
Obviamente, el factor de amplicaci on del error decisivo es yj 2 xj 2 /|yj xj | para un valor propio (este factor puede ser grande para matrices no normales), mientras que para un vector propio, tambien juega un rol importante la separaci on de los valores propios.

5.2. Transformaci on de similaridad unitaria de una matriz n n a una forma de Hessenberg o tridiagonal La soluci on del problema de valores propios para una matriz no esparsa siempre empieza con la transformaci on de la matriz a una forma condensada. Esa transformaci on genera nueavos errores de redondeo. Para que la matriz transformada aun sea usable, los valores propios no deben ser m as falsicados que como si la matriz fuera modicada dentro de la exactitud aritm etica. Por lo tanto, s olo es practicable la transformaci on a la forma de Hessenberg. La transformaci on parte de la matriz A, luego determinamos matrices unitarias y hermitianas U1 , . . . , Un2 tales que Un2 . . . U1 AU1 . . . Un2 es una matriz del tipo Hessenberg. Ahora, si la matriz A es hermitiana, entonces (Un2 . . . U1 AU1 . . . Un2 ) = Un2 . . . U1 AU1 . . . Un2 , o sea la matriz del tipo Hessenberg es hermitiana y por lo tanto, tridiagonal.

116

5. EL PROBLEMA DE VALORES PROPIOS DE UNA MATRIZ

La transfomacion procede en n 2 pasos. Supongamos que despu es de j 1 pasos, tenemos la matriz (j ) (j ) (j ) (j ) (j ) 11 12 1,j 1 1j 1n (j ) (j ) (j ) (j ) (j ) 2n 21 22 2,j 1 2j . . . (j ) . . . 0 32 . . . . ... (j ) (j ) (j ) . Aj = Uj 1 . . . U1 AU1 . . . Uj 1 = . 0 j,j 1 jj jn . . . . .. (j ) . . . . . . 0 j +1,j . . . . . . . . . . . . . . . . 0 0 nj
(j )

nn

(j )

Si denimos Aj +1 := Uj Aj Uj , tenemos (ver Teorema 3.6) Uj = I 0 j , 0 U

j = I k w jw j U ,

j de tal forma que donde se determina U (j ) 1 j +1,j . = exp(i) 0 j . U . j . . . . (j ) nj 0 El Teorema 3.6 entrega las f ormulas (j ) exp(ij ) j +1,j + j (j ) j +2,j j = w . . . nj
(j ) (j )

j +1,j = exp(ij ) j +1,j ,

(j )

1/2 (j ) 2 kj

j =
k=j +1

j =

1 j (j + |j +1,j |)
(j )

Puesto que las primeras j columnas de Uj son columnas unitarias, la multiplicaci on de Uj Aj desde la derecha con Uj no cambia los ceros recien generados en la columna j . La multiplicaci on de Aj desde la izquierda por Uj no cambia las primeras j las. Por lo tanto, hemos desarrollado completamente la transformaci on. Para la ejecuci on pr actica de la transformaci on, aprovechamos la estructura especial de Uj . Sea Aj = A11 A21
(j ) (j )

A12 A22

(j ) (j )

DE UNA MATRIZ A FORMA DE HESSENBERG O TRIDIAGONAL 5.2. TRANSFORMACION

117

En este caso, obtenemos Aj +1 = donde j A(j ) = A(j ) j wj w A(j ) U 21 21 j 21


(j ) (j ) (j ) A12 U j = A12 j A12 wj wj (j ) (j ) j = (I j w j A( j ) U w jw j ) A22 j A22 w U 22

A11

(j )

(j ) A12 U j

j A( j ) U j A(j ) U j U 21 22

jz = A21 u j,
(j )

(j )

j u = A12 y j,

(j ) (j ) (j ) (j ) j j u ju j w j j u A22 w = A22 u A22 A22 w j j + w j j (j ) j j u = A22 u s u tj u j j, 2 j 2

donde denimos j := j w j, u
(j ) j, tj := A22 w j j := w tj , (j ) j s A22 , j := w (j ) j z A21 , j := w (j ) j := A12 j. y w

Pero, seg un hip otesis, 0 . = . . (j ) 0 j +1,j . . . . . . , (j ) 0 nj 0 exp(ij )j . . . 0 . . . . . . . 0 0 tj = sj ,

A21

(j )

j A(j ) expl de manera que no hay que calcular U citamente: 21 0 . . . = . . . 0

j A(j ) U 21

Adem as, en el caso A hermitiana tomamos en cuenta que


(j ) (j ) , A12 U j = Uj A21

entonces una computaci on expl cita no es necesaria, y el esfuerzo computacional total se 5 3 n + O(n2 ) operaciones; para reduce a menos que la mitad. Para A general, necesitamos 3 2 3 A hermitiana, solo 3 n + O(n2 ) operaciones esenciales.

118

5. EL PROBLEMA DE VALORES PROPIOS DE UNA MATRIZ

5.3. Computaci on de los valores propios de una matriz tridiagonal hermitiana Consideremos la matriz T Cnn dada por 1 1 .. .. . . i , i = 1, . . . , n 1; T= 1 . , i = . . . . . n1 n1 n

i R,

i = 1, . . . , n. (5.10)

Se supone que i = 0 para i = 1, . . . , n 1, sino la matriz tridiagonal puede ser particionada en dos matrices tridiagonales de tama no menor cuyos problemas de valores propios pueden ser estudiados separadamente. Teorema 5.10. Si T es una matriz tridiagonal hermitiana de la forma indicada en (5.10) y i = 0 para i = 1, . . . , n 1, entonces T s olo tiene valores propios reales simples. Demostraci on. Tarea. Comentamos si T es una matriz real no sim etrica con i i > 0, i = 1, . . . , n 1, entonces mediante una transformaci on de similaridad con D = diag(1 , . . . , n ), 1 := 1, i+1 := i i /i , := DTD1 . Entonces tales T puede ser transformada a una matriz sim etrica y tridiagonal T matrices T tambi en poseen solo valores propios reales y simples. Para la computaci on de valores propios de T, necesitamos el Teorema de la Ley de Inercia de Sylvester. Denici on 5.1. Sea A Cnn hermitiana. Entonces se dene como inercia de A al triple (m, z, p), donde m, z y p es el n umero de valores propios negativos, zero, y positivos, respectivamente. Teorema 5.11 (Ley de Inercia de Sylvester). Si A Cnn es hermitiana y X Cnn es regular, entonces A y X AX tienen la misma inercia. Demostraci on. Supongamos que 1 (A) 2 (A) ... n (A) son los valores propios de A, contados con su multiplicidad, y que para alg un r {1, . . . , n}, n r (A) es un valor propio de A positivo. Denimos el subespacio S0 R a trav es de donde Aqi = i (A)qi para i = 1, . . . , r. Utilizando la caracterizaci on minimax de r (X AX), donde se supone que 1 (X AX) ... n (X AX) (5.11) son los valores propios de X AX, obtenemos r (X AX) = m ax m n R(x; X AX) | x = 0, v V : x v = 0 .
V Vnr

S0 := span{X1 q1 , . . . , X1 qr },

q1 = 0, . . . , qr = 0,

5.3. VALORES PROPIOS DE UNA MATRIZ TRIDIAGONAL HERMITIANA

119

Ahora, escogiendo
:= S0 V := w Rn | v S0 : w v = 0 Vnr ,

deducimos de (5.11) que r (X AX) = m n R(x; X AX) | x = 0, x S0 r (A). Si 1 (X) y Rn , ... : x v = 0 m n R(x; X AX) | x = 0, v V

n (X) son los valores singulares de X, podemos demostrar que para cada R ( y , X X) n (X)2 .

Entonces, conluimos que r (X AX)


yS0

m n

y (X AX)y y (X X)y y (X X)y y y

r (A)n (X)2 .

(5.12)

Un argumento an alogo, con los roles de A y X AX intercambiados muestra que r ( A) r (X AX)n (X1 )2 = r (X AX) . 1 (X)2 (5.13)

Combinando (5.12) y (5.13), concluimos que r (A) y r (X AX) tienen el mismo signo, por lo tanto, A y X AX tienen el mismo n umero de valores propios positivos. Aplicando este resultado a A, concluimos que A y X AX tienen el mismo n umero de valores propios negativos, y obviamente, el mismo n umero de valores propios zero (debidamente contados con su multiplicidad). El Teorema 5.11 implica que las matrices A I y X (A I)X tienen los mismos n umeros de valores propios positivos, cero, y negativos, es decir, A tiene los mismos n umeros de valores propios > , = , y < ( R). Queremos aplicar este resultado ahora a la matriz T con 1 1 ... ... X Cnn , donde X1 = , . . . n1 1 X (T I)X = Q = diag(q1 , . . . , qn ), 1 ... = 1 . .. qi R,

donde se debe cumplir que o sea

T I = (X )1 QX1

.. . n1

1 1 q1 .. .. q2 . . . . . . . . n1 . qn 1 1

120

5. EL PROBLEMA DE VALORES PROPIOS DE UNA MATRIZ

Entonces, los qi son cuocientes de subdeterminantes principales sucesivos de T I. Obviamente, q1 1 = 1 q1 = 1 , 1 = 1 = 1 ), (= q1

q2 + q1 |1 |2 = 2 , q2 2 = 2 , etc. En general, tenemos

2 qk + qk1 |k qk k = k , k = 1, . . . , n, 1 | = k , 0 := 0, q0 := 1, n := 0.

Dado que k = 0 para k = 1, . . . , n 1, el valor k existe para qk = 0, k = 1, . . . , n 1, es decir, k = k /qk para k = 1, . . . , n 1. Si sucede que qk = 0, remplazamos este valor por 1, es decir remplazamos k por k + . Debido al Teorema 4.7, eso cambia los valores propios s olo en . Entonces, siempre se calcula (k1 )2 , qk 1 Seg un el Teorema 5.11, sabemos que qk = k #{k | qk < 0, 1 k k = 1, . . . , n, q0 := 1, 0 := 0. (5.14)

Este resultado lo podemos aprovechar directamente para crear un m etodo de bisecci on para calcular valores propios arbitrarios j de T. Partiendo de la enumeraci on 1 2 . . . n y, por ejemplo, de la inclusi on trivial la cual incluye todos los valores propios de T, ponemos para s N0 : as + b s , s := 2 m := #{qk | qk < 0, calculados de (5.14) con = s }, as+1 := as s si m sino, j, bs+1 := s bs si m sino. j, [a0 , b0 ] := [ T
,

n} = #{ | es valor propio de T, < }.

],

(5.15)

(5.16) (5.17) (5.18)

Para este m etodo sabemos que


s

l m s = j .

(5.19)

Este m etodo es muy robusto y extremadamente eciente. Ejemplo 5.5. Queremos determinar el valor propio 2 de 1 1 0 0 1 3 2 0 T= 0 2 5 3 , 0 0 3 7

5.3. VALORES PROPIOS DE UNA MATRIZ TRIDIAGONAL HERMITIANA

121

s 0 1 2 3 4 5 6 7 8

1 ,5 1,75 1,625 1,6875 1,71875 1,734375 1,7421875 1,74609375 1,744140625

q1 0,500000 0,750000 0,625000 0,687500 0,718750 0,734375 0,742187 0,746094 0,744141

q2 3,500000 2,583333 2,975000 2,767045 2,672554 2,627327 2,605181 2,594220 2,599691

q3 2,357143 1,701613 2,030462 1,866915 1,784555 1,743165 1,722410 1,712017 1,717215

q4 1,681818 0,039100 0,942511 0,491712 0,237975 0,102603 0,032577 0,003050 0,014816

m 1 2 1 1 1 1 1 2 1

Cuadro 5.1. Ejemplo 5.5 (m etodo de bisecci on).

2 2 2 2 2 2 2 2 2

> 1,5 < 1,75 > 1,625 > 1,6875 > 1,71875 > 1,734375 > 1,7421875 < 1,74609375 > 1,744140625

empezando con [a0 , b0 ] := [1, 2]. El m etodo de bisecci on entrega la informaci on del Cuadro 5.1. Ejemplo 5.6 (Tarea 30, Curso 2006). Se considera la matriz 10 6 8 A = 6 17 2 . 8 2 20

a) Transformar A a forma tridiagonal y aplicar el m etodo de bisecci on para demostrar que A es denida positiva. b) Usando el m etodo de bisecci on, determinar el valor propio m as peque no hasta un error del valor absoluto 0,5. Soluci on sugerida. a) La transformaci on de la matriz a forma tridiagonal necesita un paso, es decir T = A2 = P1 A1 P1 con A1 = A. Sabemos que 1 0 0 1 = I 1 w , P 1w 1 P1 = 0 . 0 P1 Aqu 1 = 1 1 16 1 = 36 + 64 = 10, 1 = = , w , 8 10(10 + 6) 160 1 0 0 10 10 0 P1 = 0 0,6 0,8 , T = P1 AP1 = 10 17 2 . 0 0 ,8 0 ,6 0 2 20

Aplicando el Teorema de Gershgorin, vemos que T solo tiene valores propios no negativos; dado que T es regular, 0 no es valor propio; entonces los valores propios de T (y los de A) son positivos. b) El valor propio mas peque no es 1 , entonces j = 1. El valor propio esta contenido en el intervalo [a0 := 0, b0 := 32]. Obtenemos la siguiente tabla.

122

5. EL PROBLEMA DE VALORES PROPIOS DE UNA MATRIZ

k ak bk k q1 q2 q3 m 0 0 32 16 6 16. 6 3,76 1 1 0 16 8 2 41 12,097 1 2 0 8 4 6 3. 6 17.09 1 3 0 4 2 8 2 ,5 16,4 0 4 2 4 3 7 2/7 31 1 5 2 3 2,5 7,5 1,16 13,5 0 Entonces sabemos que 1 [2,5, 3]. Ejemplo 5.7 (Certamen 2, Curso 2010). Se considera la matriz 10 3 4 A = 3 2 1 . 4 1 16

a) Demostrar sin calcular el polinomio caracter stico que A tiene tres valores propios reales distintos. b) Transformar A unitariamente a forma tridiagonal sim etrica. c) Determinar n umeros i , i , i = 1, 2, 3, tales que i i i , i = 1, 2, 3, donde 1 , 2 , 3 son los valores propios de A, y |i i | 0,25, mediante el m etodo de bisecci on. Soluci on sugerida. a) Puesto que A es sim etrica, sus valores propios son reales. Los c rculos de Gershgorin son K1 = [17, 3], K2 = [2, 6], K3 = [11, 21].

Dado que Ki Kj = para i = j , cada uno de los c rculos contiene exactamente un valor propio, es decir i Ki para i = 1, 2, 3. b) Siguiendo el procedimiento can onico, determinamos U1 = 1 0 33 1 R 0 U

1 = I 1 w w T R22 ; con tal que U 1 = se tiene aqu = w 3 + 1 4 = 8 , 4 1 = 1 1 = ; 5 (5 + 3) 40 1 = 1 3 4 U 5 4 3 s2 + (4)2 = 5

y la matriz tridiagonal deseada 1 0 0 10 3 4 1 0 0 10 5 0 T = 0 0,6 0,8 3 2 1 0 0,6 0,8 = 5 10 7 . 4 1 16 0 0 ,8 0 ,6 0 7 8 0 0,8 0,6

5.3. VALORES PROPIOS DE UNA MATRIZ TRIDIAGONAL HERMITIANA

123

c) El m etodo de bisecci on, aplicado a la matriz T, requiere de la computaci on sucesiva de las cantidades q0 = 1, 0 = 0;

2 0 = 10 , q0 25 2 q2 = 2 1 = 10 , q1 q1 2 49 q3 = 3 2 = 8 , q2 q2

q1 = 1

donde el valor se ajusta seg un lo especicado en (5.16)(5.18). Se recomienda empezar la iteraci on con un intervalo cuya longitud sea una potencia de 2. As obtenemos los resultados s a b q1 q2 q3 m 0 -19.0000 -3.0000 1.0000 -4.0000 31.2500 1 1 -19.0000 -11.0000 5.0000 20.0000 20.5500 0 2 -15.0000 -11.0000 3.0000 14.6667 17.6591 0 3 -13.0000 -11.0000 2.0000 9.5000 14.8421 0 4 -12.0000 -11.0000 1.5000 4.8333 9.3621 0 5 -11.5000 -11.0000 1.2500 1.2500 -19.9500 1 6 -11.5000 -11.2500 para j = 1, por lo tanto, 1 [11,5, 11,25], s a b q1 q2 q3 m 0 -2.0000 6.0000 -12.0000 10.0833 1.1405 1 1 2.0000 6.0000 -14.0000 7.7857 -2.2936 2 2 2.0000 4.0000 -13.0000 8.9231 -0.4914 2 3 2.0000 3.0000 -12.5000 9.5000 0.3421 1 4 2.5000 3.0000 -12.7500 9.2108 -0.0699 2 5 2.5000 2.7500 a 8.0000 16.0000 16.0000 16.0000 16.0000 16.5000 16.5000 b 24.0000 24.0000 20.0000 18.0000 17.0000 17.0000 16.7500 q1 -26.0000 -30.0000 -28.0000 -27.0000 -26.5000 -26.7500 q2 -5.0385 -9.1667 -7.1071 -6.0741 -5.5566 -5.8154 q3 m 1.7252 2 -6.6545 3 -3.1055 3 -0.9329 3 0.3183 2 -0.3241 3

para j = 2, por lo tanto, 2 [2,5, 2,75], y s 0 1 2 3 4 5 6

para j = 3, por lo tanto, 3 [16,5, 16,75]. (Los valores exactos son 1 = 11,3301, 2 = 2,7080 y 3 = 16,6221.)

124

5. EL PROBLEMA DE VALORES PROPIOS DE UNA MATRIZ

5.4. Determinaci on de los vectores propios de una matriz tridiagonal hermitiana En lo siguiente, se supone que T es una matriz tridiagonal hermitiana con i = 0, i = 1, . . . , n, y sea una aproximaci on de un valor propio de T determinada con exactitud de m aquina (por ejemplo, usando el m etodo de bisecci on). Sabemos que para arbitrario, rango(T I) n 1, y que ning un de los elementos subdiagonales de T desaparece, tal que la descomposici on triangular de T I puede ser realizada completamente con intercambios de las. Para la b usqueda del pivote en la columna (medida indispensable aqu ) tenemos la descomposici on triangular P(T I) = LR, |
jj |

|j |,

j = 1, . . . , n 1.

Si = j , entonces para una computaci on sin errores de redondeo tenemos nn = 0 y una soluci on x de Rx = 0 con n = 1 sera un vector propio de T. En la pr actica, no es asegurado que siempre resulta un valor de nn peque no, incluso cuando es una muy buena aproximaci on del valor propio. El siguiente teorema informa como a pesar de lo anterior, podemos determinar una buena aproximaci on del vector propio, siempre que la aproximaci on del valor propio es sucientemente buena. Teorema 5.12. Sea T una matriz tridiagonal y hermitiana, P(T I) = LR una descomposici on triangular (determinada con b usqueda del pivote en la columna), y una aproximaci on del valor propio j de T con = j + f (n) T 2 , || 1, sucientemente peque no. Sean todos los elementos subdiagonales de T diferentes de cero. Entonces existe (por lo menos) un ndice i {1, . . . , n} tal que la soluci on xi de i1 . . Rxi = ei ii , xi = . in (con nn := 1 si i = n y xi xi
2 nn

= 0) satisface d
2

= uj + d,

|| = 1, donde U := u1 y Tui = i ui , i = 1, . . . , n. Demostraci on. Sea yi := mos En el caso y denimos

un es un sistema ortonormalizado de vectores propios de T Entonces, con T = UU , := diag(1 , . . . , n ), teneii P T

n3 f (n) T 2 + O(2 ), m n{|i j |, i = j }

(5.20)

T ii PL ei .

(T I)xi = PT LRxi =
nn

Lei = yi .
nn

= 0, ya no hay que demostrar nada, entonces podemos asumir que xi := 1


ii

= 0,

xi ,

yi :=

1
ii

yi ,

i = 1, . . . , n,

5.4. VECTORES PROPIOS DE UNA MATRIZ TRIDIAGONAL HERMITIANA

125

y se supone que xi e yi , i = 1, . . . , n, poseen las representaciones


n n

xi :=
k=1

ik uk ,

yi :=
k=1

ik uk .

Entonces sabemos que Dado que los elementos de Lei son 1 en valor absoluto, sabemos que i1 . . . = U PT Lei = | n, i, k = 1, . . . , n, ik | in
1 i n T T m ax | ij | = m ax eT j U P Lei = L PUej 1 i n

ik = (k ) ik ,

i, k = 1, . . . , n.

(5.21) 1 n3/2 . (5.22)

PUej (L1 )T

Pero deniendo := m n |i j |} > 0,


i= j

sabemos que i = 1, . . . , n, k = j : ik | = | | ik | |k j f (n) T


2

(para sucientemente peque no, T 2 f (n) > 0), mientras que para un ndice i apropiado, 1 | ij | ij | = . (5.23) | 3 / 2 |f (n) T 2 | n f (n) T 2 Para este ndice i, tenemos que 1/2 1/2 n n 2 ik | | ik |2 ij |(1 + ij ), ij | | = | xi 2 = = | 1 + ij |2 |
k=1

n f (n) T

donde, tomando en cuenta que 1 0

1+

k=1 k=j

1 + para 0
2 4 2 2 2 n f (n)

1,

ij

Entonces, xi xi
2

T . ( f (n) T 2 )2
n

xi = xi

ij = u + ij |(1 + ij ) j |

k=1 k=j

ik ij uk = u + d, ij |(1 + ij ) ij | j | | n3 f (n) T + O(2 ).

donde el vector d satisface 1 (n 1) nn3/2 f (n) T 2 d 2 1 + 2 f (n) T 2 n4 f 2 (n) T 2 2 1+ 2 ( f (n) T 2 )

126

5. EL PROBLEMA DE VALORES PROPIOS DE UNA MATRIZ

Seg un nuestra derivaci on, es obvio que el factor n3 en el enunciado del Teorema (5.12) es muy pesimista. Se puede suponer que en la mayor a de los cases, el factor 1 es m as apropiado 3 que n . Se puede demostrar que i = n es apropiado si con R= R11 0 r
nn

1 la norm a R na (por ejemplo, 11 r 2 es peque forma: se resuelven los sistemas

n). En la pr actica, se procede de la siguiente (5.24)

Rxi =

ii ei ,

i = n, n 1, . . . ; | ii | , 100n

la computaci on termina cuando por primera vez, xi

(5.25)

donde el factor 1/(100n) remplaza (de forma un poco arbitraria) e n t ermino 1/(n3/2 f (n)) (no se conoce el verdadero en ). Si (5.25) no se cumple para ning un i, tambi en la aproximaci on puede ser considerada de mala calidad. Normalmente, el test ya est a satisfecho para i = n. Adem as, se puede domostrar que modicaciones de T del orden T 2 causan errores en los vectores propios del orden n T 2 . m n |i j |
i= j

Entonces, el Teorema 5.12 representa un resultado excelente. Adem as, podemos demostrar que los errores de redondeo cometidos al calcular la descomposici on triangular de P(T I) y durante la soluci on de (5.24) no afectan seriamente el resultado. 5.5. Valores propios de una matriz de tipo Hessenberg En los cap tulos anteriores vimos que es relativamente f acil determinar un valor propio individual (y el vector propio asociado) de una matriz hermitiana. Por otro lado, vimos que cada matriz de Cnn puede ser transformada unitariamente a la forma de Hessenberg superior. Para la determinaci on de un valor propio de una tal matriz, es importante que el polinomio caracter stico pn (; A), y posiblemente su su derivada pn (; A), pueden ser evaluados facilmente. Sea = 0 arbitrario. Para la componente n del sistema lineal (11 )1 + 12 2 + + 1n n = , 21 1 + (22 )2 + + 2n n = 0, . . . (5.26)

n1 1 + n2 2 + + (nn )n = 0

DIRECTA SEGUN VON MISES Y EL METODO 5.6. LA ITERACION DE WIELANDT

127

( = 0 arbitrario) tenemos, seg un la regla de Cramer, a11 a12 a21 a22 . 1 ... . . n = det(A I) . . . n1 0 . (1)n 2 1 . . . n,n1 ... . . = , det(A I) .. . n1,n1 0 n,n1 0 ... a1,n1 a2,n1 . . .

es decir, cuando ponemos n := 1 y solvemos el sistema lineal por substituci on, obtenemos = (1)n det(A I) . 21 32 . . . n,n1 (5.27)

Esto signica que hasta un factor com un, obtenemos det(A I) y (d/d) det(A I) por la siguiente recursi on, conocida como M etodo de Hyman; xn () := 1 (corresponde a n ), d xn () := 0 (corresponde a n ,) d n 1 xni () := xni+1 () ni+1,k xk () , ni+1,ni k=ni+1 xni () := 1 ni+1,ni
n

i = 1, . . . , n 1,

xni+1 () + xni+1 ()

k=ni+1

ni+1,k xk () .

Ahora, considerando la primera ecuaci on de (5.26) obtenemos


n

det(A I) = (1)n+1 21 32 n,n1 (11 )x1 () + y an alogamente d det(A I) = (1)n+1 21 32 n,n1 d

1k xk ()
k=2

(11 )x1 () x1 () +

1k xk () .
k=2

Con m etodos iterativos para los ceros de un polinomio podemos facilmente calcular los valores propios. Hay que considerar que este m etodo es muy diferente a la computaci on de los coecientes del polinomio caracter stico. 5.6. La iteraci on directa seg un von Mises y el m etodo de Wielandt Ahora consideramos m etodos que nos entregan un vector propio aproximado. Ya vimos como mediante el cuociente de Rayleigh podemos obtener un valor propio aproximado usando el vector propio aproximado. La versi on b asica de la iteraci on de von Mises (pero de poca utilidad pr actica) es la siguiente. 1. Sea 0 = x0 Cn apropiado. 2. Para k = 0, 1, 2, . . . , sea xk+1 = Axk .

128

5. EL PROBLEMA DE VALORES PROPIOS DE UNA MATRIZ

Teorema 5.13. Sea A Cnn diagonalizable y U = u1 propios de A, con Aui = i ui , i = 1, . . . , n. Ademas sea
n

un un sistema de vectores

x0 =
i=1

i ui ,

1 = 0,

|1 | > |2 |

...

|n |.

Entonces la sucesi on de vectores {xk }kN0 satisface xk = 1 k 1 u1 + O 2 1


k

,
k

(5.28) . (5.29)

R(xk ; A) = 1 1 + O Demostraci on. Calculamos que xk = A k x 0 = u1


n k un diag(k 1 , . . . , n ) u1

2 1

un

u1

un

=
i=1

k i i ui
n

1 . . . n

1 k 1

u1 +
i=2

i 1

i 1

ui

lo cual implica (5.28) si tomamos en cuenta que i 1 Por otro lado, R(xk ; A) = x k xk+1 x k xk 1 k u + O 1 1 1 k u + O 1 1 = 1 1 + O 2 1
k k

2 1

2 1

k +1 1 k u1 + O 1 k

2 1 2 1

k+1

2 1 .

1 k 1 u1 + O

DIRECTA SEGUN VON MISES Y EL METODO 5.6. LA ITERACION DE WIELANDT

129

Comentamos primero que para |1 | = 1, la versi on presentada aqu rapidamente entrega n umeros extremadamente grandes o peque nos. Por lo tanto, se preere calcular {xk } de la siguiente forma: k+1 x k+1 := Axk , xk+1 := . x k+1 x En este caso, xk = k u1 + O 2 1
k

|k | = 1,

k+1 = 1 1 + O x kx

2 1

Si xk se normaliza a (xk )j = 1 para una componente j con (u1 )j = 0, entonces {xk }kN converge para k a un m ultiple de u1 . El Teorema 5.13 es an alogamente v alido para 1 = . . . = r , |1 | > |r+1 | . . . |n |, cuando
n r

x0 =
i=1

i ui ,
i=1

|i | = 0.

La presuposici on 1 = 0 o |1 | + . . . + |r | = 0 siempre est a satisfecha en la pr actica debido a errores de redondeo, includo cuando x0 no es apropiado. La matriz A no necesariamente debe ser diagonalizable. Pero si A no lo es, el t ermino k de error O(|2 /1 | ) es remplazado por O(1/k ). Cuando A posee diferentes valores propios dominantes (del mismo valor absoluto), por ejemplo en el caso de un valor propio dominante en valor absoluto complejo de una matriz A real, no hay convergencia (pero existen generalizaciones de este m etodo para este caso). Ejemplo 5.8 (Tarea 28, Curso 2006). 1 1 1 A = 1 2 1 1 1 10

a) Demostrar que A satisface las condiciones para la ejecuci on de la iteraci on directa T (m etodo de von Mises), y que x0 := (0, 0, 1) es un vector apropiado para iniciar la iteraci on. b) Determinar x3 (usando el algoritmo b asico, sin normalizar), y calcular usando x3 y x2 un valor aproximado del valor propio. Para esta aproximaci on estimar el error rigurosamente. Soluci on sugerida. a) La matriz A es real y sim etrica, entonces posee un sistema completo de vectores propios Q := [u1 u2 u3 ]. Seg un el Teorema de Gershgrorin hay dos valores propios, 2 y 3 , en el intervalo [1, 4]; el tercer, 1 , de valor absoluto m aximo, pertenece al intervalo [8, 12]. Ahora sea Aui = i ui . Supongamos que 1 = 0 en la representaci on x0 = 1 u1 + 2 u2 + 3 u3 . En este caso, tendriamos Ax0 = 2 2 u2 + 3 3 u3 , entonces Ax0 m ax |2 |, |3 | x0 4 x0 = 4,

130

5. EL PROBLEMA DE VALORES PROPIOS DE UNA MATRIZ

mientras que Ax0 = (1, 1, 10)T 10,

una contradicci on. Entonces 1 = 0, y el vector x0 es apropiado. b) Sin normalizar obtenemos x1 = (1, 1, 10)T , x1 = (10, 11, 102)T , x1 = (101, 114, 1041)T .

El valor propio aproximado correspondiente es


T 1 = x2 x3 = 108446 = 10,206682. 10625 xT 2 x2

Seg un el Teorema 4.4, sabemos que para una matriz A normal (por ejemplo, A sim etrica), entonces existe un valor propio j = 0 de A con j j Ax x , Ax 2

donde x y son aproximaciones del vector y del valor propio, respectivamente. Aqui 1 = 10,206682, y x3 = Ax para obtener hay que usar x = x2 , = 1 j j Entonces, 1| |j 0,001931|j | 0,001931 12 = 0,023167. x3 x2 x3 2
2

2,031146 = 0,001931. 1052,0827

Ejemplo 5.9 (Certamen 2, Curso 2010). Se considera la matriz 10 1 1 1 A = 1 2 1 1 10

a) Demostrar que A posee tres valores propio 1 < 2 < 3 , en particular 1 , 2 , 3 R, y que x0 := (0, 0, 1)T es un vector apropiado para iniciar la iteraci on directa (m etodo de von Mises). b) Determinar x3 (usando el algoritmo b asico, sin normalizar), y calcular usando x3 y x2 un valor aproximado del valor propio. Para esta aproximaci on estimar el error rigurosamente. Soluci on sugerida. a) Dado que A es sim etrica, sus valores propios son reales, y los c rculos de Gershgorin son K1 = [12, 8], K2 = [0, 4], K3 = [8, 12].

Dado que Ki Kj = para i = j , cada uno de los c rculos contiene exactamente un valor propio, es decir i Ki para i = 1, 2, 3. En el presente caso, aun no podemos

DIRECTA SEGUN VON MISES Y EL METODO 5.6. LA ITERACION DE WIELANDT

131

decidir si el valor propio de valor absoluto m aximo pertenece a K1 o a K3 . Para obtener m as informaci on, calculamos el polinomio car acteristico: 10 1 1 1 2 1 p(; A) = det(A I) = = 3 + 22 + 103 200. 1 1 10 Debido al signo del coeciente de 3 reales, concluimos que >0 =0 < 0 p() = 0 >0 =0 <0 y sabiendo ya que hay tres valores propios distintos para para para para para para para < 1 , = 1 , 1 < < 2 , = 2 , 2 < < 3 , = 3 , > 3 .

Ahora, evaluando p(2; A) = 6 > 0 concluimos que 2 < 2. Por otro lado, la traza de A es la suma de sus valores propios. En nuestro caso, 1 + 2 + 3 = 2, es decir 1 + 3 = 2 2 > 0, es decir 1 > 3 . Dado que 1 < 0 y 3 > 0, esta desigualdad implica que |1 | = 1 < 3 = |3 |. Por lo tanto, 3 es el valor propio de mayor valor absoluto. Como A es sim etrica, A posee un sistema de vectores propios ortonormales. Sean u1 , u2 , u3 los vectores propios correspondiente a los valores propios respectivos 1 , 2 , 3 . Sea x0 = 1 u1 + 2 u2 + 3 u3 . De acuerdo al Teorema 5.13 hay que demostrar que 3 = 0. Ahora, si fuera 3 = 0, se tendria que R(x0 ; A) = R(1 u1 + 2 u2 ; A) =
T (1 uT 1 2 u2 )(1 1 u1 + 2 2 u2 ) 2 2 1 + 2 2 2 2 2 1 1 + 2 2 1 2 = = + 2 2 2 2 1 2 2 2 [1 , 2 ] [12, 2]. 1 + 2 1 + 2 1 + 2

Pero, efectivamente, R(x0 ; A) = (0, 0, 1)(1, 1, 10)T = 10 [12, 2], es decir 3 = 0; por lo tanto, x0 es apropiado. b) Iterando obtenemos 1 1 1 , x2 = Ax1 = 13 , x1 = Ax0 = 10 102 105 x3 = Ax2 = 129 , 1034

132

5. EL PROBLEMA DE VALORES PROPIOS DE UNA MATRIZ

y el valor propio aproximado 3 = R ( x2 ; A ) = 3 xT 2 x3 = 10,1428. T x2 x2

Para estimar el error, utilixamos la parte (iii) del Teorema 5.4. Aqu esto signica que existe un valor propio j de A tal que 3 j j Ax2 10,1428x2 Ax2 2
2

x3 10,1428x2 94,9019 = = 0,0906. x3 2 1047,3

Mejores cotas son posibles.

Evidentemente, j = 3, y puesto que 3 [8, 12], llegamos a 3 | 12 0,0906 = 1,0874. |3

Nos damos cuenta que la velocidad de convergencia de la iteraci on directa depende decisivamente del cuociente |2 /1 | < 1. Ahora, sean 1 , . . . , n los valores propios de A. Si es una aproximaci on del valor propio i y 0 < |i | < |j | para todo j {1, . . . , n}\{i}, entonces A I es regular y los valores propios k = 1/(k ) de (A I)1 satisfacen Ademas, m axi=j |j |/|i | es peque no cuando la aproximaci on del valor propio era buena. Entonces, la iteraci on directa aplicada a (A I)1 converge rapidamente. Esta es la idea del m etodo de interaci on inversa, conocida tambi en como M etodo de Wielandt. La observaci on decisiva para su ejecuci on es que no hay que calcular expl citamente la matriz (A I)1 ; al lugar de eso, en cada paso se resuleve el sistema lineal 1 k+1 , (A I) xk+1 = xk , xk+1 := x k+1 x lo que cuesta poco esfuerzo computacional si una vez por siempre se ha calculado una descomposici on triangular (por lo menos, con b usqueda del privot en la columna) de A I. Entonces, si P(A I)Q = LR, calculamos para k N0 sucesivamente los vectores zk , vk , k+1 : wk y x zk = Pxk , Lvk = zk , Rwk = vk , k+1 = wk . QT x k+1 , podriamos calcular una nueva aproximaci Ahora, usando la nueva aproximaci on x on del valor propio, determinar una nueva descomposici on triangular, etc. Pero, en general, el esfuerzo computacional para realizar eso es exagerado. El siguiente teorema provee informaci on acerca de un vector inicial apropiado. Teorema 5.14. Sea A Cnn diagonalizable, Aui = ui , i = 1, . . . , n, ui 2 = 1 para todo i, donde U = u1 . . . un es el sistema completo de vectores propios de A y P(A I)Q = LR con matrices de permutaci on P y Q, L una matriz triangular inferior con diagonal (1, . . . , 1) y elementos de valor absoluto 1, y R una matriz triangular superior. Adem as denimos el ndice s a trav es de |
ss |

j {1, . . . , n}\{i} :

|i | > |j |.

= m n |
1 i n

ii |,

DIRECTA SEGUN VON MISES Y EL METODO 5.6. LA ITERACION DE WIELANDT

133

:= diag( yR
1 j n

1 11 , . . . ,

m n |j |

1 nn )R.

En este caso, si no es un valor propio, sabemos que


2

n|

ss | cond

(U)

1 j n

m n |j | L1
ss e1

1 R

cond

(U) n.

Si denimos x1 por RQT x1 = (lo que corresponde a x0 :=


ss P T

Les ), y el ndice k por


n

|k | = m ax |i |,
1 i n

donde x1 =
i=1

i ui ,

entonces sabemos que el valor propio corespondiente k satisface |k | n3/2 |


ss | cond
2

(U).

Eso signica que si los valores propios de A son sucientemente separados (comparado con m n1 j n |j |), entonces |k | = m n |j |
1 j n

y x1 es una aproximaci on apropiada para iniciar la iteraci on inversa. Demostraci on. Cambiar el valor cambiar A I a
ss

en la descomposici on triangular a cero es equivalente a


ss P T T Les eT sQ ,

y la matriz B es singular. En este caso, el Teorema 5.7, aplicado al valor propio cero de B y el valor propio i de A, entrega que n| ss | cond 2 (U) |i | = 0 (i ) cond 2 (U)| ss | PT Les eT sQ 2 para un ndice i apropiado. Ahora, sea j0 denido por
1 i n

B := A I

|j0 | = m n |i |. Dado que uj0


2

= 1, sabemos que (A I)1 uj0 = 1 uj , j0 0 L1


2

lo que implica que 1 |j0 | es decir,

(A I)1 |
ss |

= (PT LRQT )1

1 R

1
2

ss |

1 j n

m n |j | L1 1 x1

1 2 . R

Seg un la denici on de x1 y del ndice k , tenemos que


2

n|k |.

134

5. EL PROBLEMA DE VALORES PROPIOS DE UNA MATRIZ

Adem as,
n

x0 =
i=1

i (i )ui ,
ss P T

|k ||k |

U1 x0 n3/2 |

= U1 U1
2

Les

|k |

ss |

n3/2 |

ss | cond

ss |
2

U1 (U).

n,

Si la descomposici on triangular se ha ejecutado con b usqueda del pivote en la matriz, satisfacen ii = 1 y | ij 1. En este caso, L1 2 R 1 2 puede entonces los elementos de R ser estimado como una funci on solamente de n. Si = j para un ndice j , entonces ss = 0 y x1 mismo es el vector propio asociado. Vemos que ss converge linealmente a cero con respecto a m n1 j n |j |. Pero no aparecen problemas num ericos al determinar x1 . Tambien aqu se puede formular un teorema an alogo al Teorema 5.12, es decir, cuando j f (n), tenemos x1 uj hasta un error del tipo O(), pero donde hay t erminos posiblemente grandes como amplicadores del error. Ejemplo 5.10 (Tarea 29, Curso 2006). Se considera la matriz 1000 10 1 1 0 0 1000 10 1 10 1 = LR. A = 1000 20 2 = 1 1 0 0 0 0 1 1 1 1 1000 20 3

a) Ejecutar un paso del m etodo de Wielandt para determinar el valor propio m as peque no T de A A usando = 0. Elegir el vector inicial para la iteraci on de Wielandt como (a, b, c)T , a, b, c = 1 de tal forma que (R1 )T x0 sea lo m as grande posible. b) Determinar una cota inferior realista para A1 2 . Soluci on sugerida. a) Usamos que 0,001 0,001 0 0,001 0 0 0,1 0,1 , (R1 )T = 0,001 0,1 0 , R1 = 0 0 0 1 0 0,1 1 entonces x0 = (1, 1, 1)T . Aprovechando que AT A = RT LT LR, podemos resolver el sistema AT Ax1 = x0 con 3000000 50000 6000 900 110 AT A = 50000 6000 110 14 para obtener 0,0014 x1 = 0,3604 . 2,3010

DIRECTA SEGUN VON MISES Y EL METODO 5.6. LA ITERACION DE WIELANDT

135

Entonces tenemos que R(x1 , AT A) = xT xT 1 x0 1 Ax1 = = 0,4909, T x 1 x1 xT 1 x1

lo que representa un valor aproximado del valor propio menor de AT A. T b) Sabemos que el valor propio mas peque no de AT A satisface m 0,4909, n (A A) 1 T T T entonces (A A ) 2,0371. Dado que para cada matriz B, BB y B B tienen los mismos valores propios, sabemos ahora que A1
2

T A1 ) = m ax (A

1 T ) m ax (A A

2,0371 = 1,4272.

Ejemplo 5.11 (Certamen 2, Curso 2010). Se 10 A= 1 1

a) Demostrar sin calcular el polinomio caracter stico que A tiene tres valores propios reales distintos, 1 < 2 < 3 . b) Partiendo de x0 = (1, 1, 1)T , y eligiendo un valor {7, 1, 8} apropiado (con justicaci on), calcular un paso de iteraci on inversa (m etodo de Wielandt) para determinar una mejor aproximaci on del vector propio que corresponde a 2 . c) Utilizando el resultado de (b), calcular una mejor aproximaci on de 2 . Soluci on sugerida. a) Los c rculos de Gershgorin son K1 = z C |z + 10| 2 , K3 = z C |z 13| K2 = z C |z 2| 2 . 2 ,

considera la matriz 1 1 2 1 . 1 13

Dado que Ki Kj = para i = j , cada uno de los c rculos contiene exactamente un valor propio, es decir i Ki , i = 1, 2, 3, por lo tanto i = j para i = j . Adem as, los valores propios deben tener partes reales diferentes, por lo tanto sabemos que 1 [12, 8], 2 [0, 4] y 3 [11, 15]. b) De los tres valores propuestos, se debe escoger aqu el que est a mas cerca de 2 que de 1 o 3 . De acuerdo al resultado anterior, sabemos que |1 + 7| 5, 9, 16, |1 1| |2 + 7| 7, 3, 4, |2 1| |3 + 7| 18; 10; 3. |3 1|

c) Una mejor aproximaci on de 2 est a dada por R(x1 ; A) = 2. (Los verdaderos valores propios de A son 1 = 10,0398, 2 = 1,9925 y 3 = 13,0473.)

Solamente el valor = 1 est a mas cerca de 2 que de 1 o 3 . La iteraci on inversa consiste en resolver el sistema (A I )x1 = x0 , en este caso 11 1 1 1 0 1 1 1 x1 = 1 , con el resultado obvio x1 = 1 . 1 1 12 1 0

|1 8|

|2 8|

|3 8|

También podría gustarte